৫. পুরানো প্রশ্ন ও উত্তর(১২ সেপ্টেম্বর ২০১৯ থেকে ২৯ জানুয়ারি ২০২১)

তোমরা নিশ্চয়ই লক্ষ করেছ প্রশ্ন করার আগে আমি সবাইকে তাদের নাম ঠিকানা দিতে বলেছি এতোদিন সেটা নিয়ে বেশি মাথা ঘামাইনি। কিন্তু আমি যেহেতু চাই তোমরা সবাই দায়িত্বশীল হয়ে বড় হও তাই আজ থেকে নাম ঠিকানা না থাকলে উত্তর দিচ্ছি না। যদি কেউ কিছু বলতে চায় তাহলে নিজের পরিচয়টা দিতে হয় তা না হলে সেটাকে কেউ গুরুত্ব দেয় না, এটাই দুনিয়ার নিয়ম এবং এটাই ভদ্রতা। যদি কখনো কিছু সেনসিটিভ হয় আমি নিজেই উত্তর দেবার সময়য় তার নাম ঠিকানা লিখি না, দরকার হলে তোমরাও আমাকে বলে দিতে পার তোমাদের নাম ঠিকানা গোপন রাখার জন্য, আমি গোপন রাখব।

২৯ জানুয়ারি ২০২১

প্রশ্ন: মানুষ কি চাইলে সবকিছু করতে পারে?Is everything possible by Homo sapiens? হৃদয় আহমেদ স্বাধীন,ঝিনাইদহ
উত্তর: প্রশ্নটা ঠিকভাবে করতে হলে তোমাকে আগে বোঝাতে হবে ‘সবকিছু’ বলতে তুমি কী বুঝিয়েছ।

প্রশ্ন: স্যার, কম্বিনেটরিক্স আর জ্যামিতির দুইটার মধ্যে যদি একটা পড়তে হয় তাহলে কোনটা বেছে নেবেন? আহমেদ ইত্তিহাদ হাসিব লালমাটিয়া, ঢাকা
উত্তর: আমি? আমার পড়াশোনা পঞ্চাশ বছর আগে শেষ। এখন যখন পড়ি, সখে পড়ি, কোনো বাধ্যবাধকতা নেই। যেটা ইচ্ছে হয়, তার সব পড়ি।

প্রশ্ন: আমি দশম শ্রেণীর একজন ছাত্রী l আমি আইনস্টাইনের থিওরি অফ রিলেটিভিটি এবং থিওরি অফ রিলেটিভিটি অনুসারে কিভাবে টাইম ট্রাভেলিং হয় সেটি আমার প্রিয় লেখক প্রফেসর ডঃ মুহাম্মদ জাফর ইকবাল স্যারের কাছ থেকে জানতে চাই I থিওরি অফ রিলেটিভিটি এবং টাইম ট্রাভেলিং এর ব্যাপারে আমার রয়েছে অনেক কৌতুহল l প্লিজ আমার প্রশ্নটার উত্তর দিবেন I প্লিজ প্রশ্নটার উত্তর দিবেনl আমি আপনার কাছে আরও অনেক অনেক অনেক প্রশ্ন করতে চাই I আমার নাম সাদিয়া হোসেন সারা I আমার বাসার ঠিকানা ঢাকা
উত্তর: থিওরি অফ রিলেটিভিটি পড়ে তুমি তো টাইম ট্রাভেলিং সম্পর্কে জানবে না, time dilation সম্পর্কে জানবে। তোমার থিওরি অফ রিলেটিভিটি নিয়ে আগ্রহ আছে জেনে খুব খুশি হলাম। খুবই মজার বিষয়। আমার লেখা ‘থিওরি অফ রিলেটিভিটি’ নামে একটা বই আছে, খুঁজে দেখ পেয়ে যাবে। আমার কাছ যত ইচ্ছা প্রশ্ন করতে পার, তবে লক্ষ করে দেখ, আমি কিন্তু চাই আগে তুমি নিজে এর উত্তরটা খুঁজে বের করতে চেষ্টা করবে।

প্রশ্ন: Sir, বিজ্ঞানের ভাষায় স্রষ্টা ও সৃষ্টির মধ‍্যে কী পার্থক‍্য? Oitri Saha New Bailyroad,Dhaka
উত্তর: বিজ্ঞানের domain এর ভেতর যেগুলো নেই বিজ্ঞান সেগুলো জানে না, সেগুলো ব্যাখ্যা করার চেষ্টাও করে না।

প্রশ্ন: Assalamualaikum Sir I am Mazharul Islam. I live in khilkhet Dhaka
You are one of my Favorite Writer. My question is are you going to write anymore parts of “Tuntuni Ar Chhotachu”. I have read 3 of them I liked all of them so can we expect anymore?
উত্তর: এই ওয়েব সাইটে আমার বইয়ের তালিকা আছে। ওখানে দেখ টুনটুনির উপর আরো কয়েকটা বই আছে।

প্রশ্ন: Sir amra zodi panite haat vigie ni tahole Agon touch korle haat pore na keno. Name:Bivore. Class:3 Faridpur.
উত্তর: আমি কিছু একটা উত্তর দিতে পারি, কিন্তু আমি চাই না কেউ হাত পানিতে ভিজিয়ে আগুনের মাঝে ঠেসে ধরুক! জেনে রাখ, হাত আগুনে ধরে রাখলে আগে হোক পরে হোক হাত পুড়বেই! আগুন থেকে সাবধান!!

প্রশ্ন: সবল নিউক্লিয় বল কার্যকর হয় এমন পরিসরে কোনো অণুজীব বাস করে কী ? -মোঃ রেদওয়ানুল ইসলাম মাইজদী,নোয়াখালী।
উত্তর: আমি ‘হ্যাঁ’ কিংবা ‘না’ বলে এক শব্দে উত্তর দিতে পারি, কিন্তু আমি চাই তুমি ইন্টারনেটে এর উত্তরটা বের কর। range strong force আর smallest size of life form লিখে একটা সার্চ দাও।

প্রশ্ন: আসসালামু আলাইকুম,স্যার।কেমন আছেন? আজকাল দেখা যায় অনেকেই ফেসবুক,ইউটিউবসহ বিভিন্ন সামাজিক যোগাযোগ সাইটে বাংলিশ ভাষায়(ইংরেজি অক্ষরে বাংলা) লেখালেখি করেন(কিংবা করে)।এটা আমাদের বাংলা ভাষার জন্য কতটা সম্মানজনক?সালাম,রফিক,জব্বার, বরকত,শফিউর,রিকশাচালক আউয়াল এবং পথশিশু অহিউল্লাহ কি এর জন্যই জীবন দিয়েছেন? আকিফ,জামালপুর
উত্তর: একসময় মানুষজন কোনো উপায় না পেয়ে এগুলো করতো। এখন কোনো কারণ নেই, বরং যারা এভাবে লিখে সবাই ভুরু কুঁচকে তাদের দিকে তাকায়। কাজেই আমি ধরে নিয়েছি যারা সুযোগ থাকার পরেও এগুলো করে তারা আমাদের এত সুন্দর ভাষাটার সৌন্দর্য উপভোগ করতে পারছে না। তাদেরকে করুণা কর। আর যারা উপায় না পেয়ে এটা করছে, তাদের একটু সময় দাও।

প্রশ্ন: অনেকে যখন প্রশ্নের উত্তর না পেয়ে মন খারাপ প্রকাশ করে, তখন কি আপনার মন খারাপ হয়? আপনার উত্তর দেয়ার জন্য প্রশ্ন বাছাই করার প্রক্রিয়া কী? আপনার চিঠিপত্র ম্যানেজ করার জন্য অ্যাসিস্টেন্ট আছে? আমি আপনার অ্যাসিস্টেন্ট হতে চাই, যদিও আমার বয়স অনেক কম! আমিও আপনার উত্তর পাইনি এর আগেরবার,তারপর একটু মন খারাপ হয়েছে। স্যার, আপনার সাথে আবার দেখা হলে কি আপনি আমার মাথায় হাত বুলিয়ে দেবেন? আমি পড়ালেখায় ভালো না, কোনকিছু পারিও না ভালো, ছোটবেলায় এসব চিন্তা করলে আপনার একটা কিশোর উপন্যাস পড়ে ফেলতাম, মন ভালো হয়ে যেত। জমিলা বুড়ির মত বলতে চাই,” শকুনের সমান পরমায়ু হোক” এটারও উত্তর না দিলে কষ্ট পাব অনেক। সাদিয়া আফরিন, দিনাজপুর
উত্তর: আমি যখন এটা শুরু করেছিলাম, তখন ভেবেছিলাম ছেলে মেয়েরা হয়তো সত্যি সত্যি তাদের মনের প্রশ্নগুলো করবে। কিন্তু এখন আমি আবিষ্কার করেছি যে আসলে প্রশ্নের উত্তরের বিষয়বস্তু নিয়ে ছেলেমেয়েদের বেশি মাথাব্যাথা নেই, তারা শুধু নিজের প্রশ্নের কোনো একটা উত্তর চায়। আমি দেখেছি হয়তো একজন এমন প্রশ্ন করেছে যেটা অন্যের প্রশ্নের উত্তরে দেওয়া হয়েছে। কিন্তু তাতে তারা খুশি না! তারা চায় আলাদাভাবে তাকেই উত্তর দেওয়া হোক। তুমি জানতে চেয়েছ উত্তর দেওয়ার জন্য আমার কোনো ম্যানেজার আছে কিনা! না নাই! সেই জন্যই তো এখানে এরকম দুরবস্থা। তুমি আমার এসিস্টেন্ট হয়ে সাহায্য করতে চাও জেনে খুব খুশি হয়েছি। থ্যাংকু থ্যাংকু থ্যাংকু! তুমি কিভাবে সাহায্য করতে পারবে তার একটা আইডিয়া দাও দেখি! (তোমার সাথে দেখা হলে আমি অবশ্যই তোমার মাথায় হাত বুলিয়ে দিব।)

প্রশ্ন: প্রিয় লেখক, টুনটুনি ও ছোটাচ্চুকে নিয়ে আর কোনো বই না লেখাই বোধহয় ভালো হবে।দিনে দিনে এটা রীতিমতো পানসে হয়ে যাচ্ছে। ইতি, সাগর শর্মা বয়রা,খুলনা
উত্তর: ঠিকই বলেছ। দেখি অন্যদের রাজী করাতে পারি কিনা।

প্রশ্ন: Sit ekhane ki thikana ki school er nam likte hobe na bashar thikana likte hobe?? -Zerin Khan, Peelkhana , Dhaka
উত্তর: যেকোনো একটা দিলেই হবে, আমি শুধু চাই যে সবাই নিজের পরিচয় দিয়ে কথা বলুক। কথা বলার আগে প্রথমে পরিচয় দেওয়া পৃথিবীর নিয়ম।

প্রশ্ন: Q1.Sir, apnar akta boi porte ( prai 100 prishtar ) koi din/ kotokhon lage?
2. apnar blackhole ar bachcha boitir moto ki ar kono boi ache? 3. ai link ar shurute apner je cartoon chobita ache sheta k akeche – kibhabe akeche? 4.apni niom kore a link a ashen na keno?! NAME : AFIA AZMAEEN [TEROKHADIA , RAJSHAHI]
উত্তর: ১. বইটা যদি ভালো হয় তাহলে ১০০ পৃষ্ঠার বই এক বসাতেই শেষ হয়ে যায়। তবে বইটা ভালো হতে হবে। ২. একটা বইয়ের মত অন্য বই কিভাবে হয় তাতো জানি না। কিন্তু একটু কৌতুক করে সায়েন্স ফিকশান তো আমি প্রায়ই লিখি। ৩. এটা এঁকেছে কার্টুনিস্ট নাসরীন সুলতানা মিতু, মনে হয় ড্রইয়িং প্যাড ব্যবহার করে কম্পিউটারে। ৪. আমার জীবনেই কোনো নিয়ম কানুন নাই, এখানে কেমন করে হঠাৎ নিয়ম মেনে চলব বল?

প্রশ্ন: স্যার, এস এস সি পরীক্ষার কি অটোপাশ দেওয়া উচিত না? এস এস সি জুনে হলে এইচ এস সি এর সিলেবাস শেষ করার সময় অনেক কমে যাবে না (যদি না এইচ এস সি তখন পিছিয়ে দেয়।)? আর এস এস সি এর পড়ার মূল্যায়ন তো উচ্চমাধ্যমিকের সিলেবাসেই হয়ে যায়। কেউ ভেক্টর এর যোগ বিয়োগ ঠিকভাবে না বুঝে ভেক্টরের গুণ পড়া শুরু করবে না! আবার ঠিক ভাবে রৈখিক বেগ গতি না বুঝলে কৌণিক বেগ ঠিকমতো বুঝতে পারবে না। আর তাছাড়াও আমাদের মতো যাদের শ্বাসকষ্টের অনেক সমস্যা আছে তাদের জন্য কি স্কুলে যাওয়াটা রিস্ক হয়ে যায় না? (হয়তো আমার চিন্তা ভাবনায় কিছু ভুল আছে,তাই আপনার কাছে জিজ্ঞেস করলাম। নাম দিচ্ছি না কারণ আমি এত বড় বড় মানুষের সিদ্ধান্তের বিরুদ্ধে কথা বলছি, কে আবার কি মনে করে!) অগ্রীম ধন্যবাদ।
উত্তর: মাস্ক না থাকলে যেরকম সার্ভিস নাই, ঠিক সেরকম নাম পরিচয় না থাকলে উত্তর নাই! সরি।

প্রশ্ন: স্যার আমি এবং আমার ভাই আপনার প্রচুর বই পড়েছি। এই করোনার সময় আপনার লেখা বই ছিল আমাদের সময় কাটানোর বন্ধু। স্যার আপনি যদি “লিটু বৃত্তান্ত” এবং “দলের নাম ব্ল্যাক ড্রাগন” এর মতো আরেকটি adventure এর গল্প লিখেন তাহলে খুব ভালো হয়।
নাম: ( পরিচয় গোপন রাখলে ভালো হয়।) গোপন এবং গোপন। বয়স : ১৩ ও ৮। ঠিকানা : চট্টগ্রাম।
উত্তর: তোমারা দুই ভাই আমার অনেক বই পড়েছ শুনে খুব খুশি হয়েছি। আমার সব বই ই তো ছোট বড় এডভেঞ্চারের গল্প, নূতন করে আবার কিভাবে লিখব বল।

প্রশ্ন: আসসালামু আলাইকুম স্যার। অন্ধকারে সাদা রং দেখা যায় কেন? ( নুসরাত ইসলাম, চট্টগ্রাম
উত্তর: সাদা রঙ সবচেয়ে বেশি আলো প্রতিফলিত করে। যখন তুমি ভাবছ অন্ধকারে আছ তখনও যে অল্প একটু আলো আছে সেটা সাদা রঙ থেকে প্রতিফলিত হয় বলে তুমি দেখতে পাও। যদি একেবারে সত্যি সত্যি অন্ধকার হয় যেখানে এতটুকু আলোও না থাকে তাহলে তুমি সাদা রংও দেখবে না।

প্রশ্ন: গ্র্যাভিটি বা মহাকর্ষ বল তো এক প্রকার শক্তি, এটাকে অন্য কোন শক্তিতে রূপান্তর করা যায় না কেন? নাম: ফাহিম ফয়সাল বোয়ালিয়া, রাজশাহী
উত্তর: কেন যাবে না? আমাদের কাপ্তাইয়ের জলবিদ্যুত কেন্দ্রে কী করা হয়?

প্রশ্ন: sir,addiction জিনিসটা সবসময়ইখারাপ, কিন্তু কেউ যদি বইয়ের প্রতি addicted হয়,তাহলে??? এই যেমন একটা বই দেখলে সেটা শেষ করা পর্যন্ত অন্য কিছুতে মনই বসে না,সারাদিন খালি বইটার কথা মাথায় গুরগুর করতে থাকে, বইটা শেষ করাটাই জীবনের একমাত্র লক্ষ্যে পরিণত হয়,,,যদি কালকে আমার ssc পরিক্ষা ও থাকে কিচ্ছু আসে যায় না, আমাকে কেবল বইটা শেষ করতে হবে তখন কি করা??? (ফাহিমা শারমিন; হলি ক্রস কলেজ, ঢাকা)
উত্তর: হা হা হা! ভালোই বলেছ! অকাট্য যুক্তি। আমরা তো নিরবিচ্ছিন্ন ভাবে ভালো কিছুই পাই না, সব কিছুর মাঝে নেতিবাচক অংশ ইচ্ছা করলেই খুঁজে বের করা যায়। আমি একেবারে তিন চার বছর থেকে বাচ্চাদের বই পড়া শিখিয়ে দিতে আগ্রহী। ডাক্তারেরা জানে তাহলে বাচ্চাদের অনেক কম বয়সে চশমা নিতে হতে পারে। কিন্তু ডাক্তারেরা তারপরেও বাচ্চাদের বই পড়তে নিষেধ করে না, এটা আমেরিকায় আমি নিজের চোখে দেখেছি! কাজেই বই পড়ার addiction এর জন্য ছোটখাট ssc পরীক্ষা একটু খারাপ হলে আমরা মেনে নিব!

প্রশ্ন: আপনার লেখা গল্প বা উপন্যাস কি আমি নিজের ওয়েবসাইটে রাখতে পারব? কোনো কমার্শিয়াল উদ্দেশ্য নাই। যেন সবাই পড়তে পারে। আর একটা কথা। অনেক গুলো অ্যাপে আপনার লেখা প্রকাশ করে ডেভলপাররা আয় করছে। তাদের কি আপনি অনুমতি দিয়েছেন? নাকি তারা কপিরাইট ভঙ্গ করছে। ইবনে শফিক, পাটুরিয়া।
উত্তর: আমাকে জিজ্ঞেস না করে একজন যা কিছু করতে পারে! কিন্তু আমাকে যদি কেউ একজন জিজ্ঞেস করে কিছু একটা বেআইনি কাজের অনুমতি চায় সেটা আমি কেমন করে দিই বল!

প্রশ্ন: Assamuilikum ami mahtab abdullah jami
উত্তর: ওয়ালাইকুম সালাম। আমি মুহম্মদ জাফর ইকবাল।

প্রশ্ন: আশায জিয়া, লীডস, ইউকে বয়স: ৯ বছর, ২ মাস, ৭ দিনপ্রশ্ন: আমার বন্ধুরা সবাই ভিডিও আর কম্পিউটার গেম খেলে কিন্তু আমাকে ভিডিও আর কম্পিউটার গেম খেলতে দেয়া হয় না। কেনো খেলতে দেয়া হয়না জানো? আমার মা সবসময় বলে যে তুমি লিখেছো যে তুমি তোমার বাচ্চাদের ভিডিও গেম খেলতে দাও নাই। সেজন্য ও-ও দিবে না। ভিডিও গেম খেললে কি আসলেই বুদ্ধি কমে যায়?
উত্তর: ইশ! পৃথিবীর সব মায়েরা যদি তোমার আম্মুর মত হত তাহলে কী চমৎকার হত! এখন হয়তো তোমার একটু মেজাজ গরম হচ্ছে, কিন্তু যখন বড় হবে তখন তুমি তোমার আম্মুকে একশ বার থ্যাংকু দেবে! ভিডিও গেম খেললে নেশার মত হয়ে যায়, একটা বাচ্চা বসে বসে সিগারেট খেলে আমরা যেরকম ভয়ে আঁতকে উঠি, তাহলে বসে বসে ভিডিও খেললে কেন ভয়ে আঁতকে উঠব না? তুমি যেহেতু অনেক স্মার্ট তাই ভিডিও গেম থেকে আনন্দ পাওয়ার থেকে অন্য কিছুতে আনন্দ পাওয়া শিখে যাও। যেমন অনেক অনেক অনেক বই পড়া!!

প্রশ্ন: স্যার আপনি কেমন আছেন?আমার সবচেয়ে প্রিয় লেখক মুহম্মদ জাফর ইকবাল। আপনার লেখা দু একটা বই ছাড়া বাকি সব বই আমি পড়েছি। আমি বই পড়তে খুব ভালোবাসি।কিন্তু আব্বু আমাকে পাঠ্যবই ব্যতীত অন্য কোনো বই পড়তে দেখলেই বকাঝকা শুরু করে দেন। এর কারনটা সম্ভবত আমি সেলফোনে pdf reader ব্যবহার করে বই পড়ি বলে।কিন্তু আমার এলাকায় বা তার আশেপাশে কোনো লাইব্রেরি এবং বই পড়ে এমন কোনো বন্ধুও আমার নেই যে আমি সেখান থেকে বই সংগ্রহ করব।এখন আমি কি করব? উত্তরের সাথে আমাকে আপনার একটা অটোগ্রাফ পাঠাবেন,স্যার? প্লিজ? মোঃ আরাফাত হোসেন (বাবর), বেগমগঞ্জ, নোয়াখালী
উত্তর: বই পড়ার জন্য যদি কারো বাবা মা তাদের ছেলেমেয়েকে বকাঝকা করে তাহলে সেটা অনেক বড় ট্র্যাজেডি। বাবা মা’কে বোঝাও। পিডিএফে বই পড়া একটা ভালো সমাধান, তোমার আব্বুকে দেখাও যে তুমি ফেসবুক করছ না, সত্যিকারের বই পড়ছ। (যারা অটোগ্রাফ চাও তারা কিন্তু পোস্টকোড সহ পুরো ঠিকানা লিখে দিও যেন সেখানে চিঠি যায়।)

প্রশ্ন: sir, is energy always conserved?? I see some contradictons. As like cosmological redshift (ex-CMB) and dark energy loses and gains energy respectively. Also dark energy gains energy way more than CMB losing energy. so,is energy is not conserved?? Rayet Zarif Mirzapur Cadet College
উত্তর: অনেক সুন্দর প্রশ্ন করেছ। আমরা ছোট স্কেলে energy conservation দেখি, কিন্তু বড় স্কেলে দেখি না। আমরা বিশ্বব্রহ্মান্ডের মাত্র ৪% ব্যাখ্যা করতে পারি, ভবিষ্যতে কী দেখব জানি না। কিন্তু এই মুহূর্তে energy conservation নেই।

প্রশ্ন: ২+২=৫ প্রমাণ করা যায় কেন? সাধারণ যুক্তিতে তো কিছুই বুঝছি না। ইমতিয়াজ আহমেদ, বরিশাল
উত্তর: তুমি কোথায় দেখেছ ২+২=৫ প্রমাণ করা যায়? যায় না। যদি যেতো তাহলে পুরো গণিতটাই তো ধ্বসে যেতো। অনেক সময় ফাঁকি ঝুকি দিয়ে এগুলো করে, কিন্তু সেগুলো তো সত্যি না, মজা করার জন্য।

প্রশ্ন: স্যার…ভার্সিটির ক্ষেত্রে কি পাবলিক,প্রাইভেট, ন্যাশনাল এই সকল কিছু ম্যাটার করে? ভর্তির ক্ষেত্রেএই অসুস্থ প্রতিযোগিতা কেন স্যার?? নামঃ তানজিম হাসান ঠিকানাঃ যাত্রাবাড়ী,ঢাকা
উত্তর: পয়সা কড়ির দিক দিয়ে হয়তো একটু করে। খারাপ প্রাইভেট ইউনিভার্সিটি যেরকম আছে, খারাপ পাবলিক ইউনিভার্সিটিও সেরকম আছে। তবে ভবিষ্যতে একজনকে তার বিশ্ববিদ্যালয়, বিভাগ কিংবা গ্রেড দিয়ে আর বিবেচনা করবে না, কতটুকু জানে, কতটুকু উৎসাহী সেটা দিয়ে বিবেচনা করবে সেটা আমি বলতে পারি।

প্রশ্ন: sir, 1.apni ki COVID-19 a ghore botho kishor-kishori der akgheami nie akta golpo likhben? ai CORONA -r shomoe apnader lekha boi gulo -e amader anondo die , onnorokom kichu hole khub moja petam. 2.sir,apnar ki banglake English e likhle porte koshto hoe ? AFIA AZMAEEN ,( Terokhadia ,Rajshahi )
উত্তর: এই কোভিড হচ্ছে একটা বিভীষিকা, আমি এটার কথা ভুলে থাকতে চাই। মনে হয় না কখনো এটা নিয়ে কিছু লিখব!

প্রশ্ন: পেয়ে গেছি আপনার নাম! বাবুল!!! ১ ঘণ্টা টানা খুঁজে অবশেষে লক্ষ্য পূরণ হল। মুহা – হা -হা -হা!!!
উত্তর: নাম পরিচয় নেই তো উত্তর নেই।

প্রশ্ন: আশা করি ভালো আছেন স্যার। আমার প্রশ্ন টা ছিল আপনার একটা বই নিয়ে। প্রশ্ন টা হলো “রঙিন চশমা ” বইয়ের প্ল্যানচেট এর ব্যাপারগুলো কি সত্যি ছিলো? (অপরাজিতা, রংপুর। )
উত্তর: হ্যাঁ সত্যি। কিন্তু দেখ, আমি কিন্তু কখনো এগুলোকে অলৌকিক কিছু বলিনি। আমার ধারনা এটা সাইকোলজিক্যাল কোনো ব্যাপার স্যাপার।

প্রশ্ন: Hasin, Dhaka. Sir, have you observed a thing? You gave the answers of questions on 28 Dec. then after 3 days 31 Dec. Then after 8 days 8 January and then after 13 days 21 January. The interval of answering the questions is increasing 5 days every time ! I think you will next answer the questions on 8 February !
উত্তর: ভেরি গুড, গবেষণা চালিয়ে যাও। তোমার গবেষণা অনুযায়ী কখন আমি আবার ডুব দেব?

প্রশ্ন: সবাই কেন মনে করে anime খারাপ? হ্যাঁ, মানলাম বেশিরভাগ ক্ষেত্রেই ওর কন্টেন্ট আমাদের দেশের কালচারাল স্ট্যান্ডার্ডে পরে না, আর অনেক খারাপ দিক ও আছে, মানে, যাইহোক। কিন্তু তাই বলে ভালো দিক গুলোর কি কোনোই গুরুত্ব নেই? অ্যাটাক অন টাইটান বা ডেথ নোট তো বিশ্বমানের। বলিউড বা হলিউড নিয়ে সমস্যা না থাকলে anime নিয়ে আছে কেন? জাপান দেশটা কি খারাপ? মোস্তফা কামাল, কুমিল্লা। (উত্তর পাঠাতে আপনার কিছু অস্বস্তি থাকতে পারে। সেরকম হলে আপনি ব্যাক্তিগতভাবে উত্তরটা পাঠাতে পারেন/ এই মেইল অ্যাড্রেস এ—… … … …
উত্তর: উত্তর পাঠাতে অস্বস্তি কেন হবে? কোনো অস্বস্তি নেই। উত্তর হচ্ছে, anime জিনিষটা কী সে সম্পর্কে আমার কোনো ধারণাই নেই! এটা না জেনেই আমার জীবন ভালোই চলে যাচ্ছে।

প্রশ্ন: স্যার এটা কিন্তু ঠিক না আমি প্রশ্ন করছি এবং নাম ঠিকানাও লিখেছি কিন্তু আপনি উত্তর দেননি। প্রশ্ন করি ১৮/১/২০২১ সালে। আহনাফ প্রত্যয়, টাঙ্গাইল।
উত্তর: তা হলে তুমি ধরে নেও, তোমার কপালটা খারাপ!

প্রশ্ন: স্যার, আমি ইরফান সাদিক। পড়াশোনা করছি বাগেরহাট সরকারি উচ্চ বিদ্যালয়ে। আমার প্রশ্নঃ সমযোজী বন্ধন কি শুধুমাত্র অধাতব পরমাণুর মধ্যে সংগঠিত হয়? আপনি কি Interstellar (২০১৪) মুভিটা দেখছেন? দেখে থাকলে কেমন লেগেছে??
উত্তর: are covalent bonds always between nonmetals? লিখে ইন্টারনেটে একটা সার্চ দাও। আমি চাই তোমরা নিজেরা উত্তর খুঁজে বের করা শিখে নাও। Interstellar দেখি নাই। আজকাল মুভি দেখার সময় পাই না।

প্রশ্ন: স্যার আপনি কোথায়? উত্তর দেন না কেন?
উত্তর: নাম পরিচয় নেই তো উত্তর নেই।

প্রশ্ন: স্যার আমি দ্বাদশ শ্রেণীতে পড়ি। স্যার আমার সমস্যা আমি সব সময় নিজেকে তুচ্ছ মনে করি। আমার সবসময় মনে হয় আমার দ্বারা কিছু হবে না ইদানীং কালে দেখা গেছে আমার কোনো কাজে (পড়াশুনায়) মন বসে না। স্যার আমার কি করা উচিৎ আসমা চট্টগ্রাম
উত্তর: তুমি যেহেতু এত কষ্ট করে এই ওয়েব সাইটে ঢুকে আমাকে একটা বিষয় জানিয়েছ, তাই আমি ধরে নিচ্ছি তোমার কাছে আমার কথার একটুখানি হলেও গুরুত্ব আছে। সেজন্য বলছি, তুমি কী আমাকে একটু খুশি করে দেবে প্লিজ? সেটা হচ্ছে তুমি কী আমাকে কয়দিন পরে আবার কী লিখে জানাতে পারবে, “স্যার আমি কী বোকা ছিলাম, নিজেকে তুচ্ছ ভাবতাম! আমি দেখেছি আসলে আমি মোটেও তুচ্ছ না! আপনি ঠিকই বলেছেন, আমি এখন সবসময় উৎসাহে টগবগ করি, লেখা পড়া করি, বই পড়ি, খেলি, বন্ধুদের সাথে সময় কাটাই। জীবনটা কী চমৎকার, কত কী করা যায়। বড় হয়ে কী করব, মনে মনে সেটা ঠিক করছি! থ্যাংকু স্যার আমাকে বলার জন্য যে আমি তুচ্ছ না!”
আসমা, তুমি মোটেও তুচ্ছ না, তুচ্ছ কেন হবে? কী বলছ?

প্রশ্ন: স্যার, কেমন আছেন? আমার প্রথম প্রশ্নঃ আমি একজন মজার মানুষ হতে চাই। আমি কি করতে পারি? আমার দ্বিতীয় প্রশ্নঃ আমি আমার অনুধাবন দক্ষতা কিভাবে বাড়াবো? আফিফা,চট্টগ্রাম।
উত্তর: মজার মানুষ কথাটার অর্থ একেকজনের কাছে একেক রকম হতে পারে। তবে তুমি যদি খুব ভালো একজন মানুষ হও, সব সময় তোমার মুখে হাসি লেগে থাকে তাহলে সবাই নিশ্চয় তোমাকে মজার মানুষ হিসেবে ধরে নেবে। অনুধাবনের দক্ষতা বাড়ানোর আমি শুধু একটা নিয়মই জানি, নিজে নিজে পড়ে বোঝা। প্রাইভেট টিউটরের কাছ কিংবা কোচিং সেন্টারে না যাওয়া।

প্রশ্ন: আকিফ,জামালপুর। আসসালামু আলাইকুম স্যার।কেমন আছেন?আমার বয়স ১৪ বছর।আমি এ পর্যন্ত ৩ টা গল্প এবং ৪টা কবিতা লিখেছি।কিছুদিন আগে একটা উপন্যাস লেখা শুরু করেছিলাম।খুবই ইন্টারেস্টিং উপন্যাস!কিন্তু ২ অধ্যায় লেখার পর আর লিখতে ইচ্ছা করছে না।আপনার ক্ষেত্রেও কি কখনো এমন হয়?
উত্তর: না, হয় না। তবে আমি যত লিখেছি প্রায় তার সমান লেখা ফেলে দিয়েছি! আর লেখার ইচ্ছে করছে না সে জন্য নয়, লিখে মনে হয়েছে লেখাটা ভালো হয়নি, সে জন্য।

প্রশ্ন: আপনি কী আপনার ভাই হুমায়ুন আহমেদ এর ভক্ত? আর আপনারা কয় ভাই-বোন? আর আপনার হুমায়ুন আহমেদ এর পড়া কয়েকটি সেরা বইয়ের নাম দিবেন প্লিজ MD SHIHAD AHMED CLASS:8, Cumill,Homna
উত্তর: হ্যাঁ, সবার মত আমিও হুমায়ুন আহমেদ এর ভক্ত। তোমার কী ধরনের বই পছন্দ সেটা তো জানি না, বাচ্চাদের বই পড়তে চাইলে ‘বোতল ভূত’ পড়। মুক্তিযুদ্ধের বই পড়তে চাইলে ‘জোছনা ও জননীর গল্প’ পড়। রহস্য গল্প পড়তে চাইলে ‘দেবী’ পড়। ভূতের বই পড়তে চাইলে ‘অতিপ্রাকৃতিক’ পড়। সায়েন্স ফিকশন পড়তে চাইলে ‘ইরিনা’ পড়। তার শৈশবের কথা জানতে হলে ‘আমার ছেলেবেলা’ পড়।

প্রশ্ন: আমার নাম অলোক মজুমদার। E-mail: majumdar.alok@yahoo.com আমি গনিত নিয়ে পড়ছি। কিছুদিন আগে আমি মানসিকভাবে খুব খারাপ অবস্থার মধ্যে দিয়ে গিয়েছি। আপনার লেখা বইগুলো আমাকে বাচিয়েছে। সাথে আমার বড় বোনের সাহায্য ছিল। ওই সময়েই আমার দেশের বাড়ি বরিশালে আমার পাশের গ্রামে একটা ছেলে আত্মহত্যা করে। সে যদি আপনার বইগুলো (আমি তপু , আমার বন্ধু রাশেদ, দিপু নাম্বার টু, টুনটুনি ও ছোটাচ্চু , দস্যি ক’জন, নিতু আর তার বন্ধুরা) পড়তো তাহলে হয়তো এটা করতো না। এই বইগুলো যেকোন বয়সের মানুষের মনে দাগ কাটে। আমার মনে হয় আপনি যদি প্রত্যন্ত গ্রামের শিশুদের বই পড়ার সুযোগ তৈরি করেন তাহলে খুব ভালো হয়। স্যার আপনার প্রতি ভালোবাসা অবিরাম।
উত্তর: তোমার কঠিন সময়ে আমার বই পড়ে এক্টুখানি হলেও সাহায্য হয়েছে জেনে ভালো লাগল। আমিও চাই প্রত্যন্ত এলাকার ছেলেমেয়েরাও যেন বই পড়ার সুযোগ পায়। সে জন্য চেষ্টাও করি। দেখি কি করা যায়।

২১ জানুয়ারি ২০২১

প্রশ্ন: স্যার আমি আগে প্রশ্ন করেছি এবং নাম ঠিকানাও লিখেছি কিন্তু আপনি উত্তর দেননি সেই প্রশ্নের উত্তর জানার খুব ইচ্ছে। প্রশ্ন এখন মনে নেই যে আবার করব । আপনি একটু খুজে আমার প্রশ্ন বের করে উত্তর দেবেন প্লিজ। স্যার আমার নামে একটি অটোগ্রাফ লিখে পাঠাবেন প্লিজ। আমার নাম আহনাফ প্রত্যয়। আহনাফ প্রত্যয়, টাঙ্গাইল।
উত্তর: তোমার যেহেতু প্রশ্নটি মনে নেই, সেজন্য ধরে নিচ্ছি সেটার উত্তর তোমার জন্য নিশ্চয়ই তেমন গুরুত্বপূর্ণ নয়। ধরে নাও এই সাইটের অসংখ্য প্রশ্নের কোনো একটি ছিল তোমার প্রশ্ন এবং সেই উত্তরটি পড়ে তুমি ধরে নাও তোমার প্রশ্নের উত্তর তুমি পেয়ে গেছ। অটোগ্রাফ পাঠাতে হলে আমার একটা ঠিকানা দরকার।

প্রশ্ন: স্যার, আমি আপনার উপর অনেক রাগ করেছি। আপনার ” দলের নাম ব্ল্যাক ড্রাগন” গল্পের মিথিলা আর আমার বয়স সমান। আমি মোটেও নাকি সুরে আমার ভাইয়ের জন্য আমার মাকে নালিশ করিনা। এবং আমার বিশ্বাস যে আমার বয়সে কোন মেয়েও করেনা! আমার ভাই কে শায়েস্তা করার জন্য আমার মায়ের দরকার কি? আমার বয়সী একটা মেয়ে নিজেই নিজের বড় ভাইকে আস্ত সমস্যায় ফেলতে পারে। (উদাহরণ জানতে চাইবেন না কারণ আমাদের শায়েস্তা করার পদ্ধতিটা কারোরই পছন্দ হবে না!) আমার মাকে আমার নাকিসুরে নালিশ করার দরকারটা কি?! স্যার আমি এই ব্যাপারটা একদম পছন্দ হয়নি! মিথিলার বয়স আরেকটু কম দিলে ঠিক ছিল। এটা আমাদের বয়সী সকল মেয়েদের জন্য অপমান ! (স্যার আপনি কি রাগ করলেন, প্লীজ রাগ করবেন না। এইটা আমার একদম পছন্দ হয়নি তাই আপনাকে খালি জানালাম। স্যার কিছু মনে করবেন না।) স্যার আপনার প্রিয় পাঠক, ফিওনা সাদাত ঠিকানা : লক্ষীপুর, রাজশাহী
উত্তর: হা হা হা! মেয়েদের নিয়ে ঠাট্টা করে কিছু একটা লিখলে মেয়েরা রাগ করে। ছেলেদের নিয়ে তামাশা করলে ছেলেরা রাগ করে! এখন আমি যাই কোথায়? মনে হয় এখন থেকে গরু ছাগলদের চরিত্র নিয়ে লিখতে হবে- তাদেরকে নিয়ে যতই তামাশা করি না কেন, তারা তো আর আমার বই পড়ে না, তাই তারা রাগও হবে না। কিংবা শুধু রবোটদের নিয়েও লিখতে পারি, সেখানে ছেলে মেয়ে নিয়ে সমস্যা নেই!

প্রশ্ন: Dear sir, I am Prodipta Saha, A student of class 10. You are my one of the favourite writer. I really like to read your books. Actually, I love to read books so I started to read some book in a very young age, I mean class 4 to class 6. Like, some books of Bibhutibhushan Bandopadhay, Maxim gorkey, Shatyajit Roy, Shunil Gangopadhay, Upendrakishore Roy, Laura Ingalls Wilder, Dan Brown e.t.c. Now that I think of these books. I think I shouldn’t have read them earlier. I should’ve read these books in class 8 or 10. I was a bit young to read these books. Now I am in class 10 and I want someone who will recommend me some books which suits my age and taste … … … … … … … … … Since you’re a experienced reader, I think that you’re the perfect person to recommend me some books that’s suitable for me. Please recommend me some books, I will read them for sure … … … …।This question is a bit urgent because book fair in February is near. I need to buy books. Stay healthy, wealthy and happy. God bless you and may you live long to write books for us. Kakrail, Dhaka.
উত্তর: খুব ছেলেবেলায় মহান লেখকদের বইগুলো পড়ে আসলে তোমার কোনো ক্ষতি হয়নি, বরং তোমার ভেতরে মহৎ সাহিত্যের জন্য একধরনের ভালোবাসার জন্ম হয়েছিল। (আমাদের পরিবারের সবাই তোমার মতন কম বয়স থেকে বড় বড় লেখকদের বই পড়ে আসছি, কোনো ক্ষতি হয়নি। বড়জোর একটু খানি ইচড়ে পাকা হয়েছি, তার বেশি কিছু নয়) এই সাইটে আমার প্রিয় ৫০টা বইয়ের তালিকা দেওয়া আছে, দেখ সেগুলো তোমার পছন্দ হয় কিনা। (তোমার সুন্দর চিঠিটা একটু বড় ছিল, সেটা কেটে একটু ছোট করে নিয়েছি)

প্রশ্ন: স্যার, আমি যদি আপনাকে অনুগ্রহ করে শুধু একটিমাত্র বইয়ের নাম বলতে বলি, শুধু একটি, আপনি কোন বইটির নাম বলবেন? ইমাম সোহরাওয়ার্দী ঢাকা ক্যান্টনমেন্ট, ঢাকা।
উত্তর: এটা নির্ভর করে তুমি কখন প্রশ্নটা করেছ তার উপর। যখন একটা সন্ত্রাসী আমাকে হামলা করতে আসছে তখন যদি জিজ্ঞেস কর তাহলে আমি বলব Geologica বইটার কথা (এই বইটার সাইজ বিশাল, 1.25 বাই 1.5 ফুট, ওজন ৭ কেজি, অস্ত্র হিসেবে ব্যবহার করা যায়!) ঠাট্টা করলাম, আসলে শুধু একটা বইয়ের নাম বলা সম্ভব না, তাহলে অসংখ্য গুরুত্বপুর্ণ বইকে অসম্মান করা হবে। যদি ৫০টা বইয়ের নাম বলতে বল, তাহলে বলতে পারি। এই ওয়েব সাইটেই তালিকাটি আছে।

প্রশ্ন: Sir, 1.Tesla coil what? 2.Amra jetake sthan-kalel chador boli,ta to ashole kono chador noi.Tahole eti ashole ki and gravity eta theke kivabe ashe? 3.Capacitance ki? 4.Rectifier ki kaj kore? 5.Amra jake khonij lobon boli ta to Na,Mg,Ca etc metal hishebe lekha hoi.Tahole eta ‘lobon’ keno? Parag Biswas, class 8,FARIDPUR ZILLA SCHOOL,FARIDPUR. (Answer na pele letter pathie pathie apnar website crash korie debo).
উত্তর: এই ওয়েব সাইটে আমি একটা প্রশ্নের উত্তর এক দুইটা বাক্য লিখে দিয়ে থাকি। তোমার ৫টা প্রশ্নের উত্তর এক দুইটা বাক্য দিয়ে দেওয়া সম্ভব না, অনেক কিছু লিখতে হবে। তোমার জন্য সবচেয়ে সহজ হবে প্রশ্নগুলো ইন্টারনেটে করা, সেখানে অনেক গুছিয়ে উত্তরগুলো পেয়ে যাবে। সেখানে যদি না পাও, আমাকে জানিও,  তখন আমি উত্তর দেব।

প্রশ্ন: স্যার, আমাদের সুন্দর ভবিষ্যতের জন্য কী অটোপাস ভলো হলো? (নুজারাত ইসলাম, চট্টগ্রাম বিশ্ববিদ্যালয়, চট্টগ্রাম)
উত্তর: তুমি কী জান, সমস্ত পৃথিবী অন্যরকম হয়ে যাচ্ছে? তুমি কোন শিক্ষা প্রতিষ্ঠানে, কোন বিষয়ে কত ভালো ফলাফল করে পাশ করেছ সেটার এখন বিশেষ গুরুত্ব নেই। এখন সবাই জানতে চায়, তুমি একটা বিষয় কত ভালো করে জান। কাজেই তোমার ভবিষ্যতের জন্য তুমি অটোপাস করেছ নাকি পরীক্ষা দিয়ে পাশ করেছ তার থেকে অনেক বেশি গুরুত্বপুর্ণ হচ্ছে তুমি কতটুকু জান।

প্রশ্ন: আসসালামু আলাইকুম স্যার, কেমন আছেন? আমি অনেক ভালো আাছি।কারণ আমি এখন সময়ের মুল্য বুঝেছি, যা শিখিয়েছেন আপনি।আপনার প্রতিটি বইয়েই কোন না কোনভাবে সময়কে গুরুত্ব দেওয়া হয়েছে। তাই আপনাকে ধন্যবাদ। নুসরাত ইসলাম, চট্টগ্রাম
উত্তর: থ্যাংকু! তুমি যদি সময়ের মূল্য দেওয়া শিখে থাকো, তাহলে অনেক বড় একটা কাজ হয়েছে। তবে আগে থেকেই তোমাকে সাবধান করে দিই, তুমি কিন্তু দেখবে তোমার আশেপাশে যারা আছে তাদের বেশির ভাগ মানুষই  কোনো মূল্য দিচ্ছে না। তুমি একটা অনুষ্ঠানে সময়মতো গিয়ে দেখবে আর কেউ আসেনি। তখন কিন্তু মন খারাপ করো না।

প্রশ্ন: স্যার টাইম ট্রাভেল কী সম্ভব? এ.কে.এম মুনেম। মিরপুর ঢাকা।
উত্তর: এর উত্তরে সরাসরি ‘হ্যাঁ ‘ বলা খুব বিপজ্জনক। আমি এভাবে বলি: নোবেল পুরস্কার বিজয়ী কিপ থর্নের ব্ল্যাক হোলের উপর লেখা একটা বইয়ে তিনি কয়েকটি সায়েন্টিফিক পেপারের কথা বলেছেন যেখানে বিশেষ পরিস্থিতিতে সময় পরিভ্রমন করা সম্ভব সেটি লেখা হয়েছে। (আমি যখন ক্যালটেকে ছিলাম তখন কিপ থর্ন আমার অফিসের পাশে একটা অফিসে বসতেন)

প্রশ্ন: Sir আবার বিরক্ত করছি। আপনি time travel এর উপর একটা প্রশ্নের উত্তরে লিখেছেন ভবিষ্যতে time travel সম্ভব হতে পারে। কিন্তু মনে করুন যদি আমি time travel করে past বা future এ যাই, তবে সেখানকার space-time এ কিছু mass বেড়ে যাবে এবং present এ কিছু mass কমে যাবে। কিন্তু total mass energy তো constant? যদি একটু বুঝিয়ে দিতেন। কঠিন করে বললেও অসুবিধা নেই।
Sir, আমার একটা request আছে আপনার কাছে। আপনি sayra scientist আর anik lumbar আরো গল্প লিখুন না প্লিজ। বিশেষ করে anik lumbar mermaid এর গল্পটা যেন হঠাৎ করেই শেষ হয়ে গেল। Sandipan Chatterjee, Krishnagar(India)
উত্তর: খুব ভালো প্রশ্ন করেছ। কাছাকাছি ছোট স্কেলে mass energy constant কিন্তু বড় স্কেলে সেটা constant না। পুরো universe প্রসারিত হচ্ছে, space এর একটা energy থাকে (dark energy) কাজেই universe এর energy কিন্তু বেড়ে যাচ্ছে।
আমি আসলে একটি চরিত্র নিয়ে একাধিক বই লিখতে চাই না। বাচ্চাদের আগ্রহে টুনটুনিকে নিয়ে লিখে যা বিপদে পড়েছি তোমাকে বুঝাতে পারব না। আমি জানি আমার অনেক লেখাই তাড়াহুড়া করে লেখা, আরো যত্ন করে লেখা উচিৎ ছিল, কিন্তু কী করব বল? কষ্ট করে মেনে নাও, প্লীজ!

প্রশ্ন: নামঃ রিসালাত মুখর Address :Gangni, Meherpur Sir please please tell me about the family of Rashed (“আমার বন্ধু রাশেদ”)
উত্তর: এটা তো একটা কাল্পনিক চরিত্র, কাজেই তার পরিবারটাও কাল্পনিক। বইটিতে রাশেদ যেহেতু ইবুকে তার পরিবারের গল্পটি বলতে পারেনি, সেটা তো সারা জীবন অজানাই থেকে যাবে।

প্রশ্ন: আমি এবং আমার ছোটভাই দুইজনই আপনার সাইন্স-ফিকসন ও অন্যান্য উপন্যাস এর অনেক বড় ভক্ত।আমার ছোট ভাই বর্তমানে ৮ম শ্রেণীতে পড়ে।সে আপনার “রুহান রুহান” উপন্যাস পড়ার পর এই গল্প নিয়ে একটা কার্টুন তৈরি করার কথা বলে।যেহেতু করনার কারনে স্কুল বন্ধ থাকায় তেমন কাজ ছিলনা।তাই ও কার্টুন অঙ্কন করতে শুরু করল।আমি তাকে সামান্য সাহায্য করেছি।সব মিলিয়ে ৪মাস পরিশ্রম করার পর ৫মিনিট ১৫ সেকেন্ডের মত দৈর্ঘ্যের একটা ছোট কার্টুন বানানো হয়েছে।এই ৫মিনিটে আপনার সম্পূর্ণ উপন্যাসের কাহিনি ফুটিয়ে তুলা সম্ভব না।সেই কারনে আমরা শুধুমাত্র একটা মুভি টেইলার টাইপের কার্টুন বানিয়েছি।আমার ভাই এর ইচ্ছা আপনাকে এই কার্টুন দেখাবে।কিন্তু আপনার সাথে যোগাযোগ করা এতটা সহজ না।আর এটা তৈরিতে আপনার অনুমতিরও প্রয়োজন ছিল।কিন্তু আমরা নেইনি অথবা নিতে পারিনি এর জন্য আমরা আন্তরিক ভাবে দূঃখিত।তবে এটা শুধু আমরা একজন ভক্ত হিসেবে তৈরি করেছি।
তবে আপনার ওয়েপ সাইটের প্রশ্ন করার অপ্সন দেখে ভাল লাগল।তবে এখানে ছবি/ভিডিও পাঠানোর ব্যাবস্থা থাকলে ভাল হইত।তাহলে আপনাকে “রুহান রুহান” নিয়ে তৈরি করা কার্টুনের কিছু অংশ অথবা পোস্টার পাঠিয়ে দিতে পারতাম।যাইহোক নিচে বিশেষ কিছু ছবি এবং পোস্টারের লিংক পাঠালাম।আপনার হাতে হয়ত সময় অনেক কম।কিন্তু তবুও যদি চান দেখে আসতে পারেন।হয়ত আপনার ভাল লাগবে। হাসিবুল হাসান প্রান্ত এবং তৌহিদ হাসান শান্ত বগুড়া।
উত্তর: আমার লেখা কিছু নিয়ে কার্টুন বানানোর জন্য কোনো অনুমতির দরকার নেই। তবে কেউ সেটা নিয়ে বাণিজ্যক কিছু করতে চাইলে আমার অনুমতি নেওয়া ভদ্রতা। আমি অবশ্য বাণিজ্যক কিছু করার অনুমতি দেই না, বই হচ্ছে পড়ার জন্য সেটা আমি সেরকমই রেখে দিতে চাই। তোমাদের পাঠানো লিংকটা দেখলাম, খুব সুন্দর হয়েছে। সরি, এই সাইটে ছবি ভিডিও পাঠানো যায় না, আমি আসলে সেরকম কিছু করতেও চাই না। আমি মানুষের ছবি, ভিডিও, নেটওয়ার্ক এসব দেখতে চাই না, সামনা সামনি দেখতে চাই।

প্রশ্ন: Hi SIR!1.Apner book gular moddhe apner favourite book konta?? 2.Apner hand writing kemon? dekte chai. 3.Prosno korar ekhane e picchi ta K? 4.”Ritin” book er Ritin name tar inspiration ki chilo?? 5.Apner kono fan club ki ase? Name: Tafri Tabassum From: Dhaka. YOUR BIG BIG BIG FAN!!!!!!!! o happy new year Sir!
উত্তর: ১. নিজের বইয়ের মাঝে কোনটা ভালো কোনটা খারাপ বের করা খুব মুশকিল। গত কয়েকদিন থেকে, রাশা আর আমার বন্ধু রাশেদের কথা বলছি (তার কারন এই বই দুইটা কিছুদিন আগে পড়েছি, আবার কোনো একদিন অন্য কোনো বই পড়ে অন্য কোনো বইয়ের নাম বলা শুরু করব!) ২. জঘন্য! এই দেখ। ৩. আমাদের ক্যাম্পাসের একটা দুষ্টু ছেল ৪. না, কোনো inspiration নাই, এটা শুধু একটা নাম। ৫. মাথা খারাপ?! নাই নাই নাই!

প্রশ্ন: আসসালামু আলাইকুম, সার।আমি তাছমিয়া জাহান সাওদা।ঝালকাঠি থেকে বলছি।৯ম শ্রেণির ছাত্রী।আশা করি ভালো আছেন।আমি আজ একটা অভিযোগ নিয়ে এসেছি।আমি আপনাকে একটা মেইল পাঠিয়েছিলাম।আপনি উত্তর দেন নি কেন?আমার মন খুবই খারাপ হয়ে গেছে। তাই আমার অনুরোধ আপনি প্লিজ আমার মেইল্টা এক্ষনি পড়ে নিন,স্যার।
উত্তর: শোন, প্রতিদিন আমার কাছে শত শত মেইল আসে। আমি খুবই ঢিলে ঢালা মানুষ, কোনো কিছু ঠিক করে করতে পারি না, ঠিক করে ম্যানেজও করতে পারি না। তাই অনেক মেইলের সময়মতো উত্তর দেয়া হয় না। যাদের দরকার বেশি তারা তিনবার চারবার মেইল পাঠায়, তারপর পাঠাতেই থাকে তখন আমি হয়তো উত্তর দেই। কাজেই তুমি মন খারাপ না করে মেইল পাঠাতে থাকো। (আমাকে আগের মেইলটা পড়তে বলেছ, আমি জন্মেও সেটা খুঁজে পাব না!)

প্রশ্ন: স্যার, আমার প্রিয় লেখক আপনি? আর আপনার প্রিয় লেখক কে? আমি আপনার লেখা বই স্থানীয় লাইব্রেরী এবং রকমারি থেকে সংগ্রহ করি।এই কারণে আমার কাছে আপনার কোনো অটোগ্রাফ নেই। আমি কিভাবে আপনার একটা অটোগ্রাফ পেতে পারি। তীর্থ রায় বানারীপাড়া, বরিশাল
উত্তর: প্রিয় লেখক বলা মুশকিল, প্রিয় বই বলা যেতে পারে। এই সাইটেই আমার প্রিয় পঞ্চাশটা বইয়ের নাম দেওয়া আছে, দেখে নিতে পার। অটোগ্রাফ পাঠানোর জন্য একটা ঠিকানা দরকার।

প্রশ্ন: sir ami proggramming shikte chai, kintu amar baba ma kobite abriti koranor jonno lege thake. ami ki korte pari? Evan, dhaka ( name ta gopon rakhben please )
উত্তর: কবিতা আবৃত্তি করলে প্রোগ্রামিং করা যাবে না কে বলেছে? কবিতা আবৃত্তি আর প্রোগ্রামিং একসাথেই করতে থাকো। তার সাথে গানও গাইতে পার, ক্রিকেট খেলতে পার, ছবিও আঁকতে পার। সমস্যা কি?

প্রশ্ন: হাসিন, ক্লাস ৯ ইংলিশ version, ঢাকা ৷ স্যার আমি খুবই কষ্ট পেলাম যে আপনি আমার প্রশ্নের উত্তর দেননি। আশা করি এই প্রশ্নের উত্তর দিবেন। আমি জীবনে আপনার প্রথম বই পড়ি ২০১৬ সালে। আপনার বই পড়া থেকেই আমার গল্পের বই (আরো শুদ্ধ করে বলা উচিত আউট বুক (!) ) পড়া শুরু হয়। করোনার এই সময় বই পড়া নিয়ে আমার সবচেয়ে বেশি সমস্যা হয়। আমার আম্মু অনেক বই পড়ত বলে বাসায় অনেক বই আছে। করোনার সময় প্রথমবারের মতো আমি দেখি যে বাসার প্রায় সব বই আমার পড়া শেষ ( শুধু বড়দের বইগুলো পড়িনি, সেগুলো পড়তে ভালো লাগে না) । আমার অনলাইন ক্লাসের জন্য একটা ল্যাপটপ ছিল তাতে অনেক দিন পিডিএফ বই পড়ি। কিন্তু পিডিএফ বই পড়তে ভালো লাগে না। কয়েক সপ্তাহ আগে ল্যাপটপে একটা সমস্যা হয় যার কারনে আমি পিডিএফ বই পড়তে পারছি না এবং আমার ধারণা ল্যাপটপটা আর ঠিক হবে না। আমার আব্বু আম্মু আমাকে বই কিনে দিতে চায় না কারন আমি গল্পের বই পড়লে আমার পরীক্ষা খারাপ হবে বলে তাদের ধারণা। এমন অবস্থায় আমি কিভাবে বই পড়ব? আমি পিডিএফে বই পড়তে চাই না কারন এতে আমার চোখের অনেক ক্ষতি হতে পারে।আপনি প্লিজ তাড়াতাড়ি উত্তর দিয়ে জানাবেন আমি কীভাবে বই পড়ব।
উত্তর: আমি সারা জীবন লাইব্রেরি থেকে বই এনে পড়েছি। কেউ যদি শুধু বই কিনে পড়তে চায় তাহলে বুঝতে হবে সে বেশি বই পড়ে না। তোমাদের ওখানে বিশ্বসাহিত্য কেন্দ্রের ভ্রাম্যমান লাব্রেরি আছে না?

প্রশ্ন: Sar thenk u amar posner utor deor jono. Amar boys ekhon 7. Amar barde 13 disimbor
উত্তর: চমৎকার। জন্মদিনে মজা করেছিলে?

প্রশ্ন: স্যার, আপনি কই? আমার ৬ বছরের ছোট ভাইয়ের প্রশ্নের উত্তর দিলেন, আমার প্রশ্নের উত্তর দিলেন না আপনি এতোদিন অপেক্ষা করছিলাম, কখন আপনি আসবেন আর সব প্রশ্নের উত্তর দিবেন। আমি কিন্তু মহা ক্ষেপে আছি। এই দেখেন (এরপর ৬২টা রাগী- রাগী মুখ) এবার বুঝলেন তো, কতো খ্যাপা? রুম্মান জারা, ৭ম শ্রেণি, মাতৃপীঠ সরকারি বালিকা উচ্চ বিদ্যালয়, চাঁদপুর।
উত্তর: হ্যাঁ আমি বুঝতে পারছি যে তুমি খুব রেগে আছ। এতোগুলি রাগী মুখ আমি একসাথে আগে দেখিনি! এই যে এখন আমি উত্তর দিলাম, এখন কি অনেকগুলো হাসিমুখ পাঠাবে?

প্রশ্ন: সাকিফা সারওয়া। ব্লু-বার্ড স্কুল এণ্ড কলেজ। সিলেট । আসসালামু আলাইকুম স্যার।কেমন আছেন? স্যার আমার মনে সবসময়ই ভয় থাকে।কিসের ভয় জানেন? আমি গণিত বিষয়টাকে ভীষণ ভয় পাই।আমার বাবা,দাদা,চাচা,মামা সবাই আমাকে নানা ভাবে গণিতের মজাদার রহস্য গুলো বোঝায়।কিন্তু আমার এ ভয়টি মন থেকে ত্যাগ করে না।বরং দিন দিন বেরেই চলেছে। কীভাবে আমি এ ভয় থেকে রক্ষা পাবো? আপনার উপদেশের অপেক্ষায়!!
উত্তর: তুমি গণিতকে ভয় পাও? কি সর্বনাশ! এরপরে বলবে “আমি রসগোল্লাকে ভয় পাই!” “আমি সন্দেশকে ভয় পাই!” আমি একটা ছেলের কথা জানতাম যে গণিতকে খুব ভয় পেতো, সেজন্য তার পরীক্ষাও খারাপ হতো। তখন সে আমার আর প্রফেসর কয়কোবাদের লেখা বই ‘নিউরনে অনুরণন’ বইটি পড়েছে, সেখানকার সমস্যাগুলো নিয়ে চিন্তা করতে করতে একসময় তার গণিতের ভয় কেটে গেছে। তারপর সে গণিতে তার ক্লাশের সবচেয়ে ভালো ছাত্র হয়ে সবচেয়ে বেশি নম্বর পেয়ে পাশ করেছে। কাজেই তুমি নিউরনে অনুরণন (এবং এর পরের খন্ড নিউরনে আবারও অনুরণন) এই দুটি বই জোগার করে পড়া শুরু করতে পার। সেখানে যে সমস্যাগুলো আছে সেটা নিয়ে চিন্তা শুরু করতে পার। এই সমস্যাগুলোর মজা হচ্ছে এগুলো করতে যোগ বিয়োগ গুণ ভাগ ছাড়া আর কিছু জানতে হয় না। এ ছাড়া আরো একটা বিষয়ে তোমার নজর দিতে হবে। তোমার যে গণিত বই আছে তার অংকগুলো করার আগে সেই চ্যাপটারের শুরুতে যে কথাগুলো লেখা আছে সেগুলো খুব মন দিয়ে বুঝে বুঝে পড়তে হবে। নিজে নিজে, টিউটরকে দিয়ে নয়। কাজেই দেরি না করে আজকে থেকেই গণিত ভীতি দূর করার প্রজেক্ট শুরু করে দাও।

প্রশ্ন: স্যার আপনি দিনের কোন সময়ে লেখালেখি করতে ভালবাসেন? আপনার প্রিয় খাবার কী? প্লিজ উত্তর দেবেন। মোঃ আরাফাত হোসেন, নোয়াখালী
উত্তর: আমাকে এত বেশি লিখতে হয় যে যখনই সময় পাই তখনই লিখি। তবে মনে হয় সবচেয়ে বেশি লেখা হয় রাত্রিবেলা। প্রিয় খাবার হচ্ছে ভাত, ডিমভাজা এবং ডাল। নাস্তা হিসেবে মুড়ি।

প্রশ্ন: আমার আগ্রহ আর্টস বিভাগে পড়ার। কিন্তু পরিবার সাইন্স নিতে চাপ দিচ্ছে। তারা বলে আর্টসের নাকি কোনো ভবিষ্যৎ নেই, এটা নাকি দুর্বলদের সাবজেক্ট। কি করি এখন? তারা কি ঠিকই বলে? যাদব ঘোষ, কিশোরগঞ্জ।
উত্তর: না না না না না আর্টস মোটেও দুর্বলদের সাবজেক্ট না। তুমি যেটা পড়তে চাও সেটাই পড়বে, এটাই নিয়ম হওয়ার কথা। তোমার পরিবারকে বোঝাও যে তোমার জীবন তুমি তোমার মত করে সাজাতে চাও।

প্রশ্ন: আনেকেই ইংরেজি বর্ণমালা দিয়ে বাংলা লেখার অপচেষ্টা চালাচ্ছে। উক্ত বিষয়ে আপনার মতামত কি? সানিকুল ইসলাম, নবম শ্রেণির ছাত্র, ব্রাহ্মণবাড়িয়া।
উত্তর: এত সুন্দর বাংলা থাকতে কোন পাগল ইংরেজি দিয়ে বাংলা লিখতে চায়? এক সময় কাজটা সোজা ছিল না, এখন তো পানির মত সোজা।

প্রশ্ন: স্যার পেপারে কলাম লিখেন না কেন? অবন্তি,ময়মনসিংহ
উত্তর: অনেক লিখেছি! আর কতো? এখন অন্য কিছু করি, কী বল?

৮ জানুয়ারি ২০২১

প্রশ্ন: আমি বইয়ের পোকা।সৌরজগতের তৃতীয় গ্রহ পৃথিবীর সুজলা–সুফলা শস্য শ্যামলা দেশ বাংলাদেশ থেকে।আমি আজই এই ওয়েবসাইটে প্রকাশিত “৫০ টি প্রিয় বই”–এর তালিকাটি দেখলাম।আপনাকে এর জন্য ধন্যবাদ।থ্যাংকস্ এ টন।কিন্তু খুবই দুঃখের সাথে জানাচ্ছি আপনি আমার অন্যতম প্রিয় লেখক হুমায়ূন হূ আহমেদের নামের বানানে ভুল করেছেন।আপনি হ-এর সাথে উ-কার দিয়েছেন।কিন্তু এটি হবে ঊ-কার। দ্বিতীয় কথা—আপনি আমার প্রশ্নের উত্তর দেন না কেন?
উত্তর: নাম আর পরিচয় সহ প্রশ্ন করে দেখো তো কোনো লাভ হয় কিনা।

প্রশ্ন: সিফাত, নাটোর। এইচ. এস. সি. দ্বিতীয় বর্ষ আপনি কি অ্যাটাক অন টাইটান দেখেছেন? জোকার মুভি দেখেছেন? এ স্টাডি ইন স্কারলেট পড়েছেন? কোনটা কেমন লাগলো নানু? দয়া করে জানাবেন।
উত্তর: যথাক্রমে: জানি না, মনে হয় দেখেছি এবং হ্যাঁ পড়েছি। কেমন লেগেছে মনে নেই, তবে আর্থার কোনান ডায়াল সবসময় ভালো লাগে এইটুকু বলতে পারি।

প্রশ্ন: টুনটুনি ও ছোটাচ্চুর বইগুলো আমার খুবই প্রিয় আসলে আমি আপনার ভক্তই হয়েছিলাম 2013 সালে টুনটুনি ও ছোটাচ্চু পড়ার মাধ্যমে। কিন্তু আমার ধারণা এখন থামার সময় এসেছে,আপনার মধ্যে সম্ভবত একটা দায়সারা ভাব চলে এসেছে। বইয়ের পৃষ্ঠার দিকে তাকালে দেখা যায় প্রতিবছর বের হওয়া বইয়ের পৃষ্ঠা সংখ্যা আগের পৃষ্ঠা সংখ্যার চেয়ে কম। গতবার বের হওয়া বইয়ের গল্পগুলো একদম ছোট ছোট এবং আগের মতো টানটান উত্তেজনা নেই গল্পগুলোতে,শেষ বই এর গল্পগুলোতে টুনটুনির বদলে ঝুনঝুনি নাম দিলেও হতো। গল্পগুলো পড়লে কেনো যেনো মনে হয় আপনি শুধু আমাদের ইচ্ছে পূরণ করতেই লিখেন(আপনি অনেক দিন আগে আমাকে এরকম একটা কথা বলেছিলেন, ” একজন লেখক তার ইচ্ছে অনুযায়ী লেখে,পাঠকের যেমন বই পড়ার ইচ্ছে সেইভাবে লেখে না”)। আমরা অবশ্যি টুনটুনির বই চাই কিন্তু আগের সেই টানটান উত্তেজনার ভাবটা না থাকলে বইটা শুধু নাম দিয়েই টুনটুনির বই হবে,কাজ দিয়ে হবে না। গত বছরের বইটার মতো যদি এবছরের বই-ও হয় তাহলে মনে হয় টুনটুনির এবার থামার পালা।(স্যার আমার কথায় কি আপনি মন খারাপ করলেন?অথবা আপনি কি ভাবছেন-“গায়ে পড়ে এমন উপদেশ দিতে এসেছে ছেলেটা?”বিশ্বাস করুন,আপনার মতো গুটি কয়েকজনকে শুধু এইভাবে প্রাণ খুলে মনের কথাটা বলা যায়)। আপনার জন্য অনেক অনেক ভালোবাসা।প্রত্যয় দাস,একাদশ শ্রেণী,এম সি কলেজ,সিলেট
উত্তর: তুমি ঠিকই বলেছ, কিন্তু বাচ্চা কাচ্চারা এমন চেচামেচি করে যে না লিখে পারি না। যখন তোমার মত অন্যরাও বলা শুরু করবে, “অনেক হয়েছে আর দরকার নেই”, তখন বন্ধ করে দেব।

প্রশ্ন: আসসালামু আলাইকুম।আমি অনেক ছোটবেলা থেকে বিশ্ব সাহিত্য কেন্দ্রের ভ্রামমাণ লাইব্রেরির সদস্য।২০২০ সালের ১৮ই ডিসেম্বর পুরো ৬ বছর পূর্ণ হয়েছে।আমি যখন ক লয়ে আকার কলা বানান করে পড়া শিখছি তখন থেকে লাইব্রেরির সদস্য।এই লাইব্রেরির বই দিয়েই মূলত আমি পড়তে শিখেছি।আমি আপনার লেখা সকল কিশোর উপন্যাস পড়ে ফেলেছি এর মাধ্যমে।আরও পড়েছি রকিব হাসান,আনিসুল হক,আহসান হাবীব,হুমায়ুন আহমেদ,সেলিনা হোসেন,জওয়ান ক্যাথেলিন রাওলিংসহ আরও অনেকের লেখা নানা interesting বই।কিন্তু দু:খের বিষয় হলো বর্তমানে ভ্রামমাণ লাইব্্ররির সদস্য সংখ্যা কমে যাচ্ছে ।সবাই মোবাইলে pdf বইয়ের দিকে বেশি আগ্রহ দেখাচ্ছে।এতে করে অনেকের চোখে এখনই চশমা পরতে হচ্ছে।এই অবস্থা কীভাবে দূর করা যায়? মল্লিকা,৬ষ্ঠ শ্রেণি,ভিকারুন নিসা নূন স্কুল।
উত্তর: যদি pdf দিয়েও বই পড়ে আমার আপত্তি নেই। কিন্তু দুঃখটা কি জান, তারা তো আর বই পড়ে না, তারা অনলাইনে সময় নষ্ট করে। এই সমস্যা মিটানোর কোনো সহজ বুদ্ধি আমার মাথায় আসেনি। এইযে আমি এই ওয়েবসাইটটা তৈরি করেছি, বাচ্চারা এটা দেখে সময় নষ্ট করছে, সেটা চিন্তা করেও আমার মাাঝে মাঝে মনে হয় কাজটা মনে হয় ঠিক হচ্ছে না। তুমি কিন্তু তোমার বই পড়ার অভ্যাসটা রেখো সারা জীবন। কারন, যারা বই পড়ে তারাই আসলে দুনিয়াটা সামনে নিয়ে যায়। অন্যরা দুনিয়াটাকে পিছনে টেনে রাখে।

প্রশ্ন: ami jei prosna kori tar uttor kibabe pabo??? jodiuttore jai tobe ato golar mage kibabe pabo/(fahim,,,,,cumilla,,,,,,,,b-para,,,,,,,madhabpur)
উত্তর: ঠিকই বলেছ, তুমি উত্তরটা কোথায় পাবে? কেমন করে পাবে? এই সমস্যা সমাধানের তোমার কোনো আইডিয়া আছে?

প্রশ্ন: উকিল এবং ব্যারিস্টারের মধ্যে পার্থক্য কি? আর লিটু বৃত্তান্ত নিয়ে short flimহলে ভাল হত। অনেকে তো ভুলেই গেছে। ফিয়াদ নওশাদ ইয়ামিন খিলগাঁও ঢাকা
উত্তর: আমি আসলে উকিল এবং ব্যারিস্টারের মধ্যে পার্থক্য কি জানি না। একটা গুগল সার্চ দিয়ে দেখ। আর, আমি আসলে চাই না আমার বই দিয়ে কেউ সিনেমা বানাক। আমি চাই, এগুলো বই হিসেবে থাকুক, বাচ্চা কাচ্চারা নিজের মত করে পড়ুক, নিজেরা নিজের মত করে সব কিছু কল্পনা করে নিক।

প্রশ্ন: Sir amra kono glass kane dile bhan bhan kore sabda hoy kano? Name:Bivor. Class:2 Rainbow School. Jhiltuli Faridpur.
উত্তর: কারনটা খুব মজার, ছোট বড় নানা কিছু কানে লাগিয়ে, দেখ নানা ধরনের শব্দ পাবে। উত্তরটা তুমি ইন্টারনেটে বের করে নাও। যদি না পাও তখন তাহলে আমি তোমাকে বলে দেব।

প্রশ্ন: Sir assalamualaikum. Ami Fiona. Apnar lekha ‘ Serina ‘ boita amar onek bhalo legeche. Jotobar pori totobar kanna peye jay. Serina r amar boyosh shoman. Tai aro bhalobhabe bujhte pari. Fiona Sadad. Thikana: Laxmipur, Rajshahi.
উত্তর: এটা আমার নিজেরও খুব প্রিয় একটা বই, আমি যখন পড়ি তখন আমারও কান্না পেয়ে যায়। কিন্তু আমি তো আর নিজের বই পড়ে কাঁদতে পারি না, লোকজন জানলে কী বলবে?

প্রশ্ন: চমৎকার একটা ওয়েবসাইট! যথেষ্ট গোছানো এবং মানসম্মত হাওয়ার পাশাপাশি ক্ষুদে ব্যবহারকারীদের জন্যে ইউজার ফ্রেন্ডলি ও বটে। কেন যে এতদিন পর এটার খোজ পেলাম আল্লায় জানে! প্রশ্ন, ঠিক না, তবে একটা অনুরোধ বলতে পারেন। এখানে একই প্রশ্ন অনেকে বারবার করছে দেখতে পাচ্ছি। যদি একটা সার্চ বাটন তৈরি করে দেন তাহলে মনে হয় ওই স্টাইলের প্রশ্নের চাপ কমবে। যেমন কে ছবি আঁকে এইসব আরকি। তাছাড়া আরো একটা পেজ যদি থাকে যেখানে সবাই আলোচনা করতে পারবে, তখন বেশ কিছু সাধারণ প্রশ্নও আপনার কাছে যাওয়ার আগেই সলভ হয়ে যাবে।( যেমন ধরুন পেস্টের দৈর্ঘ্য ও অন্যান্য) আপনার মত একজন স্কিলড প্রযুক্তিবিদ এর জন্যে তো এটা পাঁচ মিনিটের কাজ। আপনার কথা ভেবেই বললাম। আশা করি অন্যভাবে নিবেন না। মাহমুদ, ঢাকা।
উত্তর: হা হা হা! তুমি ভাবছ আমি এটাকে দলাই মলাই করে সত্যিকারের একটা জনপ্রিয় ওয়েবসাইট করব? মোটেও না, যদি দেখি বেশি বাচ্চাকাচ্চা এসে গেছে, এটাতে সময় নষ্ট করছে, আমি তাহলে লম্বা সময়ের জন্য একটা ডুব দেব!

প্রশ্ন: Dear sir, I wrote you a quesion but you didn”t give me the answer..now i asked you that “CAN YOU DRAW PICTURE,IF YES PLEASE SHOW ME YOUR PICTURE” HAVE A NICE DAY . ARE YOU WELL????? TTTTTTTHHHHHHHHHHHHHHAAAAAAAAAANNNNNNSSSSSSSSSS FOR READ IT………
উত্তর: কিন্তু আমি যে নিয়ম করে রেখেছি, নিজের পরিচয় না দেওয়া পর্যন্ত উত্তর নাই!

প্রশ্ন: আসসালামু আলাইকুম, স্যার আপনি এখন কোথায় আছেন?(নুসরাত ইসলাম,চট্টগ্রাম ৫-১-২০২১)
উত্তর: ঢাকাতে। মশার উৎপাতে জীবন বের হয়ে যাচ্ছে। কোথায় গিয়ে লুকিয়ে থাকা যায় চিন্তা করছি।

প্রশ্ন: আসসালামু আলাইকুম স্যার। স্যার কেমন আছেন? স্যার আমি অনেকদিন যাবৎ আপনার নাম্বার খুঁজছি।অবশেষে একটা নাম্বার পেয়েছি। (০১৭৪*******) স্যার এটা কি আপনার নাম্বার?? যদি হয়ে থাকে শুধু হ্যাঁ বললেই হবে।আমি প্রমিজ করছি কখনো বিরক্ত করব না।শুধু আমার ফোনে প্রিয় মানুষ ও প্রিয় স্যারের নাম্বার টা সেইভ রাখব। এইতো। উত্তর দেওয়ার সময় নাম্বার টি কেটে দিয়েন স্যার।। ভালো থাকবেন,সাবধানে থাকবেন।বাসা থেকে একদম বের হবেন না। অবন্তি,ময়মনসিংহ।
উত্তর: না, এটা আমার নম্বর না। (একবার একটা বাচ্চা ছেলে আমাকে ফোন করেছে। সে আমাকে জিজ্ঞেস করল আমিই কি ‘আমি’ কিনা। তারপর বলল, সে তিনশ টাকা দিয়ে আমার নম্বরটি কিনেছে, এখন চেক করে দেখছে নম্বরটা খাঁটি নাকি ভূয়া। তুমি কত টাকা পর্যন্ত দিতে রাজী আছ?)

প্রশ্ন: আসসালামু আলাইকুম, স্যার আপনাকে অসংখ্য ধন্যবাদ “টুকি ও ঝা এর (প্রায়) দুঃসাহসিক অভিযান ” বই এর ‘তিন নম্বর গ্রহের কাহিনী ‘ লেখার জন্য।সম্পূর্ণ বইটি ছিল ভীষণ মজার।এরকম আরও বই লিখবেন স্যার।(নুসরাত ইসলাম, চট্টগ্রাম)
উত্তর: থ্যাংকু। (তিন নম্বর গ্রহে কি হয়েছিল ভুলে গেছি! বইটা খুঁজে পেলে পড়ে দেখতে হবে!) ঠিক আছে, এওকম বই আরো লেখার চেষ্টা করে দেখব।

প্রশ্ন: উত্তর দ্যান না কেন?
উত্তর: ভালো প্রশ্ন। কেন এবারেও উত্তর দিলাম না?

প্রশ্ন: Sir konara koantam tunnel korte pare kintu amra ki pari na? Bivore class:3 faridpur
উত্তর: আমরাও পারি, কিন্তু সেটা ঘটার সম্ভাবনা এতো কম যে চেষ্টা করে লাভ নাই। না হলে দৌড়ে এসে একটা দেওয়ালে ধাক্কা দিয়ে দেখতাম!

প্রশ্ন: Sir,metallic bond er positive charge gulo to eke oporke repel kore.Tahole ei bond thake ki kore? Parag Biswas,class 7,faridpur zilla school
উত্তর: কিন্তু সেখানে ভ্যালেন্স ইলেকট্রনও থাকে, সেগুলো নানা পরমানুর ভেতর ঘুরোঘুরি করে, তাদের আকর্ষণের কারনে বন্ডগুলো কাজ করে।

প্রশ্ন: আসসালামু আলাইকুম স্যার। আমার নাম শিল্পী।দয়া করে আমার সাইন্টিস্ট মামার সেকেন্ড পার্ট বের করবেন। প্লিজ প্লিজ প্লিজ। নাম: শিল্পী আক্তার ঠিকানা: ঢাকা, উত্তরা
উত্তর: টুনটুনি দ্বিতীয়বার লিখতে গিয়েই কত বিপদে পড়েছি, এখন আরেকটা বিপদ ডেকে আনব?

প্রশ্ন: সিজ্জাদ মোহাম্মদ, কেরাণীগঞ্জ। আমার আরো একটা প্রশ্ন, আপনার অনেকগুলো ছবি দেখলাম পেইন্টিং এর সামনে, কিন্তু আমার কাছে ওই মহামূল্যবান শিল্প গুলো বাচ্চাদের হাবিজাবি দাগানোর মতোই মনে হল। বাড়ি ঘর সব ত্যাড়া ব্যাকা, মাথা মুন্ডু কিচ্ছু বুঝলাম না। আমি কি সত্যিই বোকা? নাকি ওগুলো বুঝতে বিশেষ কোনো প্রশিক্ষণ নিতে হয়। আপনি এক্কেবারে সত্যি করে বলেন কিচ্ছু বুঝছেন কিনা।
উত্তর: শিল্প বোঝার ব্যাপার নয়, উপভোগ করার ব্যাপার। যদি উপভোগ করতে পার তুমি তাহলে নিজেই ছবিতে অনেক কিছু খুঁজে পাবে। কাজেই তুমি যদি কোনো ছবি উপভোগ না কর, অন্য কিছু দেখ। অনেকে ক্লাসিক্যাল মিউজিক উপভোগ করতে পারে না। তার মানে এই নয় যে ক্লাসিক্যাল মিউজিক অর্থহীন কিছু।

প্রশ্ন: নাম ফারহানা। আপনার ছবিটা চমৎকার হোয়েছে। একটা প্রশ্ন, যদি আমার কোনো লেখা আপনাকে পাঠাই, আপনি কি পড়বেন? ছোট গল্প।
উত্তর: কোন ছবিটার কথা বলছ? লেখা পাঠিয়ে দেখতে পার, কিন্তু সব সময় যে পড়তে পারি তা নয়। অনেকে জানতে চায় যে কেমন হয়েছে বলতে, তখন আমি তো আর ভালো-মন্দ বলতে পারি না তাই চোখ বুঁজে ঢালাও ভাবে বলি ‘ভালো হয়েছে’! তা ছাড়া আমি তো সমালোচক না, আমি হচ্ছি পাঠক, ভালো লাগলে পড়ি, ভালো না লাগলে পড়ি না।

প্রশ্ন: Sir Assalamualaikum. Ami Drishti. Amar proshno holo, kishore uponnasher kon boi ti apnar favourite? Drishti Islam. Thikana: Bohorompur, Rajshahi.
উত্তর: ডিসেম্বরের ২৮ তারিখ আমি যে প্রশ্নের উত্তরগুলো দিয়েছিলাম, সেখানে একটু চোখ বুলাও প্লিজ, তোমার উত্তরটা পেয়ে যাবে।

প্রশ্ন: Dear sir, In response to your post, published on your Facebook page “EI LEKHATI OVIVABOKDER JONYO”, I would like to propose a solution for the weak students of this country. Could you please SMS me your personal phone number and email ID so that I could explain my proposal in details? My name is Zahid Anwar, and I reside at kalyanpur in Dhaka. Here is my personal phone number: 0178*******.
উত্তর: আসলে মাঝে মাঝে আমার কোনো একটা ব্যাপার একটু বিচলিত করে তখন আমি কিছু একটা লিখি। বড় কোনো সমস্যার একটা সমাধান আমি বের করে ফেলতে পারব সেটা কখনোই চিন্তা করিনি। তার চেষ্টাও করি না। তাই তুমি যদি একটা জটিল সমস্যার ভালো কোনো সমাধান পেয়ে যাও, সেটা তোমার নিজেকেই বাস্তবায়ন করার চেষ্টা করতে হবে। এটা বাচ্চা কাচ্চাদের জন্য একটা ছেলেমানুষী ওয়েবসাইট, এখানে আমি আসলে গুরুতর কোনো বিষয় নিয়ে আলোচনাও করি না!

প্রশ্ন: সার, আমি ফারিয়া। আমি ঢাকায় থাকি। আমি আপনার অনেক বড় ভক্ত। আপনি please টুনটুনি ও ছোটাচ্চুর ব বইগুলো টাইপ করে দিয়েন। প্লিজ প্লিজ প্লিজ Please please please.
উত্তর: আমি শুনেছি আমার প্রায় সব বইয়ের পিডিএফ নেটে পাওয়া যায়। একটু খুঁজে নেবে প্লিজ? আমি তো সব কাগজে কলম দিয়ে লিখি!

প্রশ্ন: EXCUSE ME I AM FROM THIS ERA AND WANTED TO KNOW ABOUT YOUR ERA I HOPE YOU WILL RESPONSE EARLY AT THIS PANDAMIC MANY TIMES OF OUR LIFE HUGELY DEPEND ON THIS DAM ELECTRONIC DEVICE SO I WANT TO KNOW ABOUT YOUR TIME ELECTRONIC DEVICE ABOUT HOW TO ACCES ABOUT ITS WORKING AND ETC ETC I KNOW YOU ARE READING THIS IF IF YOUUU NOT RESPONS WITHIN AN HALF OF A YEAR I WOULD BE KIND ENOUGH TO GRANT YOU A HUGE SPIDER FROM BANGLADESH AND OBLIGE YOUR WEB BY HACK IT THERE BY YOUR WEB PAGE WILL TURN INTO A BEAUTIFUL MEETING ROOM OF SPIDER LOVERS AND IF YOU REPLY IT I WOULD LIKE TO INFORM MY IDENTITY BYA SECRET NUMBER AND THIS WILL TELL U ABOUT YOUR POSITION ON YOURWORK BTW I THINK THAT U DONT WRIYE BOOKS FOR ONLY MONEY YOU NOT RUN AFTER MONEY SO I THINK I DONT NEED TO TAKE ANYSTEP AGAINST YOUR WEB IF I SSEE THAT YOU DONT DO ANY REPLY I WILL TURN YOUR WEB TO DARK AND IF YOU WILL I WOULD REPLY YOU MY IDENTITY WITH THIS NUMBER :000000001 YOU CAN WRITE AFTER IT AND THE REPLY NUMBER 00000001 OOOOO WILL SHOW GOOD JOB AND 00000001 OOO WILL SHOW NEED PROGRESS AND 00000001 O FAIL AND 00000001 OOOO WILL SHOW GOOD THANK U FOR READING IT
উত্তর: কিন্তু  আমার যে একটা ছোট নিয়ম আছে, পরিচয় নেই তো উত্তর নেই!

প্রশ্ন: মুহাম্মদ রুমি, ঢাকা। স্যার প্লিজ আপনার মেইল আইডি টা দিবেন আমাকে??? আমার কিছু কথা বলার ছিলো স্যার।
উত্তর: প্লিজ খুঁজে বের করে নাও। দুনিয়ার মানুষ জানে তুমি জান না কেন?

প্রশ্ন: মাহিউল নাবিল, তেজগাঁও 1.স্যার HSC তে অনেক গুলো টেক্স বই একটা বিষয় এর জন্য। কিন্তু যদি board থেকে যেকোন একটি বই ঠিক করে দিলে ভালো হতো। 2.Sir besirbhak khetre student ra jekono University teke BSc kore Phd korte foreign e jai.tobe HSC ses kore BSc foreign e korar kono way ase jara bangla version e pore.
Apnar dhirghau kamona kori
উত্তর: আসলে উল্টোটা সত্যি। এস.এস.সি.তেও যদি অনেক গুলো বই থাকতো সেখানে ছেলেমেয়েরা অনেক কিছু পড়ে শিখতে পারত। যেহেতু ছেলেমেয়েরা কোচিংয়ে গিয়ে প্যাকেজ এক ধরনের পরীক্ষা দেওয়ার জন্য কিছু জিনিষ শিখে তাই এগুলো এই দেশে এখনো হচ্ছে না। এক সময় হবে নিশ্চয়ই। তুমি ইন্টারনেটে বিশ্ববিদ্যালয়ের ওয়েবসাইটে গিয়ে তাদের আন্ডারগ্রাজুয়েট ভর্তির নিয়ম কানুন দেখ, সব কিছু জেনে যাবে।

প্রশ্ন: মোঃতারিকুল ইসলাম প্রাগপুর মাধ্যমিক বিদ্যালয়।দশম শ্রেণী।। প্রশ্ন১ঃ 0÷5=0 হলে 5÷0=অনির্ণয় কেন??? প্রশ্ন২ঃ১ এর ঘাত যায়ই হয় না কেন সেটা ১হয়। যেমনঃ১^১০০০=১।আবার ১^১০০০০০=১। তাহলে,১এর ঘাত অসীম হলে অনির্ণেয় হয় কেন???((১ হওয়া উচিৎ ছিল)) পুর্ণশ্চঃআমি অনেক জায়গায় প্রশ্ন করেছি কিন্তু সন্তশজনক উত্তর পায়নি।।
উত্তর: 1. lim x->0 (from left) 1/x-> -infinity but lim x->0 (from right) 1/x-> +infinity. That means you do not know the value of 1/x at x=0 (because you get two different values if approch zero from negative side or positive side). 2. If 1^infinity =1 then log(1^infinity)=log 1 or infinity.0=0 which does not make sense.

প্রশ্ন: মিতু তিতুর টাইমমেশিন গল্পটি আমার খুবই ভালো লেগেছে। এরকম আরো বইয়ের আশায় রইলাম। মাহি ,,,,, ভাটারা ,,ঢাকা।
উত্তর:  থ্যাংকু।  মিতু তিতুকে নিয়ে আরেকটা বই লিখেছি সেটা দেখেছ?

প্রশ্ন: স্যার, আস্সালামু আলাইকুম। আমি সৈয়দ মাহমুদুর রহমান মাহি সাঁথিয়া,পাবনা,রাজশাহী। শ্রেণীঃ ৯ম (২০২০) বিভাগঃ বিজ্ঞান । আমার প্রশ্ন হলো, বিজ্ঞান অলিম্পিয়াডে ভালো করার জন্য কোন বই পড়া উচিৎ ?
উত্তর: আমার তো মনে হয় না সেরকম কোনো বই আছে। তোমার যদি বিজ্ঞান নিয়ে আগ্রহ থাকে বিজ্ঞানের নানা ধরনের বই পড় তাহলে বিজ্ঞান অলিম্পিয়াডে ভালো করতে পার।

প্রশ্ন: sir, chithi na likhe konobhabe ki apnar sathe jogajog kora jay??? mrinnoye mridha bagerhat
উত্তর: টেলিপ্যাথি চেষ্টা করতে চাও? (এর থেকে সোজা হচ্ছে ই-মেইল।)

প্রশ্ন: sir, jekhane tuntuni sekhANE CHOTACCHU boitir PDF copy ki deoa jay??? ashjiba tajnia, Khulna
উত্তর: আমি যতদূর জানি আমার বইগুলোর PDF নেটে পাওয়া যায়, একটু চেষ্টা করে দেখ প্লিজ।

৩১ ডিসেম্বর ২০২০

প্রশ্ন: শ্রদ্ধেয় স্যার, আপনি আমার খুব প্রিয় একজন মানুষ। আমি আপনার অনে………ক বই পড়েছি।কাল রাতে গ্লিনা বইটা শেষ করলাম। আপনাকে অনেক ধন্যবাদ আমাদের জন্য এতো সুন্দর বই লেখার জন্য।আমি এখন দ্বাদশ শ্রেণীতে পড়ছি। আমি স্কুলে ভর্তি হতে গিয়ে গানের স্কুলে ভর্তি হয়ে গিয়েছিলাম। ছোটবেলায় আমি চূড়ান্ত দুষ্টু ছিলাম। স্কুলের সবই আমার ভালো লাগত শুধুমাত্র গণিত ক্লাস ছাড়া।গণিত স্যারকে ভীষণ ভয় পেতাম।স্যারের হাতে সবসময় একটা বেত থাকত, প্রায় সময় তাকে কারো না কারো উপর বেত চালাতে দেখা যেত।গণিত বই হাতে নিলেই তার মুখটা আমার চোখের সামনে ভেসে উঠতো। আমার স্কুল জীবনের দশটি বছর তার ভয়েই কেটে গেল। আমার মা চেয়েছিল আমি সাইন্স নিয়ে পড়ব,আমি একেবারে মানবিকে নেমে গেলাম। আমার মা ছোটবেলা থেকেই আমাকে বিভিন্ন জায়গায় গানের প্রতিযোগিতায় নিয়ে যেতেন,স্কুল থেকেও গিয়েছি কয়েকবার।বেশ কয়েকবার জিতেও যাই।বিচারকরা প্রশংসা করেছেন।তখন ছোট ছিলাম ভালো লাগত। ……….. ……….. ………… ………. ………. ………. ……… ……….. ………. ……… ……….. ………. ……….. ……….. ……… ………আসলে আমি এই কথাগুলো কারো সাথে শেয়ার করতে চাইছিলাম। অতি পরিচিত মানুষের সাথে সবসময় সবকিছু শেয়ার করা যায় না! আপনার বই পড়ে আমি আপনাকে চিনেছি কিন্তু আপনি আমাকে চেনেন না তাই আপনার কাছেই আমার কথাগুলো লিখে ফেললাম।আমি খুব সরি এতো প্যানপ্যান করার জন্য ।খুব খুব সরি।…… …… …… (নাম ঠিকানা গোপন রাখবেন প্লিজ)
উত্তর: আমি এখনই কোনো উত্তর দিচ্ছি না, কারন আরেকটু ব্যক্তিগতভাবে তোমার ই-মেইলে উত্তর দিতে চাই। তাই তোমার ই-মেইল এড্রেসটা আমাকে জানাও। না, তুমি মোটেও প্যানপ্যান কর নি, তুমি আমার সাথে কথা বলেছ। আমরা সবাই একজন আরেকজনের সাথে কথা বলি।

প্রশ্ন: পাওয়ার লাইনে পজিটিভ নেগেটিভ উল্টে দিলে কেন সেখানে সর্ট সার্কিট হয়ে সব এলোমেলো হয়ে যায়? তাসনুভা,৫ম শ্রেণি,ভিকারুননিসা নূন স্কুল
উত্তর: সাধারনত পাওয়ার লাইনে একটাতে হাই ভোল্টেজ থাকে, অন্যটা নিউট্রাল। কাজেই তুমি যদি পজিটীভ নেগেটিভ উলটে দাও তাহলে হাই ভোল্টেজটা নিউট্রালকে স্পর্শ করে, সেটাই শর্ট সার্কিট।

প্রশ্ন: স্যার, এর আগে দুটো প্রশ্ন পাঠিয়েছি, আপনি উত্তর দেন নি। কেন???? আমার একটা প্রশ্ন। আমি শুনেছি ১৮ বছর বয়সের নিচে কারও করোনা ভ্যাকসিন না দিলেও হবে। এটা কি সত্যি? আবরার ফাহাদ
উত্তর: যখন শুরুতে অনেক দেশের কাছে যথেষ্ট ভ্যাক্সিন নেই তখন, তখন তারা কমবয়সীদের ভ্যাক্সিন দিচ্ছে না, কারন তারা আক্রান্ত হয় কম, আক্রান্ত হলেও বেশি অসুস্থ হয় না। বাসার বয়স্ক মানুষেরা যদি ভ্যাক্সিন নিয়ে নিরাপদ থাকে তাহলে কমবয়সীরা সংক্রমন ছড়ালেও তেমন কোনো ঝুকি নেই, এটা হচ্ছে পিছনের লজিক।

প্রশ্ন: স্যার, আপনি এতদিন কোথায় ছিলেন ? জানেন আমি প্রতিদিন দুইবার এই ওয়েবসাইটে আসি, আপনি উত্তর দিয়েছেন কিনা দেখার জন্য ! আপনি এসেছেন দেখে মরুভূমিতে পানি অথবা সাগরে মরুভূমি দেখলাম ! ধন্যবাদ ! নাবিল আহসান, পাবনা ৷
উত্তর: টানা আট মাস যদি প্রতিদিন দুইবার করে এসে থাক, তাহলে তুমি একটা ওয়েবসাইটের পিছনে কত সময় নষ্ট করেছ, চিন্তা করে দেখেছ? এটাই যদি তোমার জীবনের মরুভুমির পানি হয়ে থাকে তাহলে তোমার জীবন নিয়ে দুশ্চিন্তা হওয়ার কথা!

প্রশ্ন: প্রিয় লেখক মুহম্মদ জাফর ইকবাল, আপনার ই-মেইল এড্রেসটি কি আমাকে দেবেন? আপনার সাথে আমার অনেক কথা আছে। রওনক ইউসুফ শুভ্র শ্রেণি: ষষ্ঠ
উত্তর: কথাগুলো এখানে বলা যায় না? নেটা সার্চ দিলে ই-মেইলটা পেয়ে যাবার কথা।

প্রশ্ন: আসসালামুয়ালাইকুম। স্যার, “টুনটুনি ও ছোটাচ্চু” সিরিজে টুনি সব কেস সমাধান করে। কিন্তু টুনি কে যদি কেউ কিডন্যাপ করে তাহলে কেমন হয়? আশা করি আমার আইডিয়া টা ভালো লাগবে। বিঃদ্রঃ আমি আগেও এটা লিখেছিলাম কিন্তু উত্তর পাইনি।
পুনরায় বিঃদ্রঃ স্যার, আপনি বলেছিলেন এখানে কেউ কারো প্রশ্ন-উত্তর দেখতে পারেনা। কিন্তু আমি সবার প্রশ্ন-উত্তর “উত্তর” সেকশনে দেখতে পাই। শুধু আমার প্রশ্নের উত্তর ই পাচ্ছি না।
উত্তর: হ্যাঁ কিডন্যাপের আইডিয়াটা ভালো। নিরাপদে উদ্ধার করতে না পারলে বিপদ হয়ে যাবে। আমি দুঃখিত সব প্রশ্নের উত্তর সব সময় গুছিয়ে দিতে পারি না। আমি মানুষটাই অগোছাল এবং ঢিলে ঢালা, প্লীজ মেনে নাও! আমি যখন প্রশ্নের উত্তর দিই তখন সেই প্রশ্নট অন্যরা দেখতে পারে, না হয় দেখতে পায় না।

প্রশ্ন: আমি তো ভেবেছিলাম আপনি ভুলেই গেছেন আপনার একটি ওয়েবসাইট আছে।জানেন স্যার আমি প্রতিদিন একবার করে আসি। কিন্তু আপনি আসেন না। এই অতগুলো দিন গেল কোথায় ছিলেন? সপ্তাহে একবার করে এলেই তো হয় বা পনেরদিনে একবার বা মাসে একবার এর বেশি না । অনেক মিস করি আপনাকে। কেমন আছেন স্যার আপনি? অবন্তি,ময়মনসিংহ।
উত্তর: আসলে কম্পিউটারের স্ক্রিনের সামনে সময় কাটাতে কাটাতে আর জুম মিটিং করতে করতে ত্যাক্ত বিরক্ত হয়ে গিয়েছিলাম। সেজন্য কিছুই করা হয়নি। আমি ভালো আছি, ঘরের ভেতরে বসে থাকাটাকে তুমি যদি ভালো বলতে রাজী হও!

প্রশ্ন: স্যার, আসসালামু আলাইকুম। আমরা আপনার (মূলত আপনার বই এর ) অনেক বড় FAN!! কিন্তু দুর্ভাগ্যক্রমে, আমরা রাজশাহীতে থাকায় কখন আপনার সাথে সরাসরি সাক্ষাৎ করতে পারিনাই। আপনার কি একটা অটোগ্রাফ পেতে পারি? মুনতাসির আহসান রুদ্র ,আফিয়া আজমাঈন। তেরোখাদিয়া, রাজশাহী।
উত্তর: তোমরা যদি তোমাদের বাসার ঠিকানাটা লিখে দাও তাহলে একটা কাগজে লিখে খামে ভরে তোমাদের অটোগ্রাফ পাঠাতেই পারি।

প্রশ্ন: আসসালামু আলাইকুম স্যার। আমার নাম ফিওনা। আমরা টুনটুনি ও ছোটাচ্চুর শান্ত ও শান্তার দেখা কবে হবে সেটা জানার জন্য খুব উত্তেজিত। নাম: ফিওনা সাদাত।
উত্তর: টুনটুনি ও ছোটাচ্চুর শান্তকে চিনতে পেরেছি (মহা দুষ্টু), কিন্তু শান্তাটি কে? কোথায় তার খোঁজ পেলে?

প্রশ্ন: Sir,tuntuni o chotachu te chotachu ar farihapur bie kobe diben? amra oshesh agrohe boshe achi!!!– Afia (terokhadia, Rajshahi )
উত্তর: আমি তো বিয়ে দিতেই চাই, ওরা রাজী হচ্ছে না, আমি কী করতে পারি?

প্রশ্ন: “সায়রা সায়েনটিসট” লেখার পেছনে বিশেষ কোনো কারণ আছে? Raina Raihan, Dhaka, Bangladesh
উত্তর: মাঝে মাঝেই আমার একটু হাসির সায়েন্স ফিকশান লেখার ইচ্ছা করে, এছাড়া আর কোনো কারন নেই।

প্রশ্ন: স্যার, প্লান চ্যাট, হাত দেখা, জ্যোতিষ চর্চা এগুলো নিয়ে পড়ার আমার ভীষণ ইচ্ছা কিন্তু আমি এই সম্পর্কিত কোন বইয়ের নাম জানিনা। স্যার এই কয়েকটা বইয়ের নাম বলবেন? নিশি, ময়মনসিংহ।
উত্তর: এইগুলোর তো কোনো বৈজ্ঞানিক ভিত্তি নেই, নেহায়েত মজা করার জন্য কমবয়সে এগুলো ঘাটাঘাটি করেছিলাম। এখন তো এগুলোর সাথে আমার কোনো সম্পর্ক নেই, কোনো বইয়ের নামও জানি না। ইন্টারনেটে খোঁজ করে দেখ না কেন?

প্রশ্ন: নামঃ মিসির আলি ঠিকানাঃ মালিবাগ আচ্ছা মিসির আলিদের কি সারাজীবন একা থাকাই শ্রেয়?
উত্তর: গল্পের মিসির আলীরা একা থাকে, বাস্তবের মিসির আলীদের একা থাকতে হবে কেন?

প্রশ্ন: আসসালামুআলাইকুম, sir আমি হামদান আহমেদ ধনী, ঢাকা থেকে বলছি- আমার problem টা এমন যে কার সাথে share করব বুঝতে পারছিলাম না। অনেক ভেবে আজ আপনাকে লিখছি। sir,আমি class-7 এর student. But আমি যদি try করি,তাহলে আমার চেয়ে অনেক ওপরের class এর math বুঝতে পারি। আর math এর formula গুলো নিয়ে খেলতে আমি আসলেই ভালোবাসি। ………… …………… ………… ………… ………… ………… …… এলোমেলো সব ভাবনায় আমি কোনোভাবেই আর class-7 এ নিজেকে মেনে নিতে পরছি না বইয়ে মন বসাতে পারছি না,কোনোভাবেই না। আমি এখন কি করব? কিভাবে কী শিখবো কিছুই বুঝতে পারছি না। এ বিষয়ে আপনার advise চাইছি। please sir,আমাকে help করুন।
উত্তর: ইন্টারেস্টিং! তুমি কী গণিত অলিম্পিয়াডে অংশ নাও? তোমার অংশ নেওয়া উচিত, সেখানে আমরা গণিতে আগ্রহী ছেলেমেয়েদের আলাদা ভাবে গণিত শেখাই। তুমি ঠিকই বলেছ তোমাদের স্কুলের স্যারেরা তাদের নিয়মে গণিত না করলে তোমাকে কোনো নম্বর দেবে না। কাজেই একটু কষ্ট করে হলেও তুমি তোমাদের স্যারদের খুশি করে যাও এবং একই সাথে আলাদা করে নিজের গণিতের প্রতিভাটা বিকশিত কর। ও লেভেল এ লেভেলের গণিতের বইগুলো বের করে করতে থাকো, দেখ কত উচু ক্লাশ পর্যন্ত তুমি যেতে পারো। তুমি শুধু কি গণিতেই ভালো নাকি অন্য কিছুতেও ভালো? তোমার হতাশ হওয়ার কিছু নেই, তোমাকে ঠিকভাবে তোমার প্রতিভাটা কাজে লাগাতে হবে।

প্রশ্ন: নিজের কোনো গল্প পড়তে আমি আগ্রহ পাইনা। এটা কি আপনার ক্ষেত্রেও হয়? এই প্রশ্নের কোনো ব্যাখ্যা আছে? তানহা জুবায়ের আরাফ। হাতিরপুল, ঢাকা সপ্তম শ্রেণি। গবর্নমেন্ট ল্যাবরেটরি হাই স্কুল, ঢাকা।
উত্তর: না, এটা আমার বেলায় হয় না, আমি নিজের লেখা নিজে পড়তে পারি। যেহেতু কি লিখেছি এটা পুরোপুরি ভুলে বসে থাকি তাই অনেক সময়েই নিজের লেখা আগ্রহ নিয়ে একটা নুতন বইয়ের মত পড়ি!

প্রশ্ন: Sir, Plz ekta autograph diben? Name : Ananyo Ratim Siam School : CPSCR
উত্তর: কোনো সমস্যা নেই, তোমার ঠিকানাটা দাও। আমি কাগজে অটোগ্রাফ দিয়ে পাঠিয়ে দেব।

প্রশ্ন: sar ame wadi boyos 6 boshor ame age khob mubail dektam amar apoke deki apnar boy pore tai amoro isa holo apnar boy pora amar bapi rokomari teke boi kene dey mitu titu taim mesin ame khob moza paysi ame ar mubail deki na apni mune hoi amar apoke senen amar apo posno koresilu mohakas gubesok hobe poramosso diben ami apnar boro bokto amar apo shotobelay silet thakto boyos 1 silo amar zommer age amar issa ame apnar sate dika kolbo isalla apni amake ekta otogaf den pilis ai lab u sir ami kilas kg te pori apnar otograp ami iskinsot tuli rakbo bai bai sar sadpor asbe
উত্তর: তোমার বয়স মাত্র ছয় আর কুটুর কুটুর করে তুমি কি সুন্দর একটা চিঠি লিখে ফেলেছ! আমি পড়েও কত্তো মজা পেয়েছি। তোমার বাপী তোমার জন্য একটা বই কিনে দিয়েছেন সেটা তুমি পড়েও ফেলেছ। কী চমৎকার! অবশ্যি আমি তোমাকে অটীগ্রাফ দিব, তুমে তোমার ঠিকানাটা লিখে পাঠাও, আমি খামে ভরে তোমার অটোগ্রাফ পাঠিয়ে দেব।

প্রশ্ন: আসসালামু আলাইকুম স্যার, যেরকম টুনটুনি সেরকম ছোটাচ্চু বইটি এখানে দিলে ভালো হতো। (নুসরাত, চট্টগ্রাম)
উত্তর: আমি নিশ্চিত ইন্টারনেটে খোঁজখুজি করলে তুমি কোথাও না কোথাও পেয়ে যাবে, আমাকে কেউ পিডিএফ করে দিলে আপলোড করতে পারি, তা না হলে অনেক যন্ত্রণা।

প্রশ্ন: কেউ যদি আমার সাথে স্বার্থপরতা করে আমিও কি তার সাথে একই আচরণ করব?(নুসরাত ইসলাম, চট্টগ্রাম)
উত্তর: তুমি যদি নিজে স্বারথপর টাইপের না হও তাহলে চেষ্টা করেও স্বার্থপর হতে পারবে না। ঘাড়ে একটা ঘুষি দেওয়া বরং সহজ।

প্রশ্ন: Abrar Hossain 269/kha, Middle Pirer Bag, Mirpur, Dhaka. Sir, my dream is to be a teacher. Please write a book on the facinating aspects of teaching profession.
উত্তর: ঠিক আছে, দেখি পারি কিনা। আমার প্রায় সব বইয়েই কিন্তু কোনো একজন ভালো টিচারের গল্প আছে।

প্রশ্ন: আসসালামু আলাইকুম!! SIR কেমন আছেন? Borna চন্দনটুলা,সিলেট।
উত্তর: আমি ভালো আছি। তুমি কেমন আছ?

প্রশ্ন: স্যার! আপনার একটি বই “আমার সাইন্টিস মামা”এই বইয়ের পাঠ২ লেখবেন? please please! দয়া করে লেখবেন। নামঃ মোঃ ফয়সাল মাহমুদ (জাহিন)
উত্তর: টুনটুনি এবং ছোটাচ্চু সেকেন্ড পার্ট লিখে কি বিপদে পড়েছি! সেই বিপদ আবার ডেক আনব?

প্রশ্ন: আসসালামু আলাইকুম স্যার, আমি আপনার অনেক বই পড়েছি।”সাইক্লোন ” বইটি এই মাত্র পড়ে শেষ করেছি।সেখানে একটি প্রশ্ন ছিল,” চাঁদ পৃথিবীর চারপাশে একবার ঘুরে আাসে কিন্তু দুইবার জোয়ার আর দুইবার ভাটা হয় কেন?” এর উত্তর নেই।তাই উত্তর টা জানাবেন প্লিজ। ( নুসরাত ইসলাম। বি এন স্কুল ও কলেজ, চট্টগ্রাম। সপ্তম শ্রেণী।)( চট্টগ্রাম)
উত্তর: আমার ‘রহস্যময় ব্ল্যাক হোল’ বইটায় ব্যাখ্যাটি দেওয়া আছে। বইটি এই ওয়েবসাইটেই আছে!

প্রশ্ন: স্যার আসসালামু আলাইকুম। আশা করি ভালোই আছেন। অনেকদিন ধরে আপনার গল্প পাচ্ছিনা। লিখছেন না কেন? আমাদেরও তো সময় কাটাতে হবে। তামিম তাহসিন, চট্টগ্রাম।
উত্তর: শুধু আমার গল্প পড়ে যদি তোমার সময় কাটানোর পরিকল্পনা করে থাকো তাহলে তোমার জীবনে দুঃখ আছে!

প্রশ্ন: Name:Sakifa Sarwa Aahir Class:5School:Blue bird school and college sylhet
A lot of love Sir,I am your big fan.l saw you in Al Haramain hospital . That time I want to your autograph but that time I am in a pandamic time. Hope you are well by the grace of Allah.
Sir after reading your some books I learn some sience and some idea of how to solve any case.You make us happy and laugh by reading your book.God bless you.
উত্তর: থ্যাঙ্কু সাকিফা, আমার বই পড়ে তুমি অনেক কিছু শিখেছ শুনে খুশি হলাম। আমি কিন্তু গল্প কাহিনীর বইগুলোতে ভুলভাল অনেক কিছু লিখি, তাই উল্টাপালটা কিছু শিখে ফেললে কিন্তু আমার দোষ নেই!

প্রশ্ন: এখন কী এই Website এ লেখা হয় না? আমি খুব মিস করি,,, অনেক কষ্টকরে Website টা পেয়েছিলা।।
I am a big fan of your Science Fiction Name: MD SHIHHAD AHMED CLASS: 8 school:HOMNA GOVT HIGH SCHOOL I still waiting for your answer,,,,, যেতদিন বেচে থাকব ততদিন এই উত্তরের অপেক্ষায় থাকব।।
উত্তর: সর্বনাশ! এই উত্তরের জন্য যদি সারা জীবন অপেক্ষা করার প্ল্যান করে থাকো, তাহলে তো আমাকে এক্ষুনি উত্তর দিতে হবে! আমার সায়েন্স ফিকশান তোমার ভালো লাগে শুনে খুব খুশি হলাম।

২৮ ডিসেম্বর ২০২০

প্রশ্ন: আসসালামুয়ালাইকুম সার, আমি আপনার অনেক বড় ভক্ত। আমার নাম রুদ্র। আমি ক্লাস থ্রিতে পড়ি। আমার প্রশ্ন এই যে, আপনিও কি ছোটদের ডিটেকটিভ বই পড়তেন? ইতি: মুনতাসির আহসান ঠিকানা: রাজশাহী , ঝাউ তলা মোর, ৩৯০/৫ রাইয়ান। জিপিও,৬০০০।
উত্তর: অবশ্যই পড়তাম, সময় পেলে এখনও পড়ব!

প্রশ্ন: Sir, ami apnar onek boi porichi ,ami jante chachilam ,kishor uponnasher mothe apnar shobche pochondo konta? AFIA AZMAEEN (Terokhadia, Rajshahi)
উত্তর: মায়েরা যেরকম নিজের বাচ্চাদের মাঝে আলাদা করে কোন বাচ্চাটি সবচেয়ে পছন্দের সেটা বলতে পারে না, ঠিক সেরকম লেখকেরাও তার কোন বইটি সবচেয়ে পছন্দের সেটা বলতে পারে না। তবে আমার রাশা বইটি মনে হয় অনেক গোছানো ভাবে লেখা হয়েছিল।

প্রশ্ন: স্যার, বাংলাদেশের আয়ত ১৪৭৫৭০ বর্গ কিমি।আয়তন আবার বর্গ কিলোমিটার হয় কিভাবে ? আমার ক্লাসের সবার পক্ষ থেকে -মাজুজা, ৯ম শ্রেণি, বগুড়া
উত্তর: হয় না! এখন থেকে এটাকে ক্ষেত্রফল বলো।

প্রশ্ন: স্যার আসসালামু আলায়কুম। স্যার আপনি কেমন আছেন? আপনকে নিয়ে খুব ভয় হয় আমার,আমাদের সবার। আমি বেশ লক্ষ্য করছি আপনি আপনার নিয়মিত আমাদের জন্য করা অনেক কাজ করছেন না। আপনি কি ভালো আছেন? মাজুজা, ৯ম শ্রেণি, বগুড়া।
উত্তর: এই ওয়েবসাইটে ঘুরোঘুরি না করা ছাড়া আমি কোন কাজে ফাঁকি দিয়েছি? চিন্তা করো না, আমি ভালো আছি।

প্রশ্ন: স্যার, শ্রদ্ধা নিবেন। আমার বই পড়ার প্রতি আগ্রহ ক্লাস সিক্স/ সেভেনে থাকতে আপনার কিশোর উপন্যাসগুলো পড়েই তৈরী হয়েছে। তবে প্রায়ই মাথায় একটি প্রশ্ন আসে- যেই মানুষটা আমরা যাতে পড়তে পারি সেজন্যে এত এত বই লিখেন, সেই মানুষটা বই পড়ার সময়/ সুযোগ কতটা পান? আমি আসলে জানতে চাচ্ছি, শৈশবে আপনি বই পড়ার জন্য তুলনামূলক যেই অফুরন্ত সময়টা পেতেন, সেটা জীবনের বহমানতার কারণে যে এখন পান না, তাই কী আক্ষেপ হয়? ফারহাত মৃত্তিকা, ঢাকা সেনানিবাস।
উত্তর: শব্দটা হয়তো আক্ষেপ না, তবে মাঝে মাঝে মনে হয় জীবনটা যখন এত ব্যস্ত ছিল না, তখন কেন আরও বেশি করে বই পড়লাম না! কাজেই তোমরা যখন ছোট তখন অনেক বেশি বই পড়ো। অন্যকিছু না, বই পড়া মস্তিষ্কের একটা অসাধারণ ব্যবহার!

প্রশ্ন: স্যার, আপনি আপনার ওয়েবসাইটে কোনো এক প্রশ্নের উত্তরে বলেছিলেন- “ আমাদের observable বিশ্বব্রহ্মান্ডে প্রায় একশ বিলিয়ন গ্যালাক্সি, এবং প্রতিটি গ্যালাক্সিতে প্রায় একশ বিলিয়ন নক্ষত্র। (অসীম কথাটা বিভ্রান্তিজনক।)“ আমার প্রশ্ন হচ্ছে, ‘অসীম’ কথাটা কেন বিভ্রান্তিজনক? তোরসা, পুরান ঢাকা।
উত্তর: 1/x কিংবা 1/x^2 দুটোই অসীমের দিকে যাবে যদি x কে শূন্যের দিকে নিতে থাক, একটা অন্যটা থেকে তাড়াতাড়ি। তাহলে কী দুটো একই রকম অসীম? সব অসীম কী এক?

প্রশ্ন: শুভ জন্মদিন জাফর স্যার। জন্মদিনের অনেক অনেক শুভেচ্ছা। সামিহা, ময়মনসিংহ।
উত্তর: থ্যাংকু, থ্যাংকু সামিহা।

প্রশ্ন: Many many happy returns of the day. Happy birthday to you sir.🍰🍰🍰 Mirpur-1 Dhaka. Fariha ibnat
উত্তর: থ্যাংকু ভেরি মাচ ফারিহা!

প্রশ্ন: শুভ জন্মদিন স্যার অনেক অনেক শুভকামনা রইলো স্যার নাম : দীপান্বিতা সাহাবসুন্ধরা আবাসিক এলাকা , ঢাকা
উত্তর:থ্যাংকু থ্যাংকু দীপান্বিতা!

প্রশ্ন: স্যার, আপনাকে জন্মদিনের জন্য অনেক অনেক অনেক অনেক অনেক……(অসীমবার) শুভেচ্ছা।আপনার জন্য অনেক অনেক অনেক অনেক অনেক….(অসীমবার) দোয়া। ভালো থাকবেন আর আমাদের জন্য সুন্দর সুন্দর উপন্যাস লিখবেন। আমার জন্যও দোয়া করতে ভুলবেন না যেন!!!!!!!! এস.এম রাইয়ান তাওসীফ,সপ্তম শ্রেণি নিউ ইস্কাটন,মগবাজার,ঢাকা
উত্তর: থ্যাংকু থ্যাংকু রাইয়ান, অসীমবার থ্যাংকু । তোমার কথামত সুন্দর সুন্দর উপন্যাস লিখতে চেষ্টা করব, এবং দোয়া করে দিলাম, বড় হও, সত্যকারের মানুষ হও।

প্রশ্ন: শুভ জন্মদিন, স্যার। আপনার জন্মদিনটা যদি ২২ ডিসেম্বর হতো, তাহলে রামানুজনের জন্মতারিখের সাথে মিলে যেত। কী ভালোই না হতো। যাহোক, আপনাকে জন্মদিনের অনেক অনেক শুভেচ্ছা। ভালো থাকবেন স্যার। আর একটা কথা, আপনার জন্মদিনটাকে যদি একটা দিবসের নাম দেয়া যায়, তবে কোন নামটি দিলে আপনি সবচেয়ে বেশি খুশি হবেন? তানিম আহমেদ, ১০ম শ্রেণি, গলাচিপা, পটুয়াখালী।
উত্তর: “রামানুজন জন্মবার্ষিকীর পরের দিন” দিলে কেমন হয়?

প্রশ্ন:  Happy birthday dear sir 🍭🍭🍭🍬🍬🍬🎂🎂🎂🍰🍰🍰🎁🎁🎁🎉🎉🎉🎊🎊🎊 Kashfia Islam Sejuti from Mymensingh.
উত্তর: থ্যাংকু থ্যাংকু কাশফিয়া। (কত্তোগুলো কেক, খেয়ে শেষ করতে পারব না!)

প্রশ্ন: শুভ জন্মদিন স্যার, আজ ২৩ শে ডিসেম্বর, ২০২০ মধ্যরাত। আজ হঠাৎ করে একটি প্রশ্ন মাথায় আসলো। স্যার আমাদের জীবনরে তো অনেকগুলো ভাগ, শৈশব কৈশোর যৌবন বার্ধক্য। আপনার কি মনে হয় যুক্তি দিয়ে চলার বয়স কি যৌবন? আর শৈশব কৈশোর আর বার্ধক্য আবেগে চলার জন্য¿ একটু ভেবে বলবেন যেন! নিপুন পাল জয়নগর,নেত্রকোনা সদর, নেত্রকোনা
উত্তর: যুক্তি আর আবেগ কী একসাথে থাকতে পারে না? অবশ্যই পারে! বয়স বাড়লে আবেগ মনে হয় একটু কমতেই থাকে, দেখি বুড়ো মানুষ পেলে জিজ্ঞেস করে দেখব!

প্রশ্ন: আসসালামুয়ালাইকুম। আমি তাসমীম তাবাসসুম। আমি গাজীপুর থেকে বলছি। আমি ভিকারুননিসা নূন স্কুলের সপ্তম শ্রেণীতে পড়ছি। টুনটুনি ও ছোটাচ্চু সিরিজের সবগুলো বই আমি পড়েছি এবং আমার কাছে খুবই ভাল লেগেছে। প্রতি বছরই আমি এই বইটির জন্য অপেক্ষায় থাকি। আমি আসলে জানতে চাচ্ছিলাম যে, এবার ২০২০ সালের ভার্চুয়াল একুশে বইমেলায় কি আমার পছন্দের বইটির নতুন কোনো সংস্করণ বের হতে যাচ্ছে?(এই বইটি আমার ও আমার বন্ধুদের খুবই পছন্দের। তাই আমাদের একান্ত অনুরোধ দয়া করে এই বইটি লেখা বন্ধ করবেন না।) আশা করছি আপনি আমার প্রশ্নটির উত্তর প্রদান করবেন। তাহলে আমি খুবই খুশি হবো এবং উপকৃত হবো। ভালো থাকবেন। ধন্যবাদ।
উত্তর: এখনও বন্ধ করছি না। এই বইমেলার জন্য আবার লিখছি। ভয় হয় টুনি কোনোদিন এসে আমার উপর না চড়াও হয়ে বলে তাকে নিয়ে আর এতো বেশি টানাটানি না করার জন্য!

প্রশ্ন: স্যার, আমি তাছমিয়া জাহান সাওদা। ৮ম শ্রেনীতে পড়ছি।আমার বাসা আসলে ঢাকার বাইরে।ঝালকাঠি। তাই আমি একুশে বইমেলায় এসে আপনার অটোগ্রাফ নিতে পারি না।এটা আমার কাছে খুবই দুঃখজনক একটা ব্যপার।আপনি কি দয়া করে আমায় একটা খাতায় লেখা অটোগ্রাফ দিবেন?আমি অনেক খুশি হতাম স্যার।প্লিজ প্লিজ প্লিজ!!আমার একটা আনকমন অটোগ্রাফ চাই কিন্তু! প্লিজ স্যার প্লিজ!
উত্তর: তুমি যদি তোমার ঠিকানাটা লিখে দাও তাহলে আমি একটা কাগজে অটোগ্রাফ দিয়ে তোমার কাছে পাঠাতে পারি।

প্রশ্ন: স্যার কেমন আছেন? সিলেট শহরটা আপনার কাছে কেমন লাগে? ইয়াশফি চৌধুরি, সিলেট।
উত্তর: ভালো আছি এবং সিলেট শহর আমার খুব ভালো লাগে। আমি অবশ্য এখন আর সিলেটে নাই।

প্রশ্ন: আমি ইরফান সাদিক। পড়াশোনা করছি বাগেরহাট সরকারি উচ্চ বিদ্যালয়ে। আমার প্রশ্ন: আমাদের এসএসসি রসায়ন বইয়ের ৩য় অধ্যায়ে ম্যাক্সওয়েলের ইলেক্ট্রোম্যাগনেটিক থিওরি সম্পর্কে সামান্য আলোচনা করা হয়েছে। এখানে লেখা আছে, ‘ইলেক্ট্রন নিউক্লিয়াসকে কেন্দ্র করে ঘূর্ণনের সময় ক্রমাগত শক্তি হারাতে থাকবে এবং এক সময় তা নিউক্লিয়াসে পতিত হবে যার ফলে পরমাণু তার অস্তিত্ত্ব হারিয়ে ফেলবে। কিন্তু প্রকৃতিতে ইহা ঘটে না।’ – তাহলে এই তত্ত্ব কিভাবে প্রতিষ্ঠা পেলো যদি তা নাই ঘটে, আসলে তত্ত্বটিতে নিশ্চই অন্য কিছু বোঝানো হয়েছে যদি সেই বিষয়টা সম্পর্কে একটু বিস্তারিত আলোচনা করতেন…..?? আমার উত্তরটা জানার খুব ইচ্ছা….
উত্তর: এক লাইনে এর উত্তর দেওয়া খুব কঠিন। ‘ইলেক্ট্রন নিউক্লিয়াসকে কেন্দ্র করে ঘূর্ণনের সময় ক্রমাগত শক্তি হারাতে থাকবে এবং এক সময় তা নিউক্লিয়াসে পতিত’ হওয়ার ব্যাপারটি হচ্ছে ক্লাসিক্যাল ফিজিক্স। পরমানুর বেলায় যে এটা হয় না সেটা ব্যাখ্যা করার জন্য সম্পুর্ণ নূতন একটা তত্ত্ব ‘কোয়ান্টাম ফিজিক্স’ তৈরি করতে হয়েছে। কাজেই পরমানুর গঠন ব্যাখ্যা করার জন্য কোয়ান্টাম ফিজিক্স ব্যবহার করতে হয়।

প্রশ্ন: স্যার, আপনি যখন কোন ফিকশন বই লেখতে বসেন তখন কি আগে থেকেই মাথায় কোন প্লট থাকে? বাংলা সাহিত্যে আপনার সবচেয়ে প্রিয় লেখক কে? (আমার একটা ইচ্ছে আছে যে একদিন আপনার সাথে দেখা হবে…)। ভালো থাকবেন স্যার।নিভান, পটুয়াখালী।
উত্তর: মোটামুটি থাকে, লিখতে লিখতে একটু পালটে যায়। প্রিয় লেখক অসংখ্য, বলে শেষ করতে পারব না। অবশ্যি কোনো একদিন দেখা হবে।

প্রশ্ন: Sir apni koba tuntuni o chotaccur notun Part Likhban? Bivor . Class:2 Faridpur.
উত্তর: এখন বসে বসে লিখছি। ক্লাশ টুয়ের একজন যদি চায় তাহলে আমি কি না লিখে পারি?

প্রশ্ন: Sir ami apnar onak baro fan Bivor Class: 2 Faridpur.
উত্তর: থ্যাংকু বিভোর!

প্রশ্ন: Good day sir. I really wanted to talk about this with a scientist. And then I found this website. …।। I am ……. From: Uttara, Dhaka.
উত্তর: তোমার প্রশ্নের উত্তর আসলে তোমার নিজের খুঁজে বের করতে হবে। বিষয়টা নিয়ে চিন্তা করতে থাকো, শুধু মনে রাখবে বিজ্ঞান সবকিছু ব্যাখা করে যুক্তি তর্ক দিয়ে, পরীক্ষা নিরীক্ষা দিয়ে। পৃথিবীর যে কোনো মানুষ বিজ্ঞানের যে কোনো বিষয় চ্যালেঞ্জ করতে পারে। কিন্তু অনেক কিছুই আছে যেগুলো ব্যাখ্যা করা হয় বিশ্বাস দিয়ে। দুটি আলাদা ধরনের জ্ঞান এবং একটা দিয়ে অন্যটি ব্যাখ্যা করার চেষ্টা করে লাভ নেই।

প্রশ্ন: জাফর ইকবাল স্যার, আমি আপনার একজন ভক্ত(মূলত আপনার বইয়ের ভক্ত)। আগামী ২২ ডিসেম্বর আমার জন্মদিন। আমি কি আপনার থেকে আমার জন্মদিনের শুভেচ্ছা পেতে পারি?স্যার আর, আমার জন্য সবচেয়ে খুশির কথা হল যে, আপনার আর আমার জন্মদিন পাশাপাশি। আপনাকেও আমার পক্ষ থেকে অগ্রিম শুভেচ্ছা রইল আপনার জন্মদিনের। নামঃ নুসরাত জাহান তুলি,
উত্তর: আমি খুবই দুঃখিত, একটু দেরি করে ফেললাম! দেরি করেই তোমার জন্মদিনের ভালোবাসা জানাই?  “শুভ জন্মদিন তুলি!”

প্রশ্ন: Sir,wave function ashole kisher torongo? PARAG BISWAS,class 7,FARIDPUR ZILLA SCHOOL
উত্তর: এটা যে জিনিষটার তরঙ্গ, সেটাকে বর্গ করা হলে প্রোবাবিলিটি বা সম্ভাবনা পাওয়া যায়। ঘোট পাকিয়ে দিলাম?

প্রশ্ন: আসসালামু আলাইকুম স্যার।আমি তাছমিয়া জাহান সাওদা।বর্তমানে ৮ম শ্রেণিতে পড়ছি। ঝালকাঠি থেকে বলছি।আমি বই পড়তে অনেক ভালোবাসি।বিশেষ করে আপনার বই হলে তো কথাই নেই!আমি কিছুদিন আগে “আমার বন্ধু রাশেদ”পড়ছি।যেদিন বইটা পড়া শেষ করলাম,সেদিনই আমাদের ফ্যামিলি ট্রিতে একটা নতুন সদস্য যোগ হলো।আমার মামাত ভাই।নাম দিতে গিয়েয়ে আমার মনে পড়ল,বইটাতে ইবুর নামকরন নিয়ে কথাটা।তাই আমি আমার মামাত ভাইয়ের নাম “লাড্ডু “দিয়ে দিলাম।ভালো করেছি না,স্যার?
উত্তর: খুবই ভালো করেছ, কিন্তু লাড্ডু যখন একটু বড় হবে তখন তার থেকে সাবধান। সে যখন জানতে পারবে তুমি তাকে এই নাম দিয়েছ তখন তোমার কপালে অনেক দুঃখ আছে!

প্রশ্ন: আমি আপনার সবগুলো সায়েন্স ফিকশন গল্প/উপন্যাস পড়েছি। আপনার কি ভবিষ্যতে আর এরকম গল্প/উপন্যাস লেখার ইচ্ছা আছে? যদি না থাকে তাহলে আমি এরকম উপন্যাস লেখার জন্যে অনুরোধ করছি। {নামঃ জিকরুল্লাহ,জেলাঃ পাবনা। }
উত্তর: হ্যাঁ ইচ্ছা আছে এবং লিখে যাচ্ছি লিখে যাচ্ছি লিখে যাচ্ছি …

প্রশ্ন: স্যার, আমি আপনার নিউরনে অনুরণন বইটি পড়ার সময় গোল্ডবাকের অনুমানটি নিয়ে ভেবেছি ও এর একটি সমাধান বের করেছি। আপনি যদি একটু সময় দেন তাহলে অত্যন্ত কৃতজ্ঞ হব। মূল্যবান সময় নষ্ট করার জন্য দুঃখিত। ইতি রাইয়্যাত জারিফ বিজিবি ১ নং গেট
উত্তর: পৃথিবীর বাঘা বাঘা গণিতবিদেরা যেহেতু দিনের পর দিন চেষ্টা করে এর সমাধান করতে পারেননি, তাই আমি জানি এর সমাধান বোঝার ক্ষমতা আমার নেই। তোমার কোনো গণিতবিদের সাথে কথা বলতে হবে।

প্রশ্ন: মুহাম্মদ জাফর ইকবাল স্যার, আপনার নাম কী? মনামী জামান,ঢাকা
উত্তর: আমি যতদূর জানি আমার নাম মুহম্মদ জাফর ইকবাল

প্রশ্ন: আমাদেরকে ছোট বেলা থেকেই ব্যাকরণ শেখানো হয়। আমি মনে করি ব্যাকরণ শিখে বাস্তব জীবনে কোন লাভ হয় না।যেই সময়টা আমরা ব্যাকরণ এর মত কিছু কাজলের ব্যবহার নেই সেগুলো শেখার পেছনে যে সময় ব্যয় করি সেই সময়টা স্কিল ডেভলপমেন্ট কোন কাজ (যেমন প্রোগ্রামিং শেখা) এর জন্য ব্যয় করা যেতে পারে। কাউকে ব্যাকরণ শিক্ষার গুরুত্ব জিজ্ঞেস করলেই সে বলে ব্যাকরণ ছাড়া ভাষা অসম্পূর্ণ।কিন্তু আমরা আগে কি শিখেছি আগে কি ব্যাকরণ শিখেছি নাকি কথা বলা শিখেছি? আচ্ছা মানলাম ব্যাকরণ শিখতে হবে, কিন্তু আমরা বড় হয়ে তো আর ভাষাবিদ হবো না। তাই ব্যাকরণ এর বেসিক নলেজ টা থাকলেই হবে বলে আমি মনে করি। কিন্তু আমাদেরকে জোর করে শেখানো হয় সমাস, কারক, সন্ধি, আরো কত কি। *শিক্ষা অর্জনের জন্য কি আসলেই এগুলোর প্রয়োজন আছে? আরিফ হাসনাত, অষ্টম শ্রেণি, সীমান্ত পাবলিক স্কুল, নওগাঁ
উত্তর: পড়াশোনা মানে কিন্তু কিছু স্কিল ডেভেলপমেন্ট না! শুধু যেটা প্রয়োজন সেটাই যদি শিখবে তাহলে স্কুল কলেজ বিশ্ববিদ্যালয় তৈরি না করে রাস্তার মোড়ে মোড়ে ট্রেইনিং সেন্টার তৈরি করেই কাজ চলে যেতো। জীবনের একটা সময়ে কিন্তু তুমি হঠাৎ করে আবিষ্কার করবে যে সমাস কি, কারক কি, সন্ধি কি সেটা জানতে হচ্ছে। তাহলে তোমাকে সেটা কখন শেখানো সঠিক সময়?

প্রশ্ন: স্যার একটুখানি বিজ্ঞান তো পাইলাম।আরো একটুখানি বিজ্ঞান ও পাইলাম।।এখন আরো আরো একটুখানি বিজ্ঞান কবে পাব? নামঃ রিসালাত মুখর LYCEUM High School Gangni, Meherpur
উত্তর: আমার থেকেই পেতে হবে কে বলেছে? আরো কত জায়গায় আরো কত বিজ্ঞান আছে তুমি কি জান? (পাইলাম না লিখে পেলাম লিখলে আরেকটু সুন্দর হতো!)

প্রশ্ন: স্যার, জ্যামিতি ও কম্বিনেটরিক্সের জন্য সবচেয়ে ভালো বই কোনটা?
উত্তর: আমার যেহেতু এটা পড়তে কিংবা পড়াতে হয় না, তাই “সবচেয়ে ভালো বই” কোনটা বলতে পারব না। ইন্টারনেটে একটা সার্চ দাও।

প্রশ্ন: স্যার, আমি শাহরিয়ার আহমেদ জিসান, কিশোরগঞ্জ থেকে বলছি, স্যার আমাদের বড়দের জন্যও একটু লেখেন প্লিজ, আপনার বই আমার অনেক ভালো লাগে, কিন্তু বন্ধুরা আপানার বই পড়তে দেখলেই বলে আরে ব্যটা জাফর ইকবালের বই বাদ দিয়া হুমায়ুন আহমেদ বই ধর, তাই স্যার প্লিজ আমাদের জন্যও লেখেন। পুনশ্চঃ( আমি আসলে খুব বেশি বড় হই নাই মাত্র ইন্টার 2year রে পড়ি)।
উত্তর: তুমি জানো দুনিয়াতে বড়দের জন্য কত অসাধারণ বই লেখা হয়েছে? জানতে চাও? তাছাড়া হুমায়ুন আহমেদ ধরতে ক্ষতি কি?

প্রশ্ন: স্যার, আমার নাম বন্যা। আমি চট্টগ্রামে থাকি। আপনি এবছর বইমেলায় টুনটুনি ও ছোটাচ্চু সমগ্র বের করেছেন, তাহলে আপনি কি আর টুনটুনি ও ছোটাচ্চু এর কোনো বই বের করবেন না??? যদি আপনার উত্তর না হয় তাহলে আমি খুব কষ্ট পাবো।তাহলে বলবো,কেন আপনি টুনটুনি ও ছোটাচ্চু এর কোনো বই লিখবেন না??? এই বইটির সিরিজ আমার খুব খুব পছন্দের। আর আমি বলছি, আপনি এই সিরিজ লেখা বন্ধ করতে পারবেন না।
উত্তর: না তোমার কষ্ট পেতে হবে না। আমি এখনও লিখছি। মনে হচ্ছে টুনি বড় হওয়ার পর, লেখা পড়া শেষ করার পর, তার বিয়ে শাদী হওয়ার পর, বাচ্চা কাচ্চা হওয়ার পর যখন নিজের কাজকর্ম শুরু করবে তখন পর্যন্ত লিখতে হবে!

প্রশ্ন: স্যার আপনার লেখা “বিজ্ঞানের ১০০ মজার খেলা” বইটি আমি কিছুদিন হল কিনেছি,মাস শেষে দেখি সব এক্সপেরিমেন্ট করা শেষ.এখন এমন আরেকটা বই পেতে পারি? (তানবিন ইসলাম. শ্রেনি: 6 (বাংলা এ না লিখতে পারার জন্য সরি).ব্রাইট স্কুল আ্যন্ড কলেজ .ডে শিফট
উত্তর: মনে হচ্ছে আরেকটা লিখতে হবে। দেখি চেষ্টা করে।

প্রশ্ন: Is Serina still in the water, Sir? And the ocean, under which she is flaunting yet, might be the Indian ocean. Won’t that be, sir? Wouldn’t that be a little bit childish if I tell you that I would someday attempt for at least one time to find her out of the abysmal water? Yes, Sir. I must try it. I got to. I am sorry, but I can’t help asking you why it had to be ended like that way? How much I feel for Serina, even Icthyander wound not catch it! Such a heart-wrencing ending! Such an incredible sensation! That little girl alone under the sea! That’s not just fair, sir. Not a bit fair. – Sadia, Dhaka.
উত্তর: সাদিয়া তুমি এভাবে বলে আমার বুকটা ভেঙ্গে দিলে। প্লিজ তাকে খুঁজে বের করে আনো, প্লিজ! আমরা সবাই তাকে দেখতে চাই, তার সাথে কথা বলতে চাই।

৭ ডিসেম্বর ২০১৯ থেকে ১ ফেব্রুয়ারি ২০২০

প্রশ্ন: Sir aber ami class 9 a odlam. Ami khub kushi hoyecilam j apni amader boi likcan. Kintu allah jibbiggan a ato mukostor pora.ami ar rokom axcept korini. Ar mukosso sommov hoccy na. Apni hoyto bolben mukosso kora lagby na bojo.kintu amio ssc ty vlo korty cai. Ata ki sommov.ami science neicilam karon ami doctor hoty cai . Dasermanuser saba korty cai. Kintu jai dasher boi amaky kosto dai. Tar ami ki korbo.apnaky ami kono prosno ba bolci na kicu koren. But yet l want say that l love science but l hate biology .kintu ami jani amaky atai na bujy porty hoby. Tai obosasy apner kas thaky kicu tips nity cacci.ami jani ami parbo jai osikkitho manusher jonno amar ai obosta tader thaky amaky sikkito hoty hoby dash goton korty hoby.tai plz tips diya help korini sir ami swapneel,Meherpur gov’t girls high school. Class 9 .roll 5(science) A shift Day
উত্তর: কিছু নামতা আর সুন্দর সুন্দর কবিতা ছাড়া আর কিছু আমি মুখস্ত করার পক্ষে নেই। একটা তথ্য যেটা বইয়ে আছে সেটা কেন আমার মস্তিষ্কে রাখতে হবে? মস্তিষ্কের কাজ হচ্ছে সমস্যা সমাধান করা কিংবা সমস্যা বিশ্লেষণ করা। আমার হাতে যদি দায়িত্ব থাকতো তাহলে আমি সব পরীক্ষা নিতাম বই দেখার সুযোগ দিয়ে। এখন কী করবে, যেটুকউ পার সিস্টেমের সাথে তাল মিলিয়ে যাও। সবাই যেহেতু যাচ্ছে, তুমিও যেতে পারবে।

প্রশ্ন: প্রিয় স্যার, আমি আপনার অনেক বড় ভক্ত । কথাটা শুনে নিশ্চয়ই আপনি অভ্যস্ত । তারপরও বলি, আপনার বই পড়ে আমি কল্পনা করতে শিখেছি। একদিন কল্পনা করলাম আপনি আমার জন্মদিন এ এসেছেন!! আর এখন আমি এ কল্পনাতে ভাসছি । শুনে একটু আজব লাগবে জানি, কিন্তু তবু বলি, আপনার সাথে আমার ৩বার দেখা হয়েছে। আপনার মনে থাকার কথা নয়, যা হোক, সামনে মার্চ মাসে আপনি যদি ফ্রি থাকেন ও ঢাকায় আসেন তবে কি ৪ তারিখ আমার কল্পনা বাস্তবায়ন করে দিবেন পুনশ্চ: আমার নিজেরই লেখাটা পড়ে মনে হচ্ছে আমি পাগল!!! কিন্তু আমি তা নই, কেবল একটা ক্ষুদ্র ইচ্ছা প্রকাশ করে ফেললাম। আপনার আদর্শ অপহরণকারী শ্রেয়া সাহা ধানমন্ডি,ঢাকা।
উত্তর: না না তুমি পাগল কেন হবে। তোমার জন্মদিনে তুমি তো আমাকে আমন্ত্রণ জানাতেই পার। এবারে অবশ্য হল না কারন আজকে মার্চ মাসের ৯ তারিখ। তোমাকে জন্মদিনের ভালোবাসা আর একটা কেক! (খেতে পারবে না যদিও)

প্রশ্ন: Ye sir tuntuni o chotacchu k aibar book fair a anen to. R koto wait korbo?
উত্তর: নাম নাই তাই উত্তর নাই! সরি!

প্রশ্ন: আপনার অনেক দিন থেকেই দাবি রেখে আসছিলেন ‘গুচ্ছ ভিত্তিতে’ বিশ্ববিদ্যালয় ভর্তি পরিক্ষার। অবশেষে সেটা বোধহয় বাস্তবিত হতে চলেছে। আমি একসময় Admission test দিয়েছি, জানি কি একটা ধকল যায়, তাই একটা ধন্যবাদ দিতে ইচ্ছে করছে সিদ্ধান্তে।
কিন্ত অনেক শিক্ষকই এইটার বিরোধ করছেন। তাদের analysis হচ্ছে, এতে নিজেদের পছন্দ আর যোগ্যতামত শিক্ষার্থী ভর্তি নেয়া সম্ভব না। (তাদের অনেকেই honest, admission এর টাকা লুটপাট করার মত দুরভিসন্ধি তাদের নেই বলেই জানি)
sir, আপনার proposal টা খুবই সুন্দর, শিক্ষার্থীদের ভোগান্তি যে একেবারে লাঘব করবে, এত কোনই সন্দেহ নেই। আমার ব্যক্তিগত একটা proposal আর একটা অনুরোধ আছে। Prosposal, একটা গুচ্ছ পরিক্ষা না করে, কয়েকটি গুচ্ছ পরিক্ষা করা হোক। যেমন School of Enginnering এর একটি exam হতে পারে, যেখান থেকে সারা দেশের Engineering Universities(BUET, KUET..) গুলো সহ বিভিন্ন Universities এর Engineering unit এ ভর্তি নেয়া হবে। এইভাবে সব কৃষি বিশ্ববিদ্যালয় একসাথে। সব Social Science একসাথে। এইভাবে সবগুলো…
অনুরোধটি হচ্ছে, আপনি যেভাবে মাধ্যমিকের বইগুলো একদল যোগ্য-কর্মঠ লোক নিয়ে সুন্দর করে বানিয়েছেন। গুচ্ছ পদ্ধতির এই ভর্তি মহাযজ্ঞটিও আপনি পক্ষে-বিপক্ষে(honest) লোকদের সাথে নিয়ে করবেন।
এত বড় উত্তর এখানে দেবার প্রয়োজন নেই, আপনার কলামেরা অপেক্ষায় আছি, সেখানেই এ নিয়ে আপনার বিস্তারিত মতামত পেয়ে যাব, আশা রাখি। এই site এ যেহেতু আপনি উত্তর দেন, সেই লেখার link দিয়ে দিয়েন, অন্যেরাও পড়ে নেবে। বরাবরের মত অনেক অনেক ভালবাসা ও শুভকামনা। Julian Jawad Ahmad, Uttara, Dhaka.
উত্তর: আমি ভর্তি পরীক্ষা নিয়ে যথেষ্ট চেচামেচি করেছি। এখন আর করছি না। তার কারণ আমার মনে হয় গুরুত্বপূর্ণ কোনো মানুষ আমাকে দুই চোখে দেখতে পারে না। (শুধু ছোট বাচ্চারা আমাকে ভালোবাসে!) তাই আমি যখন কিছু একটা বলি তখন সবাই সেটার বিরুদ্ধে কাজ করতে শুরু করে! ভর্তি পরীক্ষার ব্যাপারে এরকম কিছু ঘটুক আমি সেটা চাই না!

প্রশ্ন: Sir, I got A(+) in all subject. Sagor (Do you remember who am I?.)
উত্তর: কংগ্রাচুলেশন্স! (না, ঠিক মনে করতে পারছি না, আমার স্মৃতি শক্তি বলে কিছু নেই!)

প্রশ্ন: টক খেলে চোখ বন্ধ হয়ে যায় কেন? কাজী মাইশা তাজরীন। কুমিল্লা।
উত্তর: যায় নাকী? কি আশ্চর্য!

প্রশ্ন: হাই তুললে চোখে পানি আসে কেন? স্যার, আমি আপনার প্রায় সব বই পরে ফেলেছি। আশা করি আরও বেশি বেশি করে আমাদের জন্য বই লিখবেন। কাজী মাইশা তাজরীন। কুমিল্লা।
উত্তর: তোমার চোখে এত মজার মজার ঘটনা ঘটে ব্যাপারটা কী? (হ্যাঁ, লিখতে চেষ্টা করব। আমি তো শুধু তোমাদের জন্যই লিখি!)

প্রশ্ন: আপনার বইয়ের সমগ্ৰের online সংস্করণের URL link চাই।হাসান জাকির।দ. কাচারিপাড়া, জামালপু্র। ‌‌‌‌‌‌‌। পুনশ্পচ: আপনার সুস্বাস্থ কামনা করি।
উত্তর: আমার তো জানা নেই। তুমি যদি খুঁজে পাও আমাকে জানিও আমি এখানে সবার জন্য প্রকাশ করে দেব।

প্রশ্ন: স্যার আমার সালাম নিবেন।আপনাকে একটা প্রস্ন করার ছিল।আপনার সায়েন্স ফিক্সন বই ”অন্দকারের গ্রহ” এর লাসট এর জায়গা টা বুজতে পারলাম নাহ।দয়া করে একটু বুজিয়ে দিলে ভালো হোতো।Name:Md.Mahir Afser Rahat Age:15 Address:Mirpur no.1,Dhaka
উত্তর: হা হা হা! বই লিখে সেটা যদি আবার বুঝিয়ে দিতে হয় তাহলে বুঝতে হবে আমার লেখাটার মঝেই সমস্যা! তুমি বরং এই বইয়ের পিছনে সময় নষ্ট না করে অন্য বই পড়। (তা ছাড়া সেই বইয়ে কি লিখেছিলাম এতদিন পরে মনেও নেই।)

প্রশ্ন: সার, আপনি অনেক বিজ্ঞান এর গনিতের বই লিকেছেন । তা থেকে ৭ টি বই নিয়ে “বিজ্ঞান ও গনিত সমগ্র” । বাকিগুল নিয়ে আরেকটি বই সমগ্র আকারে বের করলে আমার মত মধ্যবিত্তদের উপকার হতো । ফাহাত মাহমুদ তাল বারিয়া ,বাকিরগঞ্জ উপজেলা,কুমিল্লা সদর ,কুমিল্লা
আপনি সর্বমোট কতগুলো বই লিখেছেন? Afia jannat ,dhaka
উত্তর: এখনও দ্বিতীয় সমগ্র বের করার মত যথেষ্ট বই হয়নি। এই সাইটটাতেই আমার বইয়ের তালিকা আছে, সেখানে বইয়ের সংখ্যাটা দেখে নাও।

প্রশ্ন: Name:Adina Hossain Address:221/A,Moghbazar,Noyatola,Dhaka.
Sir, shobai bole,bhalo schoole porlei naki bhalo result kora jai.kothata-ke thik
উত্তর: না, কথাটা মোটেও ঠিক না। পড়া লেখা নিজের উপর, স্কুলের উপর না। তা ছাড়া ভালো স্কুল বলতে তুমি যেটা বোঝাও সেগুলো ভালো স্কুলের নাম রাখার জন্য যারা পরীক্ষায় খারাপ করে তাদেরকে স্কুলে রেখে পড়ায় না।

প্রশ্ন: স্যার, আপনার কয়টি বই বের হয়েছে , ইমতিয়াজ আহম্মেদ, টিএনটি রোড, ভান্ডারিয়া, পিরোজপুর।
উত্তর: এই সাইটটাতেই আমার বইয়ের তালিকা আছে, সেখানে বইয়ের সংখ্যাটা দেখে নাও।

প্রশ্ন: ইদানীং আপনার বইয়ের দাম এত বেড়ে গেছে যে সে টাকা দিয়ে অনায়াসেই অন্যান্য লেখকের ২-৩ টা বই কেনা যাবে।আমি এটা বলছি না সব লেখকের বইয়ের মান একই।তবে আপনি ই প্রকাশনীকে বলে বইয়ের দাম একটু কমাতে পারেন না?১০০০ টাকা সাথে বই মেলায় গেলে সর্বোচ্চ আপনার ৩টা বই কেনা যাবে,তাহলে অন্যান্য লেখকের বই কিনব কীভাবে???????????????আমার আবার বই জমানোর খুব শখ,মোটামুটি ৩০০ বই আমার সংগ্রহে আছে।এখন আপনিই বলুন কোন মা-বাবা বই কেনার জন্য হাজার হাজার টাকা দেবে?আমরা যেটুকু জমায় তা আসলে এত দামী বই কেনার জন্য যথেষ্ট না।তাই আমরা অনেকেই অপেক্ষা করি কবে বইটার পিডিএফ ভার্সন আসবে(যা বেশিরভাগ সময়ই কোন আইনের তোয়াক্কা না করে ছাসড়া হয়)কিন্তু এভাবে আর কতদিন>>>>>>>??????????ঃ( সাজ্জাদুর রহমান রাজউক উত্তরা মডেল কলেজ
উত্তর: আমরা যখন তোমাদের বয়সী ছিলাম, তখন বইয়ের সংগ্রহ নিয়ে মাথা ঘামাইনি। বই পড়ার চেষ্টা করেছি। আমি সবসময় আমার পাবলিশারদের দাম কমাতে বলে আসছি, আর কী করার আছে? আমার বইয়ের পিডিএফ নিয়ে আইনী সমস্যা নেই। যতখুশি করতে থাকুক।

প্রশ্ন: স্যার আপনাকে একটি কাজ করতে হবে । এই বইমেলায় আপনার যেসব বই বের হবে তার নাম এবং ধরণ লিখে দিন প্লিজ…প্লিজ…প্লিজ…প্লিজ…প্লিজ…প্লিজ…প্লিজ…প্লিজ…প্লিজ…প্লিজ…প্লিজ…প্লিজ…প্লিজ …………………………….. মোস্তফা নুরুল আলম , দশম শ্রেণী , গোপালগঞ্জ ।
উত্তর: বই মেলা কবে শেষ হয়ে গেছে, এখনো কি নাম জানতে চাও? এই সাইটটাতেই আমার বইয়ের তালিকা এবং ধরণ দেয়া আছে, একটু কষ্ট করে দেখে নাও, প্লিজ!

প্রশ্ন: স্যার আমি আপনাকে এই প্রশ্ন আগে করেছি কিনা তাই নিয়ে সন্দেহ ছিল । তাই আবার করছি । ভরকে যেমন শক্তিতে রূপান্তর করা যায় তেমনি শক্তিকে কি ভরে রূপান্তর করা যায় ???????????????????????????????? মোস্তফা নুরুল আলম , দশম শ্রেণী , গোপালগঞ্জ ।
উত্তর: আমার যতদূর মনে পড়ে আমি এই প্রশ্নের উত্তর একবার দিয়েছি। আবার দিচ্ছি। হ্যাঁ, শক্তিকে ভরে রুপান্তর করা যায়। সবচেয়ে সহজ উদাহরণ হচ্ছে পেয়ার প্রোডাকশন, যেখানে গামা রে থেকে ইলেকট্রন এবং পজিট্রন তৈরি হয়। (আরো বেশি জানতে চাইলে pair production লিখে ইন্টারনেটে একটা সার্চ দাও।)

প্রশ্ন: তাড়াতাড়ি উত্তর দেন বইমেলায় কবে কবে যাবেন??????????????????????? JABIR KUSHTIA
উত্তর: বই মেলা কবে শেষ! উত্তর কেমন করে দেব?

প্রশ্ন: নাম পরিচয় গোপন রাখতে চাই। ……………………… এছাড়া আরো কিছু আইডিয়া ছিলো। বাকিটা আপনার উত্তরের অপেক্ষায়।
উত্তর: আমাকে বললে আমি নাম গোপন করে দেব, কিন্তু যদি আমাকেও নাম পরিচয় না লিখ, উত্তর দেব কেমন করে?

প্রশ্ন: আপনি গ্রামের নাম কাকনডুবি আরেকটি সিকোয়েল লিখেন। প্লিজ ! !প্লিজ ! প্লিজ ! প্লিজ ! আহনাফ প্রত্যয়, টাংগাইল।
উত্তর: আমি আসলে কখনো সিকোয়েল লিখি না! সরি! (টুনটুনির দ্বিতীয় বই লিখে কত বড় বিপদে পড়েছি দেখছ না?)

প্রশ্ন: স্যার, “আমি তপু” এর মত আরেকটা বই লিখবেন? এই বইমেলায় আপনার কয়টা কিশোর উপন্যাস বের হবে? আনিসুর রহমান লিমন, টাংগাইল
উত্তর: কখনো কি একটা বইয়ের মত আরেকটা বই লেখা যায়? আবার হয়তো কখনো একটা বই লেখা হয়ে যাবে যেটা তোমার ভালো লেগে যাব। সেজন্য তুমিও অপেক্ষা কর, আমিও করি।

প্রশ্ন: আপনি কি টুনটুনি সিরিজের নতুন বই বের করবেন এই বইমেলায়? আহনাফ প্রত্যয়, টাংগাইল।
উত্তর: বই মেলা শেষ, উত্তর কি দিতে হবে?

প্রশ্ন: হাসিন,ঢাকা ।স্যার আপনি কখনো ডিজনি ওয়ার্লডে গিয়েছেন ?
উত্তর: গিয়েছি। ওসব জায়গায় বাচ্চাদের নিয়ে যেতে হয়! বড়দের  জন্য বেশি মজা নেই!

প্রশ্ন: এখন পর্যন্ত টুসটুনি ও ছোটাচ্চু সিরিজের কয়টি বই বের হয়েছে। ইমতিয়াজ আহমেদ তাওসিফ। ভান্ডারিয়া।
উত্তর: এই সাইটটাতেই আমার বইয়ের তালিকা দেয়া আছে, একটু কষ্ট করে দেখে নাও, প্লিজ!

প্রশ্ন: ২০২০ সালে আপনার নতুন বই কি কি? এই বই গুলোর প্রচ্ছদের ছবি দিন প্লিজ !প্লিজ! প্লিজ ! প্লিজ ! প্লিজ! আহনাফ প্রত্যয়, টাংগাইল।
উত্তর: বই মেলা কবে শেষ হয়ে গেছে, এখনো কি দরকার আছে? তবু দিলাম তোমার জন্য।

প্রশ্ন: Dear Sir, how are you?What about your physical condition? where are you now living this moment? Thanks. -Purnata, Jamalpur.
উত্তর: আমি এখন ঢাকায়, মনে হয় ভালোই আছি!

প্রশ্ন: আপনি কেমন আছেন স্যার?
উত্তর: নাম নাই তাই উত্তর নাই।

প্রশ্ন: মানুষের যখন পতন আসে তখন পদে পদে ভুল হতে থাকে কোন প্রসঙ্গে লেখক এ কথা বলেছিলেন?
উত্তর: নাম নাই তাই উত্তর নাই!

প্রশ্ন:,,,,,,,,,♥️♥️♥️♥️♥️♥️♥️♥️♥️♥️♥️♥️♥️♥️♥️♥️♥️♥️♥️♥️♥️♥️♥️♥️♥️♥️♥️♥️♥️♥️♥️♥️♥️♥️♥️♥️♥️♥️♥️♥️♥️♥️♥️♥️♥️♥️♥️♥️♥️For you dear zafor sir,Preontie………From Bagura.Class 6.Good bye sir, May Allah bless you.
…………………………………………………,………..,……………………………………………………………………………………
এও খানি ভালোবাসা আপনার জন্য ♥️♥️♥️♥️♥️♥️♥️♥️♥️♥️♥️♥️♥️♥️♥️♥️♥️
উত্তর: থ্যাঙ্ক ইউ থ্যাঙ্ক ইউ! তোমার এত্তো খানি ভালোবাসা রাখব কোথায়? বাসায় তো জায়গাই নেই!

প্রশ্ন: আপনি জন্মদিনে কয়টি ই-মেইল পেয়েছেন? আমার টা কি দেখেছেন?(…………,গোপন রাখবেন।)
উত্তর: বেশ অনেকগুলো পেয়েছি, তোমারটাও নিশ্চয়ই পেয়েছি।

প্রশ্ন: স্যার,একটি স্ট্রয়ে কয়টি ফুটো থাকে? – কাজী মাইশা তাজরীন। কুমিল্লা।
উত্তর: মনে হচ্ছে এই প্রশ্নটা tricky প্রশ্ন! স্ট্রটা চিপা দিয়ে flat করে ফেললে একটা ফুটো হবে সেটূকু বলতে পারি।

প্রশ্ন: Sir Ami class 10 a pori.Sobai bole Ami programming shikar Jonno naki Choto apni ki mone Loren Ashiqul Islam Ayon Brahmanbaria Bangladesh Gas Fields School & College
উত্তর: না, ক্লাশ টেন প্রোগ্রামিং শেখার জন্য মোটেও বেশি ছোট নয়। আমরা আরো ছোটো ছেলেমেয়েদেরকে প্রোগ্রামিং করতে দেখেছি।

প্রশ্ন: তন্ময় রায়, বরিশাল (স্যার আমার প্রশ্নটা মহাবিশ্বের সীমা নিয়ে, আমার মনে যে ধারনা আছে তা হল: মহাবিশ্বে কোটি কোটি গ্যালাক্সি আছে, আর প্রত্যেকটি গ্যালাক্সিতে আছে অসীম সংখ্যক গ্রহ উপগ্রহ) এই ধারনাটি কতটুকু সত্যি?
উত্তর: আরো নিঁখুত ভাবা বলা যায়। আমাদের observable বিশ্বব্রহ্মান্ডে প্রায় একশ বিলিয়ন গ্যালাক্সি, এবং প্রতিটি গ্যালাক্সিতে প্রায় একশ বিলিয়ন নক্ষত্র। (অসীম কথাটা বিভ্রান্তিজনক।)

প্রশ্ন: সোনিয়া, শাহজাহানপুর বইমেলায় ভীষণ ভীরের মাঝে দূর থেকে আপনার অটোগ্রাফ দেওয়ার দৃশ্যটি দেখার অপেক্ষায় আছি। আপনি কি জানেন এই দৃশ্যটি আমার জীবনে দেখা অন্যতম সুন্দর দৃশ্য??
উত্তর: অটোগ্রাফ দেওয়ার দৃশ্যটি সুন্দর হতেও পারে, কিন্তু সেলফি তোলার দৃশ্য? সেটা ভয়াবহ। এটাই তো বেশি হয় আজকাল!

প্রশ্ন: স্যার, আপনি যে অল্প কিছু উপন্যাস লিখেছেন, সেগুলো আমার বেশ প্রিয়। বিশেষ করে ‘বিবর্ণ তুষার’।আমি এখনো শীতের রাতে লেপের নিচে চা খেতে খেতে (নাকি পান করতে করতে!) এই উপন্যাসটি পড়ি, বেশ লাগে। এধরণের উপন্যাস কি আরো কিছু লেখা যায় না? প্লিজ? প্লিজ? প্লিজ? নামঃ রিফাত ঠিকানাঃ ১/৬৩ গুহলক্ষীপুর, ফরিদপুর – ৭৮০০।
উত্তর: আমার ইচ্ছা আছে, কিন্তু লেখার মত সময় পাচ্ছি না। দেখা যাক।

প্রশ্ন: 0! = 1 হয় কেন ? মোঃ ফেরদৌস সিদ্দিকী ৭/৮,স্যার সৈয়দ রোড,মোহাম্মদপুর,ঢাকা
উত্তর: n! = n(n-1)! যদি 0! = 1 না হয় তাহলে n! define করতে পারবে না। তখন n! হবে n(n-1)(n-2)…3.2.1.0. (-1)(-2)….. যার মান সবসময় 0.

প্রশ্ন: নমস্কার, জাফর দাদু। পোষ সংক্রান্তির শুভেচ্ছা। আমি আদ্রিতা, সুনামগঞ্জ। ‘আমার ছেলেবেলা ‘ বইয়ে হুমায়ুন আহমেদ লিখেছেন আপনাদের বাবা সবার নাম বদলে দিয়েছেন। আপনার আগের নাম কি ছিল?
উত্তর: আমার বাবার ডাইরীতে আমার একটা ডাকনাম দেখেছি। সেটা বলব? তাহলে সবাই আমাকে সেই নামে ডাকাডাকি শুরু করে দেবে না তো?

প্রশ্ন: নিউটনের জনক কে,নাম পারভেজ ,ঠিকানা নাটোর
উত্তর: গুগলে একটা সার্চ দাও। গুগল তৈরীই হয়েছে এরকম কাজের জন্য!

প্রশ্ন: আশা, ঢাকা আসসালামুয়ালাইকুম স্যার টুকুনজিল আমার পড়া প্রথম বই, তাও ক্লাস ৯ এ থাকতে পরেছিলাম, আর এখন আমি এইচ.এস.সির পরীক্ষার্থী । একটা সময় আমি ভাবতাম মানুষ বই পড়ে কি মজা পায়! আর স্যার বিশ্বাস করবেন না,সেই মেয়েটাই এখন বই পড়ার জন্য পাগল হয়ে যায়।আর আমার এই পরিবর্তন এর জন্য ধন্যবাদ টা আপনারই প্রাপ্য, আপনার সুন্দর ম্যাজিক এর মত লিখা আমাকে বশীভূত করেছে। লাইব্রেরি তে আপনার কোন বই দেখে আমি পাগল হয়ে যাই, আমার হার্ট বিট বেরে যায়, নেয়ার জন্য। স্যার আপনি জানেন আমরা বন্ধুরা ঠিক করেছি আমরা যখন বড় হব হুট করে আপনার বাড়ি তে চলে যাব, আপনাকে অবাক করে দিতে! আমি এখন আফসোস করি কেন শৈশব এ আপনার বই পড়লাম না, আমার মনের সকল মন খারাপের সময় চলে যায়। তৈরি হয় আনন্দের সময়, আপনার বই পড়লে। মনটা আমার যে এত ভাল লাগে !! সৃষ্টিকর্তা কে ধন্যবাদ আপনার মত একজন লেখক আমাদের দেশে রাখার জন্য। আপনাকে অনেক ধন্যবাদ স্যার, আপনি আপনার লিখা চালিয়ে যান, নাহলে আমাদের সময় কাটানোর মত কিছুই থকবেনা, এই যান্ত্রিক জীবন থেকে নিজের জন্য আলাদা জগৎ পাওয়া যায় আপনার বই পড়ার মাধ্যমে। নিজের যত্ন নিবেন স্যার। ইনশাআল্লাহ বই মেলাতে আপনাকে ২য় বার দেখার সুযোগ পাব।
উত্তর: থ্যাঙ্ক ইউ! থ্যাঙ্ক ইউ! বই মেলা তো শেষ, তোমার সাথে কি বই মেলায় দেখা হয়েছিল? এই বছর আমি বই মেলায় বেশিদিন যেতে পারিনি।

প্রশ্ন: স্যার আজকাল সবাই ডাক্তার ইন্জিনিয়ার হতে চায়|আমার মাথায় একটু অন্যরকম চিন্তা কাজ করে আমি আর্মি অফিসার হতে চাই অথবা মিসাইল তৈরী করতে চাই|যা আমার দেশকে রক্ষা করবে|আমি যেহেতু অমুসলিম আরো ক্যাডেট কলেজের নই তাই আর্মি অফিসার হতে পারার সম্ভাবনা খুবই ক্ষীন|তাই মিসাইল বা এরকম অস্ত্র নিয়ে কাজ করতে হলে আমার কি বিষয়ে পড়াশোনা করতে হবে আর আমাদের দেশে সেটা সম্ভব কিনা?আর পড়াশোনার পর কাজ করতে হলে অনুমতি পাব কোথায়?প্লিজ স্যার জানা থাকলে উত্তর দিবেন|প্রর্থনা করি আপনি সুস্থ থাকুন| সৌমিক,খুলনা
উত্তর: বাংলাদেশে আর্মিতে যেতে হলে মুসলমান হতেই হবে কে বলেছে? মিসাইল নিয়ে গবেষণা করতে হলে অনেক বিষয়েরই দরকার হওয়ার কথা। এরোনোটিক্যাল ইঞ্জিনিয়ারিং্যের কথা প্রথমে মনে আসে। গুগলে একটা সার্চ দাও আরো ভালো করে জানতে পারবে।

প্রশ্ন: স্যার,, আপনি বইমেলায় কবে যাবেন? (মেহরিন তাসনিম, সাভার,ঢাকা?
উত্তর: আজকেই গিয়েছিলাম। আরো অনেকবার যাব, তোমার সাথে নিশ্চয়ই দেখা হবে।

প্রশ্ন: ইফতেখার, নিউ ইয়র্ক থেকে।
পৃথিবীতে খুব বেশী মানুষকে আমি মন থেকে সম্মান করি না, তবে মানুষ হিসেবে শ্রদ্ধা করি সবাইকেই। আমার মা-বাবার পরে, হাতে গোনা যে ক’জন মানুষকে মন থেকে সম্মান করি আপনি তাদের একজন। বহুবছর আগে একবার বই মেলায় আপনাকে সামনে থেকে দেখার সৌভাগ্য হয়েছিলো, কিন্তু কথা বলা বা আপনার সাথে ছবি তোলার মতো সুযোগ হয় নি মানুষের ভীড়ের কারনে।
হুমায়ুন আহমেদ স্যার অসুস্থ হয়ে নিউ ইয়র্ক আসার পরও বেশ চেষ্টা করেছিলাম স্যারকে হাসপাতালে একবার দেখতে যাওয়ার। সফল হতে পারিনি। ভীষণ খারাপ লেগেছিলো স্যারের চলে যাওয়ায়। স্যার, আগামীতে আপনি কি কখনো নিউ ইয়র্ক বেড়াতে বা অন্য কোন কাজে আসবেন বা আসার সম্ভাবনা আছে? আপনার পরিবারের সবার সুস্বাস্থ্য কামনা করছি। ধন্যবাদ।
উত্তর: থ্যাঙ্ক ইউ! আসলে শীতটা কমলে সত্যি আমার নিউইয়র্কে যাওয়ার একটা পরিকল্পনা আছে। কিন্তু ফ্লাইটগুলো এতো লম্বা যে প্লেনে ওঠার ইচ্ছা করে না!

প্রশ্ন: স্যার,যদি, বিগ ব্যাং এর মাধ্যমে মহাবিশ্ব সৃষ্টি হয় তাহলে, বিগ ব্যাং এর আগে কি ছিল? আর ঐ বিশাল আকার গোলকটাই বা কিভাবে সৃষ্টি হয়েছিল? শেষ প্রশ্ন এই সাইটে কি ছদ্মনাম ব্যবহার করা যাবে? মো রাইয়ান আল রোহান ৬ষ্ঠ(নতুন ক্লাসে উঠেছি)
চাটমোহ, পাবনা (বাবা ট্রান্সফার হয়েছে, কালকেই চাটমোহরে যাব।)
উত্তর: আসলে বিজ্ঞান সে প্রশ্নের উত্তর এখনো দিতে পারে না। তুমি ধরে নাও সময়টাই বিগ ব্যাং এর মুহুর্ত থেকে শুরু হয়েছে, তা হলে আর এই প্রশ্নটার উত্তর দিতে হবে না। উত্তর মেরুর উত্তরে কী আছে সেই প্রশ্নের উত্তর যেরকম কাউকে দিতে হয়না, অনেকটা সেরকম!

প্রশ্ন: মুমিন,চট্টগ্রাম আপনি তো কাগজ-কলমে লিখেন বোধহয়।তারপর কম্পিউটারেও কি টাইপ নিজেই করেন?
উত্তর: হ্যাঁ নিজেই করি। কে আমাকে টাইপ করে দেবে?

প্রশ্ন: আমরা জানি, অক্সিজেন ছাড়া আগুন জ্বলতে পারে না। তাহলে সূর্যের কাছাকাছি কোনো জিনিস গেলে তাতে আগুন লাগে কেন এবং কীভাবে? হাসান রুহানি লাবিব, শ্রেণী ৯ম, শেরপুর সরকারি ভিক্টোরিয়া একাডেমি, শেরপুর।
উত্তর: ‘আগুন লাগে’ না বলে, বলা উচিত প্রচন্ড তাপে ভষ্মীভূত হয়ে যায় কিংবা ধ্বংস হয়ে যায়।

প্রশ্ন: ২০১৯ এ কোন বই পড়ে আপনার সবচেয়ে বেশি ভালো লেগেছে? জেসমিন, ঢাকা।
উত্তর: বই পড়ার সময়টা পেলাম কোথায়? যেগুলো পড়েছি তার মাঝে ইন্টারেস্টিং ছিল হারারীর লেখা, 21 lessons for the 21st century.

প্রশ্ন: Sir ,miss you….. where are you??? Saiba,Dhaka.
উত্তর: এইতো এইখানে! একেবারে তোমার সামনে!

প্রশ্ন: জন্মদিন কেমন কাটলো?কি কি করলেন? আর নতুন বই এর খবর কি? কি কি বই বের করবেন নাম গুলো আগে থেকেই বলবেন যাতে কিনতে পারি। শ্রাবণ। হাজিপাড়া।
উত্তর: জন্মদিন? এই বয়সে কী আবার কেউ জন্মদিনের কথা মনে রাখে? ামি উত্তর দিতে এত দেরী করেছি যে আমার বইগুলো বের হয়ে গেছে! সেগুলোর নাম হচ্ছে: প্রজেক্ট আকাশলীন (সায়েন্স ফিকশান), গ্লিনা (সায়েন্স ফিকশান), আমার সাইন্টিস মামা (কিশোর উপন্যাস), বাংলাদেশের মেয়ে বাংলাদেশের নারী ও অন্যান্য (কলাম ২০১৯), যেরকম টুনটুনি সেরকম ছোটাচ্চু (বের হবে হবে করছে), রহস্যময় ব্ল্যাক হোল (বিজ্ঞান), টুনটুনি ও ছোটাচ্চু সমগ্র (সংকলন), মিতু তিতুর সাবমেরিন (শিশুতোষ), ছোট্ট একটা নেংটি ইঁদুর (শিশুতোষ, বের হবে হবে করছে)!

প্রশ্ন: Sir, Happy New Year Swapneel, Meherpur
উত্তর: তোমাকেও হ্যাপি নিউ ইয়ার। (একটু বেশি পুরানো হয়ে গেল আর কী!)

প্রশ্ন: Happy New Year 2020 Sir.আপনার নতুন বছর ভাল কাটুক। স্যার আমি class 6 থেকে class 7 এ উঠেছি।আমি class 6 এর ফাইনাল এক্সাম এ চতুর্থ হয়েছি।আপনি please আমার জন্য দোয়া করবেন যাতে আমি class 7 এ আরও ভাল করতে পারি।নামঃতরী।ঠিকানাঃদিনাজপুর।
উত্তর: দোয়া করছি যেন ক্লাস সেভেনে আরো ভালো করতে পার। (আমি যখন আরো ভালো বলি তখন কিন্তু পরীক্ষায় ভালোর কথা বলি না, অ্ন্য সব কিছু ভালো এর কথা বলি!)

প্রশ্ন: আপনার বইগুলো শুধু বইমেলায় প্রকাশ না করে বছরের অন্যান্য সময় প্রকাশ করলে আমার মত পাঠকদের বেশ সুবিধে হত । এক মেলায় আপনার ৭-৮ টা বই কেনা সাধারণ পাঠকদের জন্য একটু কষ্টকর । ঈমাদ, মিরপুর
উত্তর: তুমি যদি একজনকে সাবকন্ট্রাক্ট দিতে পার যে আমার বইগুলো লিখে দিবে, তাহলে সারা বছর ধরে আমি বই প্রকাশ করে যেতে পারি! (আসলে বই মেলার চাপের কারনে এই সময় কয়েকটা বই লেখা হয়। তোমাকে সব বই বইমেলায় কিনতে হবে কে বলেছে? সারা বছর ধরে কিনো। আসলে বই পড়তে হলে তো বই কিনতে হয় না। কেউ যদি থিক করে সে শুধু বই কিনে পড়বে তাহলে তো বেশি বই পড়তে পারবে না!)

প্রশ্ন: দীপাঞ্জনা, মুক্তাগাছা,ময়মনসিংহ। স্যার আমি একজন শিক্ষিকা। বতর্মান শিশুদের মাঝে বই পড়ার আগ্রহ কমে যাচ্ছে, যা সামাজিক উন্নয়নের অন্তরায়। শিশুদের বই পড়ার আগ্রহী করে তোলার জন্য আমাদের শিক্ষকদের করণীয় কি?
উত্তর: আমি খুব খুশি হয়েছি যে আপনি এই বিষয়টা নিয়ে ভাবছেন। আমি নিশ্চিত চিন্তা করে করে আপনিই একটা উপায় বের করে ফেলতে পারবেন। শিক্ষক হিসেবে আপনি তাদের অনুপ্রাণীত করতে পারেন যেটা হয়তো অন্যরা পারবে না। বাচ্চারা যে বই পছন্দ করে তাদেরকে সেরকম বই দিয়ে পড়ার অভ্যাস গড়ে তুলতে পারলে ভালো। যে বই পড়তে তাদের ভাল লাগে না শুরুতে সেরকম বই জোর করে পড়ানোর চেষ্টা করলে খুব লাভ হবে না। বিশ্ব সাহিত্য কেন্দ্রের চমৎকার প্রোগ্রাম আছে, তাদের সাথে যোগাযোগ করতে পারেন।

প্রশ্ন: Sir, I Got A+ In The J.S.C Exam. Plz Bless Me.Swapneel, Meherpur
উত্তর: কী চমৎকার! তুমি কী জান, আমি তোমার মতই A+? (তবে আমি অবশ্যি লেখা পড়া করে A+ হতে পেরিনি, আমার রক্তের গ্রুপ দিয়ে হয়েছি!)

প্রশ্ন: Assalamualaikum, sir. Amar nam Sania, Dhakar Green Road e thaki. Ami apnar lekhar onneek boro FAN! Amar bookshelf bhorti khali apnar boi. To jai hok, ekhon question ta kori. Amar Physics porte onek bhalo lage. Physics er moto easy ar mojar subject ar nei (amar mote)! Kintu Chemistry ektu bhoy lage. Ami shunechi science er sob shakha ektar sathe ekta related. Ekhon, ami Chemistry er moddhe Physics er moja kibhabe ber korbo? Chemistry er bhoy kibhabe dur korbo? Please give some advice.
উত্তর: তুমি কিসে পড় জানলে আমার জন্য উত্তর দেওয়া সহজ হত। একটা বিষয় ঠিক করে না বুঝলে একটু ভয় লাগতে পারে। তা ছাড়া কোনো সাব্জেক্ট ভালো লাগে আবার কোনো সাব্জেক্ট তত ভালো লাগে না, এরকম তো হতেই পারে। যদি একটা বই পড়ে ঠিক বোঝা না যায় তাহলে আরো কয়েকটা বই পড়ে দেখতে পার।

প্রশ্ন: আবিদা সুলতানা। চন্দনটুলা,,অাম্বরখানা, দরগা গেট, সিলেট। sir, আপনি কি আমার যেকনো প্রশ্নটা উত্তর দেবেন কী?
উত্তর: হ্যাঁ দিব! (এই যে দিলাম! তুমি প্রশ্ন করেছ আমি তোমার প্রশ্নের উত্ত দিব কি না, আমি উত্তর দিয়েছি হ্যাঁ দিব!)

প্রশ্ন: মানুষের চুল পাকে কেন? আর পেকে শুধু সাদা রং ই হয় কেন? নাম:জান্নাতুল আসমান সাগর; বটতলা, শেরপুর
উত্তর: why does hair turn grey লিখে ইন্টারনেটের গুগলে একটা সার্চ দাও, দেখবে খুব ভালো করে বুঝিয়ে দেওয়া আছে। (আমি আসলে তোমাদের সবাইকে ইন্টারনেট ব্যবহার করে দুনিয়ার সব প্রশ্নের উত্তর বের করে ফেলা শিখাতে চাই।)

প্রশ্ন: আসসালামুয়ালাইকুম স্যার। আমার নাম ইমরান।আমি আপনার সাইন্স ফিকশনের বইগুলো খুব বেশি পছন্দ করি
উত্তর: থ্যাঙ্ক ইউ ইমরান। যতদিন তোমরা আমার বই পড়বে ততদিন আমি লিখব!

প্রশ্ন: আমার একটা প্রশ্ন আছে তা হলো,মানুষ কি অন্যের mind read করতে পারে বা অন্যের মস্তিষ্কে কি চলছে তা বুঝতে পারে
জালালাবা, বায়েজি, চট্টগ্রাম
উত্তর: পারার কথা না! মুখ দেখে হয়তো মনের অবস্থা একটু অনুমান করা যায় কিন্তু তার বেশি কিছু কেমন করে করবে? বড় জোর এটা নিয়ে সায়েন্স ফিকশান লেখা যেতে পারে।

প্রশ্ন: দাদুভাই, আমি বড়ো হয়ে মহাকাশ গবেষক হতে চাই। এ বিষয়ে আপনার পরামর্শ চাই। প্লিজ। রুম্মান জারা, চাঁদপুর।
উত্তর: কোনো সমস্যা নেই। গণিত আর বিজ্ঞান বেশি বেশি করে শিখ এবং প্রবলেম সমাধানে জোর দাও। তাহলেই যখন সময় হবে তখন তুমি প্রস্তুত থাকবে।

প্রশ্ন: আপনার অনেকগুলো বাংলা ও ইংরেজী বই এর নাম জানতে চাই । আনিশা ইবনাত রংপুর সপ্তম শ্রেণি।
উত্তর: এই ওয়েবসাইটেই আমার সব বইয়ের তালিকা আছে। খুঁজে বের করে নাও প্লিজ! আমি বলতে গেলে ভুল করে ফেলব।

প্রশ্ন: স্যার,কিছুদিন ধরে শুনছি psc এবং jsc পরীক্ষা বাদ দেওয়া হবে। এ বিষয়েে আপনার মতামত কী? যদি আমার ক্লাস ৮ এ ওঠার আগেই jsc পরিক্ষা বাদ দিলে আমি খুব খুশি হবো। আবির হাসান আরাফাত,নোয়াখালী। ক্লাস ৬
উত্তর: পরীক্ষা নিয়ে মাথা ঘামিও না। শিখে যাও। (লক্ষ করেছ, আমি কিন্তু বলি নাই ‘লেখাপড়া করে যাও’, বলেছি ‘শিখে যাও’।) তুমি যেহেতু মাত্র ক্লাস সিক্সে পড় অনেক বেশি বই পড়বে, দুনিয়ার যত বই পাও সব পড়ে ফেলবে।)

প্রশ্ন: শীতে গরমের চেয়ে একটু ব্যাথাই বেশী লাগে কেন? নাম:জান্নাতুল নাঈম স্মরণ; বটতলা ,শেরপুর
উত্তর: লাগে নাকি? আমি তো লক্ষ করিনি। তুমি ইন্টারনেটে একটা সার্চ দিয়ে বের করে ফেলে কারণটা আমাকে জানাও। ঠিক আছে?

প্রশ্ন: আংকেল, এবার জন্মদিনে কী কী করলেন? (জন্মদিনের অনেক শুভেচ্ছা এবং ভালোবাসা) ঢাকা থেকে ইশতিয়াক
উত্তর: এতোদিন আগে জন্মদিন পার হয়েছে যে আমি এখন ভুলেই গেছি। মনে হয় ভাইবোনেরা এসেছে, তাদের সাথে একটু গল্প গুজব করেছি!

প্রশ্ন: Name:zarin Cumilla, Sir no matter whatever people say about u, u will always be my hero pls stay strong for us. we love u.dont pay heed to ungrateful peoples…i would grow up as a jerk if i have naver read yous story…these have enlighten my life
উত্তর: আমাকে নিয়ে কোনো দুশ্চিন্তা করো না। কে আমাকে নিয়ে কি বলছে আমি সেগুলো নিয়ে মাথা ঘামাই না। যে আমাকে এত ঘৃণা করে যে আমাকে মেরে ফেলতে চেয়েছিল, তার উপরেও আমার কোনো রাগ নেই! আমি জানি তোমাদের মত অনেক ছোট ছোট ছেলে মেয়ে আছে যারা আমাকে অনেক ভালোবাসে। আমি তাহলে কেন ওই মানুষগুলো নিয়ে দুশ্চিন্তা করব? (আরেকটু হলে তুমি jerk হয়ে বড় হতে পড়ে খুব মজা পেলাম। হা হা হা!)

প্রশ্ন: শুভজন্মদিন প্রিয় লেখকওব্যক্তি। আপনার দীর্ঘায়ু কামনাকরছি। টুটুল, ফেনী। স্যারআপনার প্রিয় গায়ক কে?
উত্তর: থ্যাঙ্ক ইউ। আমার প্রিয় গায়ক? আমি তো সবার গান শুনি। তালিকা লিখলে সেটা এতো বড় হয়ে যাবে যে এখানে আটবেই না।

প্রশ্ন: কিশোর উপন্যাস সমগ্র 6 বের হবে কবে নাগাদ! Mehjabeen Holy Cross College
উত্তর: ছয়টা কিশোর উপন্যাস জমা হয়ে গেলে সেগুলো নিয়ে একটা সমগ্র বের হয়। এই সাইটে আমার লেখা সব বইয়ের তালিকা আছে, তুমি সেটা দেখে নিজেই বের করে ফেলতে পারবে পরের সমগ্রটা বের হতে আর কত দেরী। আমি বলতে গেলে ভুল করে ফেলতে পারি।

প্রশ্ন: জানেন, আপনার বইগুলো পড়তে পড়তে একসময় অবাক হয়ে ভাবতাম- ধুর! এইসব চরিত্রের মতো এত উদার, এত ইতিবাচক, এত সাহসী, ইতিহাস-সচেতন, শিশুর মতো সরল কোনো কিশোর বয়সী ছেলে বা মেয়ে কি বাস্তবে আছে? এত পরিপক্কতা কি এমন অল্প বয়সে সত্যিই থাকা সম্ভব? যেহেতু নিজের ক্ষুদ্র জীবনে কাউকে কোনোদিন দেখিনি ডোরা, খোকন, রাশেদ কিংবা টুনির বয়সে ওদের ধারে-কাছেও যেতে ; সুতরাং মাথা নিচু করে মেনে নিয়েছিলাম বাস্তবতার অত্যাচার, কল্পনার জগৎ থেকে সাময়িক ছুটি নিয়ে হতাশ হয়ে ফিরে গিয়েছিলাম দৈনন্দিন জীবনে।
কিন্তু আপনার, আমার এবং লাখো শিশু-কিশোরের স্বপ্নের জগৎ সত্যি হয়েছিল এক শুভদিনে, যখন পরিচয় হয়েছিল জান্নাত আপুর সাথে। ……
সবশেষে, জান্নাত আপু ছাড়াও আরেকজন মানুষ আছে, যে আপনার কাল্পনিক জগতের প্রতিটি কাল্পনিক চরিত্রের প্রতিনিধিত্ব করে বাস্তবেই। তিনি আর কেউ নন, আপনি নিজেই। কয়েক প্রজন্মের পাঠককে কয়েক দশক ধরে অনেক দিয়েছেন আপনি, আজও দিচ্ছেন। এখন আর সত্যিই কিছু চাওয়ার নেই আপনার কাছে। কিছু দেবো-ই বা কী? একরাশ ভালোবাসা ছাড়া?
অরিত্র আহমেদ, নবম শ্রেণি, ঢাকা।
উত্তর: তোমাদের এত প্রিয় একজন মানুষ এভাবে তোমাদের ছেড়ে চলে গেছে, আমি বুঝতে পারছি তোমাদের কত খারাপ লাগছে। মানুষগুলো থাকে না, শুধু স্মৃতিটা রয়ে যায়। আমার সবসময়েই মনে হয়েছে মানুষের যেটুকু প্রয়োজন তার থেকে বেশি মায়া দিয়ে দেওয়া হয়েছে। সেজন্য আমরা কষ্ট পাই বেশি।

প্রশ্ন: হাসিন, ঢাকা lশুভ জন্মদিন, স্যার l
উত্তর: থ্যাঙ্ক ইউ, হাসিন। (একটু দেরী করে ফেললাম, তাই না?)

প্রশ্ন: জানেন স্যার আমাকে কেউ বাড়ির বাইরে কোনো জায়গাতে যেতে দেয়না । আমার এভাবে থাকতে খুব কষ্ট হয় ।আমি বাড়ির বাইরে গেলে বাবা আমার পড়ালেখা বন্দ করে দেবে বলে । আমার দম আটকে আসে।এটা লিখছি আপুর ল্যাপটপ দিয়ে আমদের বাড়ি আসেছে।বলুন না আমি কি করব? ঘুমের মেডিসিন খাব ?
উত্তর: না না না, ঘুমের মেডিসিন খবে কেন? সহ্য করে যাও, দেখতে দেখতে তুমি বড় হয়ে যাবে, তখন তুমি নিজের ইচ্ছা মত সব কিছু করতে পারবে। যদি আর কিছু করত না পার, বই পড়। এবং যদি পার লেখার চেষ্টা কর। তুমি মনে করো না আমি শুধু শান্তনা দেওয়ার জন্য এটা বলছি, তোমার চাইতেও ভয়ংকর অবস্থায় ছিল এরকম অনেকেই কিন্তু শেষে একেবারে স্বাভাবিক সুন্দর জীবন পেয়েছে।
প্রশ্ন: Sir,23 December apnar birthday…
“” শুভ জন্মদিন, স্যার”” মেহরিন তাসনিম
উত্তর: থ্যাঙ্ক ইউ মেহরিন। (ফেব্রুয়ারি মাসে ডিসেম্বরের শুভেচ্ছে নিতে খুবই লজ্জা লাগছে!)

প্রশ্ন: শীতে এত ঠান্ডার মধ্যেও ভূগর্ভস্থ পানি গরম থাকে কিভাবে? মোঃ জান্নাতুল আদন শ্রেণীঃ ৮ম রংপুর জিলা স্কুল, রংপুর |
উত্তর: উপরের ঠাণ্ডা কিংবা গরম মাটির নিচে যেতে পারে না তাই এটাকে শুধু শীতের সময় গরম মনে হয় তা নয় গরমের সময় ঠাণ্ডা মনে হয়।

প্রশ্ন: স্যার, বাসাটা উল্টা কেন?আমি বুঝতে পারছি না। সত্যি কি এমন? অবন্তি, ময়মনসিংহ।
উত্তর: ছবিটা উলটো করে দেখ। বাসাটাকে এভাবে সাজানো হয়েছে!

প্রশ্ন: স্যার, নতুন কিছু ছবি জুড়ে দিয়েন। সাইরা, ঢাকা
উত্তর: দিলাম। পুরানো ছবি ঘাটতে ঘাটতে এটা পেয়েছি। আমি যখন পর্বতারোহী ছিলাম তখনকার ছবি। দেখি, আমাকে বের করতে পার কীনা!

প্রশ্ন: স্যার, আপনি Linkin Park এর গান শোনেন? অমিত চৌধুরী, সাভার
উত্তর: না, শুনি না। রবীন্দ্র সংগীত ছাড়া আর কোনো গান আসলে শোনা হয় না!

প্রশ্ন: আপনার ইমেইল এড্রেসটা কী দিবেন ?সবাই কেমন করে জানি পেয়ে যায়,আমিই খুঁজে পাই না। স্বপ্ন সেন্ট্রাল রোড, রংপুর
উত্তর: পাবে, তুমিও পাবে। আরো একটু খুঁজো!!

প্রশ্ন: Sir. আমার বয়স 7 বছর 7 মাস ۔এর মধ্যে আমি আপনার সব কিশোর উপন্যাস সমগ্র পড়েছি এবং ২৫ তার মতন সাইন্স ফিকশন ও পড়শি ۔আমি কি আপনার একটা অটোগ্রাফ পেতে পারি !
এই টাইপ টা আমার বোন করলো মাহতাব আব্দুল্লাহ জামি আমার বাবা পিএইচডি করে বাচ্চাদের 8 এর আগে স্কুল নয় আমি এখনো স্কুল যাই না
উত্তর: তুমি সাত বছর বয়সে স্কুলে যাবার আগেই এতোগুলো বই পড়ে ফেলেছ? কী আশ্চর্য!! তোমাকে একটা কেন, দশটা অটোগ্রাফ দিব!

প্রশ্ন: ১.স্যার, আমরা জানি যে স্বপ্ন পূরণে মানুষকে অনেক পরিশ্রম করতে হয়। আমারও অনেক স্বপ্ন আছে।সত্যি বলতে এত এত স্বপ্ন আছে যে স্বপ্ন দেখার ঠেলায় আমি আর কোন কাজই করতে পারি না! সারাদিন বসে বসে শুধু স্বপ্নই দেখি!স্যার আপনার কি কখনো এরকম হয়েছে? ২.শহীদ জননী জাহানারা ইমামের কথা মনে হলে আমার খুব কষ্ট হয়।তিনি যে কষ্টগুলো পেয়েছেন ঠিক সেরকম কষ্টই একাত্তরে এদেশের সব মানুষ পেয়েছিল। এত কষ্ট এত দুঃখ!স্যার আমি এখন প্রায়ই ‘green green grass’ গানটা শুনি।আর যখন শুনি তখন যেন চোখের সামনে একাত্তরের সেই দুঃসহ দিনগুলি একে একে জড়ো হতে থাকে।স্যার, যদি জাহানারা ইমামকে বলতেন যে আপনি তাঁর প্রিয় রুমীর কলেজ সহপাঠী তিনি হয়তো খুশি হতেন।গতকাল গিয়েছে ১৬ডিসেম্বর, ২০১৯ ।সামনে বিজয় দিবসের সুবর্ণজয়ন্তী।আমি আল্লাহর কাছে প্রার্থনা করব যেন সব মানুষের কষ্টগুলো একটু হলেও কমে ।-রিশা, ঢাকা।
উত্তর: স্বপ্ন দেখা ভালো, শুধু স্বপ্ন দেখেই একটা জীবন আনন্দে কাটিয়ে দোয়া যায়। তবে একটা কথা মনে করিয়ে দিই, স্বপ্ন যে সব সময় সত্যি হয় তা কিন্তু নয়। স্বপ্ন সত্যি হওয়া কিন্তু জীবনের উদ্দেশ্য নয় স্বপ্নের জন্য কাজ করা হচ্ছে জীবনের উদ্দেশ্য! শহীদ জননী জাহানারা ইমাম অনেক কষ্ট পেয়েছেন সত্যি কিন্তু তিনি তার কষ্টটাকে বুকে চেপে আমাদের দেশের তরুণদের একটা স্বপ্ন দেখিয়েছিলেন, সেটা হচ্ছে এই দেশে যুদ্ধাপরাধীদের বিচার করতে হবে। তাই যখন শাহবাগে গণজাগরণ মঞ্চ হয়েছিল তখন সেখানে তার বিশাল একটা ছবি টানিয়ে রাখা হয়েছিল।

প্রশ্ন: স্যার, আমি ক্লাস ৯ এ পরি আমার কোন বই পড়া উচিৎ? শামস,ঢাকা।
উত্তর: যে কোনো বই। তুমি এখন বড় হয়ে গেছ, কাজেই গল্প উপন্যাস প্রবন্ধ যা ইচ্ছা তাই পড়তে পার। ইংরেজি কিংবা বাংলা যা ইচ্ছা।

প্রশ্ন: প্রত্যয়, দশম শ্রেণি,সিলেট(স্যার ঠিকানাটা প্লিজ গোপন রাখবেন) আপনার টেলিফোন নাম্বার টা আমার কাছে আছে( আপনার এক ছাত্র, সে সাস্টে ,সিএসই ডিপার্টমেন্টএ পড়ত,আমাকে দিয়েছিল নাম্বারটা। আপনার নাম্বার ফোন করলে কি আপনার সাথে কথা বলতে পারবো?
উত্তর: মনে হয় পারবে না। কারণ আমি আসলে ফোন ধরি না, বড় জোর দরকার হলে ফোন করি। সরি।

প্রশ্ন: স্যার, শহীদ জননী জাহানারা ইমামের সাথে কি আপনার ব্যক্তিগত পরিচয় ছিল? এরকম চমত্‍কার একজন মানুষ আমি বেশি দেখিনি। কী চমত্‍কার তার চিন্তাশক্তি, কী বিশাল তাঁর সহ্যক্ষমতা, কত দয়ার্দ্র তাঁর মন, কী SOPHISTICATED একজন মানুষ! এই মানুষটার কথা মনে হলে এত কষ্ট লাগে!কী বলব। বিশেষ করে জুড়ুর কথা। মনে হয় ছোট্ট জুড়ু যেন ছোট্ট সেই দুর্গার মত! এত মায়াময়! জাহানারা ইমামের যাদের সাথে পরিচয় ছিল তাদের জানার চেষ্টা করছি। স্যার, ছিল কি আপনার পরিচয় উনার সাথে? পুনশ্চঃআপনার শরীর কেমন আছে? মাথায় ব্যথা যন্ত্রনা কিছু হয়? অনেক অনেক ভালবাসা রিশা, ঢাকা।
উত্তর: হ্যাঁ ছিল। জাহানারা ইমাম আমাকে খুব স্নেহ করতেন। আমি যখন আমেরিকা ছিলাম তখন একবার আমার বাসাতেও ছিলেন। লম্বা লম্বা ড্রাইভে গাড়ীতে বসে তার সাথে রাজ্যের গল্প করতাম। আমি কখনো আমার কোনো বইয়ের পিছনে কাউকে দিয়ে কোনো কিছু লিখাই না, শধু প্রথম দিকের একটা বইয়ে এরকম লেখা আছে কারণ সেটা লিখেছিলেন শহীদ জননী জাহানারা ইমাম! আর কী আশ্চর্য জান, বহুদিন পরে তাঁর একটা বইয়ের মুখবন্ধে আমি তাঁর বইটা নিয়ে লিখেছি। আমার যে কী অবাক লেগেছে তখন। একবার তাঁর খুব শরীর খারাপ হল তখন আমি তাকে একটা প্যাকেটে ভরে কিছু গিফট পাঠালাম তার মন ভালো করার জন্য। গিফট পেয়ে খুশি হয়ে তিনি আমাকে একটা চিঠি পাঠালেন, কি যে সুইট সেই চিঠিটা আমি এখনো যত্ন করে সেটা রেখে দিয়েছি। (আমার শরীর ভালো আছে। না, আমার মাথায় কোনো যন্ত্রণা নেই!)

প্রশ্ন: রায়হান মাসুদ, যশোর। এবার hsc পরীক্ষা দেব। তিন মাসে তিন বছরের পড়া যেভাবে পড়েছেন, তিন মাস যেন ওই ভাবে পড়তে পারি দোয়া করেন।
উত্তর: দোয়া করলাম। কিন্তু তোমাকে কেন তিন বছরের পড়া পড়তে হচ্ছে? দেখো, আমার মত দরজা জানালা বন্ধ করে পড়লে গায়ের রঙ ইটের নিচে চাপা পড়ে থাকা ঘাসের মত ফর্সা হয়ে যাবে!

প্রশ্ন: আংকেল, প্রতিবার যখন উত্তর দিবেন তখন আপনার পছন্দের কতগুলি বইয়ের নামও দিয়ে দিবেন প্লিজ। এতে করে আমরা বইয়ের সাজেশনও পাবো আবার আপনার পছন্দের বই সম্পর্কে জানতেও পারবো! বুদ্ধিটা কেমন? ইশতিয়াক, ঢাকা
উত্তর: বুদ্ধিটা ভালো, কিন্তু এখন আমার যে বই পছন্দ তোমাদের তো সেই বই পছন্দ হওয়ার কথা না।

প্রশ্ন: আপনি আপনার লেখক জীবনে নিশ্চয়ই অনেকবার অসম্মান , অবজ্ঞা বা করো হিংসার শিকার হয়েছেন । তখন আপনার কি সব কিছু ছাড়তে ইচ্ছা করেনি ? মোস্তফা নুরুল আলম , গোপালগঞ্জ ।
উত্তর: মন খারাপ হয়েছে, কিন্তু সবকিছু ছেড়ে ছুড়ে চলে যাব সেটা কখনোই মনে হয়নি।

প্রশ্ন: স্যার , আপনার কাছে প্রতিদিন গড়ে কয়টি প্রশ্ন আসে এবং কয়টির উত্তর দেন ? মোস্তফা নুরুল আলম , গোপালগঞ্জ ।
উত্তর: সেভাবে তো বলতে পারব না। এই ওয়েবসাইটটার কথা খুব বেশি মানুষ জানে না, সেজন্য আগের মত খুব বেশি চাপ নেই!

প্রশ্ন: স্যার, কতবার ????? সাগর খান ,ঢাকা
উত্তর: যতবার !!!!!

প্রশ্ন: আসসালামুয়ালাইকুম স্যার । টুনটুনি ও ছোটাচ্চুর সব বই আমার খুব প্রিয়। যখন টুনটুনি তখন ছোটাচ্চুর পর কি আর কোনো বই প্রকাশ হয়েছে ? সেই বইটার নাম কী ? কোন প্রকাশনায় বের হবে ? রামিশা বিনতে শামস্ , বনশ্রী , ঢাকা ।
উত্তর: এই সাইটটাতেই আমার সব বইয়ের লিস্ট আছে। সেটা দেখে নাও প্লীজ!

প্রশ্ন: Sir, could you please tell me how can I improve my reading comprehension? When I open an English book and try to read something, I need to check the dictionary all the time! And I quit reading even before I finish reading one page!! That’s how I can’t read a book. I wish I could read English books fluently some day in future. But I think that’s never gonna happen! So please tell me how did you learn to read English books? Mostafizur Rahman Rahman Nagar, Bogura
উত্তর: সাধারন লেখলেখিতে খুব বেশি ইংরেজি শব্দ ব্যবহার হয় না। একবার সেই শব্দগুলো শিখে গেলে দেখবে তোমার কোনো সমস্যা হচ্ছে না। কাজেই আর অল্প কিছুদিন দাঁতে দাঁত কামড়ে ডিকশনারি দেখে দেখে বই পড়।

প্রশ্ন: স‍্যার ছোটাচ্চু আর ফারিয়াপুর বিয়ে দিয়ে দেন , বুলবুল নাটোর
উত্তর: আমি তো দিতেই চাই, ওরা রাজী হচ্ছে না।

প্রশ্ন: আসসালামু আলাইকুম স্যার, স্যার আগে আমার বই(গল্প,উপন্যাস) পড়তে ভালো লাগতো না, নবম শ্রেণী পর্যন্ত একটি বইও আমি সম্পুর্ণ পড়ি নাই, আপনার লেখা একটি বই ”আমি তপু” বইটি পড়ার পর আমি বইয়ের প্রতি এতো আকৃষ্ট হয়েছি যে বই ছাড়া আমার অবসর চিন্তাও করতে পারি না,স্যার আমি আপনার অনেক বড় ফ্যান, স্যার আপনি আমার নামটা লিখে প্লিজ একটা স্বাক্ষর দিন শায়েখ আল হাসান কালিগঞ্জ, গাজিপুর
উত্তর: দিলাম। শুনে খুব খুশি হলাম যে তুমি বই পড়া শিখে গেছ, এখন সারা জীবনের জন্য তুমি নিশ্চিন্ত!

প্রশ্ন: স্যার মনে করেন কোনো বস্তু ১ লক্ষ km/sec. বেগে যাচ্ছে । তাহলে তার সাপেক্ষে আলোর বেগ হবে ৩ লক্ষ km/sec. (আপনি একটি বইয়ে লিখেছিলেন) । এখন ২য় কোনো বস্তুর সাপেক্ষে আলোর বেগ কত হবে ?
মোস্তফা নুরুল আলম,নবম শ্রেণি , গোপালগঞ্জ ।
উত্তর: কোনটার সাপেক্ষে কোনটা কত বেগে যাচ্ছে সেটা আগে জানতে হবে, তারপর হিসেব করে বের করতে হবে। শুধু বলা যায় যে কখনোই কারো সাপেক্ষেই গতিবেগ আলোর বেগ থেকে বেশি হতে পারবে না।

প্রশ্ন: স্যার, হঠাৎ একদিন ঘুম ভেঙ্গে দেখলেন আপনি আপনার কোন একটা সাইন্স ফিকশনের জগতে চলে এসেছেন (যেমন: ইরনের স্পেসশিপে বা ক্রোমিয়াম অরন্যের সেই পৃথিবীতে ) আপনি কি করবেন? “তন্ময় রায়, বরিশাল”
উত্তর: ধমক দিয়ে বলব, আমাকে এখানে এনেছ কেন? এক্ষুনি আমাকে বাসায় দিয়ে এসো।

প্রশ্ন: শ্রদ্ধেয় স্যার,আমি একটি বৈজ্ঞানিক কল্পকাহিনী ভিত্তিক গল্প লিখতে চাই। লেখার আউটলাইনটি কিভাবে তৈরী করবো জানাবেন কি, প্লিজ? শ্রদ্ধা সহকারে, শাইয়্যান মোহাম্মদ ফাছিহ-উল ইসলাম
উত্তর: আমার মনে হয় এর কোনো ধরাবাধা নিয়ম নেই। প্রত্যেকে নিজের মত করে তার আউটলাইন তৈরী করে। তুমি তোমার পদ্ধতিটা বের করে নাও। আমি বললে তো হবে না!

প্রশ্ন: স্যার অল্পকিছু দিন পরেই আমার এসএসসি পরীক্ষা।আমাদের ব্যাচটাই প্রথম ময়মনসিংহ বোর্ডে পরীক্ষা হবে। আমার জন্য দোয়া করবেন স্যার।। পরীক্ষাটা শেষ হলেই আমি হাজির বই মেলায়,এবং আপনার সাথে দেখা করব।। অবন্তি, ময়মনসিংহ।
উত্তর: ঠিক আছে। ভালো করে পরীক্ষা দাও। পরীক্ষার আগের রাতে ভালো করে ঘুমাবে।

প্রশ্ন: হ্যাঁ স্যার আমার ইচ্ছে আপনাকে ইকবাল স্যার বলেই ডাকব। যখন আপনার সাথে দেখা হবে তখন ইকবাল স্যার বলে ডাকলেই আপনি চিনতে পারবেন। বলবেন আরে এতো অবন্তি!! তাই না স্যার??? অবন্তি, দশম শ্রেণি, ময়মনসিংহ।
উত্তর: চেষ্টা করব। কিন্তু আমার স্মৃতি শক্তি বলতে গেলে কিছুই নাই তাই যদি আমি না বলতে পারি তাহলে তুমিই বলে দিও, স্যার, আমি অবন্তি!

প্রশ্ন: Hallow sir, I am borna. I love science very much. i have a question. That is, what is black hole? pleace, give me the answer of this question sir. name:borna adress: chandantula, ambarkhana, dargagaet, sylhet.
উত্তর: এই সাইটেই একটা বই আছে, সেটা ডাউনলোড করে পড়ে দেখ। সেখানে অনেক গুছিয়ে লিখেছি। এখানে লিখলে এক দুই বাক্যে লিখতে হবে।

প্রশ্ন: আসসালামুআলাইকুম স্যার। টুনটুনি ও ছোটাচ্চু নিয়ে আবার কবে বই প্রকাশ করবেন? জারা, চাঁদপু।
উত্তর: এই একটু আগে বইমেলেয় গিয়ে ম্যানাস্ক্রিপ্টটা প্রকাশককে দিয়ে এসেছি। এখন দেখা যাক কতদিন লাগে বইটা বের হতে।

প্রশ্ন: Sir Apnaky Havvy Sundor Lagcy Ki Kory Janlam Bolen Dini.Karon Amar Mony Hoi Amar Spcial Powar Asay Sayta Thaky Bollam.Swapneel,meherpur
উত্তর: তোমার স্পেশাল পাওয়ার দিয়ে আর কী কী করতে পার শুনি? মশা মারতে পারবে? মশার উৎপাতে বাসায় বসে কাজ করতে পারি না।

প্রশ্ন: স্যার,হুমায়ুন স্যারের একটা বই শুভ্র গেছে বনে ওখানে এক অংশে আছে”you can stop the time if you want to stop the time” যে কেউ তো চাইবে সময় আটকাতে আপনি যদি কথাটার আসল অথটা বোঝাতেন তবে আমার মন একটু শান্ত হতো।স্বপনিল (আমি একজন মেয়ে নাম শুনে সবাই আবাক হয়।আমার তো নামটা ভালোই লাগে।নামের পেছনের কারন মার কাছ থেকে শুনেছি অন্য দিন বলব আজ থাক), মেহেরপুর
উত্তর: হুমায়ুন আহমেদ এই কথাটা দিয়ে ঠিক কী বোঝাতে চেয়েছিল সেটা তো আমি জানি না! তা ছাড়া ঠিক কখন কী পরিস্থিতিতে এটা বলেছিল সেটাও তো আমার মনে নেই। সাধারন ভাবে যেটা বোঝায় সেটাই না হয় বুঝে নাও।

প্রশ্ন: স্যার নাট বল্টু গল্পে “টুথপেস্ট কত লম্বা ” এখানে আপনি টুথপেস্টের মাপ প্রাকটিক্যালি মেপে ছিলেন???? বায়েজিদ হোসেন বুলবুল,লালপুর,নাটোর
উত্তর: প্রাকটিক্যালি না মেপেই এটা বের করা সম্ভব। একটা টুথপেস্ট টিউবের ব্যাসার্ধ আর টিউব থেকে বের হওয়া টুথপেস্টের ব্যসার্ধ জানা থাকলে অনুমান করে ফেলা যায়।

প্রশ্ন: শ্রদ্ধেয় স্যার,
আমার সালাম নিবেন।আমি আপনার একজন গুণগ্রাহী-ভক্ত।আপনার সায়েন্স ফিকশন পড়েই আমার বই পড়ার অভিযাত্রার সূত্রপাত। পত্রিকায় আপনার কলাম যেদিন ছাপা হয়, সেদিন বেশ আগ্রহ নিয়ে সেটা পড়ি। আপনাকে আমি সামনাসামনি একবারই দেখেছিলাম, সেটা এ বছরের শুরুর দিকে, গণিত উৎসবের জাতীয় পর্বে। আপনার সাথে সেলফি তোলা এবং আপনার বক্তব্য সামনাসামনি উপভোগ করা অসাধারণ এক অভিজ্ঞতা ছিলো।তো,আপনাকে যখন এত আপন করে নিয়েইছি,(আমার ধারণা, এদেশের কিশোরদের অধিকাংশই!)তখন মনে হলো আমার মনের দুটো কথা আপনাকে জানাই।
………
………
এদেশে বিবর্তন নিয়ে গবেষণা হবে না? কাজ হবে ন।। আমাদের দেশ গ্রিনিচের ছয় ঘণ্টা অগ্রবর্তী সময় দিয়ে চলে, কিন্ত আমার হাইপোথিসিস বলে ওদের জ্ঞানের ঘড়ি আমাদের চেয়ে আগে ঘুরতে শুরু করছে এবং তার গতিবেগ ও বেশি। বিবর্তন নিয়ে গবেষণা অধিকাংশই পশ্চিমকেন্দ্রিক।আমি দেখেছি মানুষ “বিবর্তন” নিয়ে কথা বললেই নাস্তিক ট্যাগ পায়, নাহলে তাকে থামিয়ে দেওয়া হয়,অথচ পরিসংখ্যান মোতাবেক প্রায় 97% বিজ্ঞানী এটার পক্ষে। এ পরিস্থিতিতে বিবর্তন কে জনগণের কাছে পৌঁছে দিতে, কিংবা তাদের ভুল ধারণা গুলো দূর করার জন্য কি যেতে পারে, এবং সেগুলো নেওয়ার যথেষ্ট প্রণোদনা আছে কি না? এ সম্পর্কে লেখালিখি বাংলা ভাষায় মোটামুটি অপ্রতুল’ই ।এ বিষয়ে বিজ্ঞান লেখকগণ কি করতে পারেন? কিভাবে আমাদের ও একজন রিচার্ড ডকিন্স কিংবা একজন ডবঝানস্কি জন্মাতে পারে? এ বিষয়টিই আপনার কাছে জানতে চাই।
ইতি, আপনার গুণমুগ্ধ ভক্ত, আবীর।
উত্তর: আমেরিকার মত দেশের অনেক স্কুলে বিবর্তন পড়ানো যায় না, চার্চের বাধার কারনে। সেই দেশে বিশাল একটা সংগঠন আছে যারা বিশ্বাস করে পৃথিবী গোলাকার নয়, পৃথিবী সমতল। টেলিভিশনে অনুষ্ঠান দেখানো হয় যেখানে প্রমান করা হয় যে মানুষ আসলে চাঁদে যায়নি। কাজেই তুমি কেন অবাক হও যদি দেখ আমাদের দেশেও কিছু মানুষ আছে যারা বিজ্ঞানের গুরুত্বপূর্ণ বিষয় বিশ্বাস করে না? বিজ্ঞান বিজ্ঞানের মত থাকবে, বিজ্ঞানীরা তাদের মত করে গবেষণা করে বিজ্ঞানকে সামনে নিয়ে যাবে। যারা বিজ্ঞান জানে না, কুসংসস্কারে ডুবে থাকে, তাদেরকে তাদের মত থাকতে দাও। আমরা সব ধরনের অলিম্পিয়াড করে হাজার হাজার ছেলে মেয়েদের বিজ্ঞানে আগ্রহী করে যাচ্ছি, তারা নিজের মত করে সত্য বের করে নেবে। মনে কর তুমিও তাদের একজন!

প্রশ্ন: হাসিন, ঢাকাlস্যার আপনি কি টুনটুনি ও ছোটাচ্চু সমগ্র বের করবেন ?
উত্তর: কী মনে হয়?

প্রশ্ন: Happy birthday sir…. Porosh. Kuahtia
উত্তর: থ্যাঙ্ক ইউ পরশ।

প্রশ্ন: Sir, ami shuneci scientist ra naki mohajagotik pranider akta signal peyece… Sey signal ta cilo naki “wow”…. Eta ki sir sotti??!?!?!?!?!????!!! Porosh Kushtia
উত্তর: আমি জানি না, কিন্তু কমনসেন্স থেকে বলা যায় এগুলো হচ্ছে ফেসবুকের স্ট্যান্ডার্ড বানোয়াট খবর।

প্রশ্ন: Sir, চাদে যদি কোনো মহাজাগতিক প্রাণীরা আসে,তাহলে কি প্রবল আকর্ষনে চাদ পৃথিবীতে আছড়ে পড়বে??? porosh Kushtia
উত্তর: মহাজাগতিক প্রাণীরা যদি কোনো কায়দা করে চাঁদের গতিকে থামিয়ে দিতে পারে তাহলে চাঁদ পৃথিবীতে আছড়ে পড়বে।

প্রশ্ন: জ্বি স্যার আমার নাম নিয়ে অনেকেই ঠাট্টা করেছে, স্যার, ছোটদের জন্যে লেখা আপনার প্রথম বইয়ের নাম কি?? এবং আমি এখন পর্যন্ত আপনার ৩৫টি বই পরেছি বায়েজিদ হোসেন বুলবুল, নাটোর
উত্তর: মনে হয় হাত কাটা রবিন।

১৩ নভেম্বর, ২০১৯ থেকে ৬ ডিসেম্বর ২০১৯

প্রশ্ন: আপনার জন্মদিন 23 এ ডিসেম্বর ,আপনি আমার পক্ষ থেকে জন্মদিনের শুভেচ্ছা গ্রহণ করবেন। আমার একটা প্রশ্ন ছিল। প্রশ্নটি হলো: আমি বাদাম খেতে খুব পছন্দ করি। কিন্তু আমার আম্মু বলে বাদাম খেলে আমি মোটা ফুটবলের মতো হয়ে যাব।বাদাম খেলে কি আসলেই মোটা হয়?  নাম:আনিকাহ নাওয়ার ঋতি। ঈশ্বরদী,পাবনা।
উত্তর: জন্মদিনের এতো আগে এডভান্স শুভেচ্ছা জানানোর জন্য অনেক ধন্যবাদ। (তবে বয়স পঞ্চাশ থেকে বেশি হয়ে গেলে কখনো জন্মদিনের কথা মনে করিয়ে দিতে হয়না! ষাটের বেশি হয়ে যাবার পর জন্মদিন নিয়ে কথাবার্তা বলা রীতিমত অপরাধ!!) বাদাম খেলে ফুটবলের মত মোটা হয় কি না, সেটা জানার জন্য আমি তোমাকে ইন্টারনেটে একটা সার্চ দেবার কথা বলতে যাচ্ছিলাম কিন্তু আমি নিজেও এটা জানতে যাচ্ছিলাম। তাই আমি এখানে সার্চ দিয়ে ইন্টারেস্টিং কিছু জানতে পেরেছি। তোমার আম্মুকে দেখাও!

প্রশ্ন: আপনার সব Science Fiction আমি পড়ে ফেলেছি। একাধিকবার পড়েছি।খুবই ভালো লাগে আপনার বইগুলো। ‘অবনীল’ বইয়ের ‘কুশান’ চরিত্র আমার সবচেয়ে প্রিয়। প্লিজ নতুন বিজ্ঞান কল্পকাহিনী লিখুন। আমি অপেক্ষায় আছি। —তিলোত্তমা বণিক। পল্টন,ঢাকা।
উত্তর: আগামী বই মেলায় দুইটা সায়েন্স ফিকশান বের হবে। আর তোমার যদি সায়েন্স ফিকশান ভালো লাগে তাহলে অন্যদের সায়েন্স ফিকশানও পড়। অনেক সুন্দর সুন্দর সায়েন্স ফিকশান আছে।

প্রশ্ন: Sir, Adventure boi-er modhya apnar priyo konta? Lekhok ke sir? হাসিন,ঢাকাl
উত্তর: ছোট থাকতে যেভাবে পড়েছি এখন তো আর সেভাবে এডভেঞ্চারের বই পড়া হয় না। তবে এখনো আমার প্রিয় বই মার্ক টোয়েনের দি এডভেঞ্চারস অফ টম সয়ার!

প্রশ্ন: Sir hacking kora ki kharap???? Sir how can I learn hacking and programming in Bangladesh??? Ashiqul Islam Ayon Brahmanbaria
উত্তর: এটা নির্ভর করে তুমি কি জন্য হ্যাকিং করছ! ইন্টারনেটে সার্চ দিয়ে জেনে নাও এটা ভালো না খারাপ, আইনী না বে-আইনী, সাদা না কালো ইত্যাদি ইত্যাদি। হ্যাকিং কোথায় শেখা যায় আমি জানি না (ঠিক জানতেও চাই না) তবে প্রোগ্রামিং শেখার জন্য প্রোগ্রামিংয়ের একটা ভালো বই কিনে প্রোগ্রামিং শুরু করে দাও।

প্রশ্ন: স্যার সিএসই পড়ার জন্য বইয়ের একটা লিস্ট দিলে ভাল হয় প্লিজ! ফাহিম খান মিরপুর-১০,ঢাকা
উত্তর: আমার ছাত্র তামিম শাহরিয়ারের সাইটগুলোতে ঢুঁ মেরে দেখতে পার।

প্রশ্ন: স্যার পৃথিবী তো পুরোপুরি একটি বৃত্ত নয়। তবে বিজ্ঞানীরা কিভাবে এর ব্যাসার্ধ বের করল? রাফিদ বিন নাসির। কুমিল্লা।
উত্তর: সেজন্য বলা হয় সবচেয়ে বেশি এবং সবচেয়ে কম ব্যাসার্ধ। পৃথিবীর ব্যাসার্ধ হচ্ছে এই দুটির গড়।

প্রশ্ন: ২০১৬ সালে তো ১১৮ টি রসায়নের মৌল আবিষ্কার হইছে,, তাহলে ২০১৬ সালের পর কি আর কোনো মৌল আবিষ্কার হয়নি?????? NAME : DM DIN ISLAM SARKER NARSINGDI,DHAKA..
উত্তর: না। এর পরের মৌলগুলো হবে খুবই অস্থিতিশীল, সেজন্য এগুলো পাওয়া দুঃসাধ্য ব্যাপার।

প্রশ্ন: আমি খুবই রাগ করেছি। আপনি টুনটুনি ছোটাচ্চুর এতগুলি সিকুয়েল লিখলেন আর দীপু নাম্বার টু-র একটা সিকুয়েল লিখতে পারলেন নাহ? দীপুর আরেকটা বইয়ের জন্য মানুষের আবদার কি টুনটুনি ছোটাচ্চুর চাইতে কম ছিল? হুম? ইশতিয়াক, ঢাকা
উত্তর: একবার ভুল করে ফেলেছি বলে কী বারবার ভুল করা যায়? তা ছাড়া এতোদিনে দীপু নিশ্চয়ই বিয়েশাদী করে সংসারী হয়েছে। বাচ্চা কাচ্চা হয়ে গেছে। তাকে নিয়ে আর কী লিখব?

প্রশ্ন: Sir Ami akta programming club kholachi jetar nam Platinum Programming Club.Aita Brahmanbaria ta… Akta Facebook page o kholachi kinto sodosso matro 11 Jon Ami ki bhave aro member barabo Alto please bolban Ashiqul Islam Ayon Brahmanbaria Bangladesh Gas Fields School & College
উত্তর: কিভাবে সদস্য বাড়ানো যায় আমি তো জানি না। আগে থেকে যে প্রোগ্রামিং ক্লাব কাজ করছ, তাদের সাথে যোগাযোগ করে একসাথে কাজ করলে কেমন হয়?

প্রশ্ন: Amar porikkha cholchhe.doa korben . Afia Jannat,Dhaka
উত্তর: এতোদিনে নিশ্চয়ই পরীক্ষা শেষ হয়ে রেজাল্ট পর্যন্ত বের হয়ে গেছে। এখনো কি দোয়া চাও?

প্রশ্ন: রাগীব শাহরিয়ার,এসএসসি পরীক্ষার্থী,চট্টগ্রাম স্যার,আপনার মা বলে একটা বই লিখেছেন?? আপনি কি অনুগ্রহ করে আমাকে বইটির নাম বলতে পারবেন??
উত্তর: এইযে এইটা আমার মায়ের বই।

প্রশ্ন: স্যার, আপনি নতুন ছবি তুললে এখানে দিয়ে দিবেন সব সময়।। আর আমদের তো ভুলেই গিয়েছেন।। অনন্যা, ময়মনসিংহ।
উত্তর: আমি নিজে তো ছবি তুলি না। যদি কখনো তুলি এখানে দয়ে দেব। (না, আমি মোটেও ভুলিনি, আমি আসলে খুব বড় ধরনের একটা চাপের মাঝে ছিলাম)

প্রশ্ন: জাফর স্যার, বছর সাস্টে পড়ে মনে হল, আমাদের দেশটা বড়ই গরিব। এখানে ভাল কোন স্কুল নেই। ইংলিশ মিডিয়াম তো নেই ই। বিশ্ববিদ্যালয়গুলোও যাচ্ছেতাই। মানুষজন ভালো না। ঐসব মানুষও যাদের এই দেশে শ্রদ্ধার চোখে দেখা হয়। এই দেশ এতই হতচ্ছাড়া যে কোন বইও এখানে পাওয়া যায় না। বই নিয়ে আসতে হয় আমেরিকা থেকে। ইয়াসমিন ম্যাডাম এই দেশের মানুষ না। তাঁকে অনেক কষ্ট করতে হয়েছিল এখানে থাকতে ।আমেরিকা ভাল ।আমেরিকা সুন্দর বাচ্চারা এইখানের খাবার খেতে পারেনি । যদিও তারা ঢাকা থেকে রান্না করা পাঠানো খাবার খেতে পারত ।বাচ্চাদের দেশ এইটা না ।তাদের দেশ আমেরিকা।আর….। খুব কষ্ট লাগল স্যার।শুধু একটি প্রশ্ন, আপনিও কি ইয়াসমিন ম্যাডামের সাথে একমত? সৌমিত্র,ঢাকা
উত্তর: হায় হায়, তুমি বইটার মূল বিষয়টা মনে হয় ধরতে পারনি। বইটা আরো একবার পড়।

প্রশ্ন: Rajanya Dey, Patharghata, Chittagong.
I read in Bsc in Mathematics, 2nd Year in NU. I am very much interested in Quantum Mechanics. I am also interested in Parapsychology(Misir ali). Is there any relation between Parapsychology and Quantum Mechanics. I want to research in Quantum Mechanics in USA. Is it possible ? Which book I should read for it? I write it in english because I use Linux and there is no Bangla front .SORRY for that.
উত্তর: প্যারাসাইকোলজি বিজ্ঞান নয়, কোয়ান্টাম মেকানিক্স বিজ্ঞান। কোয়ান্টাম মেকানিক্স শেখার জন্য আমার প্রিয় বই হচ্ছে:

প্রশ্ন: প্রিয় নানু(যেহেতু তুমি তোমার “তোমাদের প্রশ্ন আমার উত্তর ” বইটিতে তোমাকে নানু বলে ডাকলে খুশি হবে বলেছিলে,তাই ঠিক করেছি এখন থেকে তোমাকে নানু বলেই ডাকব ! ), আশা করি ভালো আছ। আমিও ভাল আছি। তবে এই ভেবে দুঃখ পাচ্ছি যে তোমার বেশীর ভাগ বই-ই আমার পড়া। যার ফলে নতুন কোন বই খুজে পাচ্ছি না যে পড়বো। আমি তোমার কাছে তিনটি বিষয় জানার জন্য এই চিঠিটি লিখতেছি।
প্রশ্ন ৩টি হলো:
১। কিভাবে আমি একজন সৎ ও নীতিবান মানুষ হতে পারব?
২।কিভাবে আমি মোবাইল আসক্তি থেকে দুরে থাকতে পারব?
৩।কিভাবে আমি প্রাইভেট না পড়ে,গাইড বই ফলো না করে পড়াশোনা করতে পারব ?
প্রশ্ন ৩টির উত্তর দাও প্লিজ, আমার খুবই প্রয়োজন । আর হ্যা,তুমি কি একদিন কুড়িগ্রামে আসবে? তোমাকে বাস্তবে দেখার খুব ইচ্ছা। আশিক বাবু রাজারহাট পাইলট উচ্চ বিদ্যালয় কুড়িগ্রাম
উত্তর: তোমার প্রশ্নের উত্তর কেমন করে দেব বুঝতে পারছি না! কারন উত্তরগুলো এরকম:
১। তুমি যদি সৎ ও নীতিবান থাক, তাহলেই তুমি সৎ ও নীতিবান মানুষ হতে পারবে।
২। তুমি যদি মোবাইল আসক্ত না হও তাহলেই তুমি মোবাইলে আসক্তি থেকে দুরে থাকতে পারবে।
৩। তুমি প্রাইভেট না পড়ে, গাইড বই ফলো না করে পড়াশোনা কর তাহলেই তুমি প্রাইভেট এবং গাইড বই ছাড়া পড়াশোনা করতে পারবে!
আমি অনেক চিন্তা করেও এভাবে ছাড়া অন্য কিভাবে তোমার প্রশ্নের উত্তর দেওয়া যায় বের করতে পারলাম না। সরি! তুমি যেহেতু সমস্যাগুলো বের করে ফেলেছ এখন সেগুলো না করাই হচ্ছে সমাধান। ঠিক কিনা?
অবশ্যই একদিন আমি তোমাদের ওখানে বেড়াতে আসব, তখন তোমার সাথে দেখা হবে।

প্রশ্ন: ইকরা, পাবনা আমার সমাপনী পরীক্ষা চলছে।দোয়া করবেন যেন ভালো কিছু করতে পারি ।
উত্তর: এতোদিনে তোমার সমাপনী পরীক্ষা শেষ হয়ে মনে হয় রেজাল্টও হয়ে গেছে। এখন দোয়া করলে কি লাভ হবে?

প্রশ্ন: Sir, please write higher class book on quantum mechanics for master’s level student in bengali.
উত্তর: ইংরেজিতে কোয়ান্টাম মেকনিক্সের উপর অসাধারণ কিছু বই আছে, আমাদের দেশে সবাইকে টানা বারো বছর ইংরেজি শেখানো হয়, তাই আমার মনে হয় আসলে বাংলায় আমার এর উপর বই লেখার কোনো প্রয়োজন নেই।

প্রশ্ন: স্যার, বায়ুতে শব্দের বেগ কত? আমাদের পদার্থবিজ্ঞান বইয়ে আছে 330 ms^1. কিন্তু আমাদের শিক্ষক আমাদের 332 ms^1 লিখতে বলেছেন। পরীক্ষার সময় কোনটা লিখব? আমর আল আনাস টঙ্গী,গাজীপুর।
উত্তর: বাতাসে শব্দের বেগ তাপমাত্রার উপর নির্ভর করে। তোমাদের বইয়ে তার উপরে একটা প্রবলেমও আছে। সেটা থেকে তোমরা নিজেরাই বের করতে পারবে কোন তাপমাত্রায় শব্দের বেগ কত।

প্রশ্ন: স্যার, আমি আপনার মতো একজন শিক্ষক হতে চাই বড় হয়ে। অবন্তি।
উত্তর: শুনে খুব খুশী হলাম। দোয়া করি যেন খুব ভালো একজন শিক্ষক হতে পার। আমার থেকে অনেক ভালো একজন শিক্ষক।

প্রশ্ন: বুঝতে পারছি আমাদের ভুলে গিয়েছেন!! স্মৃতি, ময়মনসিংহ।
উত্তর: ভুলি নাই। চাইলেও তোমরা কি ভুলতে দেবে?

প্রশ্ন: স্যার, সাস্টের ওয়েবসাইটে আপনার টেলিফোন নাম্বার দেওয়া আছে। আমি সেটাতে অনেকবার ফোন দিয়েছি কিন্তু ফোন যায় না কেন স্যার??? অবন্তি, ময়মনসিংহ ।
উত্তর: আমকে ফোনে পাওয়া যায় না, তার কারন আমি ফোন-টোন ধরি না। দরকার হলে কখনো কখনো ফোন করি!

প্রশ্ন: আপনাকে টুনটুনি সিরিজ আরও লিখতে হবে।আহনাফ প্রত্যয়, টাংগাইল।
উত্তর: আচ্ছা, শুনে রাখলাম।

প্রশ্ন: স্যার, ১৪ই অক্টোবর আপনি ময়মনসিংহ আসছিলেন। দুর্ভাগ্যবসত আমি জানতাম না। আমি সন্ধায় জানতে পারছিলাম। স্যার কোথায় কোথায় গিয়েছিলেন ?? আবার কবে আসবেন ???? অবন্তি, ময়মনসিংহ।
উত্তর: মনে হয় কোনো একটা বড় অনুষ্ঠানে, এখন মনে করতে পারছি না। আবার কবে যাব এখনো জানি না।

প্রশ্ন: Sir, remember, I wanted your blessing before the JSC exam. I did all the tests well.
Thanks for the blessing ………. (Although there is no reason, please, do not reveal my name.)
উত্তর: চমৎকার! শুনে খুব ভালো লাগল। (তোমার নামটা গোপন রাখলাম!)

প্রশ্ন: আংকেল, আপনি না অবসর নিয়েছেন। এখন কি মজা আপনার! দিনকাল কাটান কীভাবে সেটা বলবেন কিন্তু। আর এই ওয়েবসাইটে (কয়েক) মাসে একবার উঁকি দিয়ে চলে যাবেন না। রেগুলার আসবেন। ইশতিয়াক, ঢাকা
উত্তর: এখনো অবসরের আনন্দ ভোগ করতে পারছি না। কাজ করতে করতে জান বের হয়ে যাচ্ছে।

প্রশ্ন: স্যার অনেক ওয়েবসাইট এ আপনিসহ অনেক লেখকের বই ই-বুক আকারে প্রকাশিত হয় । এগুল কি কপিরাইট আইনের বিরুদ্ধে ??????????? প্লিজ উত্তর লিখবেন । মোস্তফা নুরুল আলম, ৯ম শ্রেণি,গোপালগঞ্জ ।
উত্তর: অনুমতি না নিয়ে বের করলে মনে হয় কপিরাইট আইনের বিরুদ্ধে।

প্রশ্ন: স্যার আপনার মতে আপনার সাইটটি ভাঙ্গাচোরা ।কিন্তু আমার ত মনে হয়যে সাইটটি চমৎকার ।আর আপনি বেশ ভাল ভাবেই এটি পরিচালনা করছেন।এ জন্য আপনাকে অনেক ধন্যবাদ। আপনার নাম-ঠিকানা ছাড়া প্রশ্নের উত্তর না দেওয়ার সিদ্ধান্তটি আমার অনেক ভাল লেগেছে। মোস্তফা নুরুল আলম, ৯ম শ্রেণি, গোপালগঞ্জ।
উত্তর: থ্যাঙ্ক ইউ! থ্যাঙ্ক ইউ!!

প্রশ্ন: আচ্ছা স্যার, আপনাকে তো অনেকে অনেক নামে সম্বোধন করে যেমন -স্যার,দাদু ,আংকল ইত্যাদি ।[ আমি নিজেই আপনাকে স্যার বলে সম্বোধন করলাম ] এটা আপনার কাছে মজাদার লাগে না ???। আমার তো অনেক মজা লাগে………… please answer me…………….নওরীন, গোপালগঞ্জ
উত্তর: আমাকে এতো মানুষ স্যার বলে ডাকে, যে মাঝে মাঝে মনে হয় যে এটাই বুঝি আমার নাম!

প্রশ্ন: Shahreen Khan Taan,10th grader Bashundhara residential area, dhaka.
আমি আপনাকে আজ স্বপ্নে দেখেছি, স্যার। আমি আপনাকে এত ভালোবাসি কেন? আমি জানি আপনি আমাদেরকে আরো অনেক বড় স্বপ্ন দেখতে বলেন। আপনি এই কথা শুনতে শুনতে বিরক্ত হয়ে গেলেও এটা সত্য যে আপনার সাথে দেখা করা আমার একটা স্বপ্ন। কিন্তু আমার কেন জানি মনে হয় কখনো দেখা হবে না।
আপনি সিলেট থাকাকালীন আমার সিলেট যাওয়া হবে না। আমি যেদিন বইমেলা যাব আপনি সেদিন থাকবেন না। আমি যেদিন লিট ফেস্টে গিয়েছি সেদিনও আপনি ছিলেন না। থাক, কি আর করার। আপনার লেখনীর মাঝেই আপনাকে খুঁজে নিতে হবে।
অনেক অনেক অনেক ভালোবাসা আপনার জন্য। এক মাল্টিভার্স সমান ভালোবাসা। সবসময় ভালো এবং সুখে থাকবেন, প্রিয় মানুষ! এটাই কামনা
উত্তর: তোমার কথাগুলো পড়ে আমার মনটা ভালো হয়ে গেল। (আমি যখন তোমার চিঠির উত্তর লিখছি, তখন আসলে আমার মনটা খুব খারাপ।) তোমরা সবাই মিলে আমার মনটা ভালো করে দাও বলেই আমি টিকে আছি। (দেখা হবে না কেন? নিশ্চয় দেখা হবে একদিন। তোমার সাথে যেদিন দেখা হবে সেদিন তুমি আমাকে মনে করিয়ে দিও যে আমার সাথে তুমি খুব দেখা করতে চাইছিলে।)

১৪ অক্টোবর থেকে ১৩ নভেম্বর, ২০১৯

প্রশ্ন: আসসালামুয়ালাইকুম স্যার। আমার নাম আফরা মিনান নুর। আমরা দুই ভাই বোন। আমাদের মা কোনোদিন খাওয়া নিয়ে জোর করে না। টুনটুনি ও ছোটাচ্চু তে এই বিষয় নিয়ে লিখলে আমারা ভীষণ খুশি হব।
উত্তর: পৃথিবীর সব আব্বু আম্মু যদি তোমাদের আববু আম্মুর মত হত তাহলে বাচ্চা কাচ্চাদের আর কোনো চিন্তা থাকতো না। টুনটুনি ও ছোটাচ্চু তে লেখাটা গুড আইডিয়া। চেষ্টা করব।

প্রশ্ন: আপনার প্রিয় 20 টি বই এর নাম একটু দয়া করে জানাবেন? কাশফিয়া , মালতিনগর বগুড়া ।

উত্তর: কয়েকটা দিলাম। তুমি কি বাংলা বইয়ের নাম জানতে চাচ্ছিলে?

প্রশ্ন: টুনটুনি ও ছোটাচ্চু এর নতুন গল্পের বই কবে বের হবে জাফর ইকবাল সার?-রাহনুমা ইসলাম,মিরপুর-১০,ঢাকা
উত্তর: বের হবে কি না এখনো জানি না। যদি লিখতে পারি তাহলে বইমেলায় বের হবে। তাহলে আগেই এই সাইটেও গল্পগুলো দিতে পারি।

প্রশ্ন: Dadu, Dry ice a hat dila hat pura jay kano? Name:Ashiqur Rahman Nurpur,Pabna
উত্তর: “why dry ice burn you” লিখে ইন্টারনেটে একটা সার্চ দাও। আমি চাই তোমরা ইন্টারনেটে তোমরা নিজেরা তোমাদের প্রশ্নের উত্তর বের করা শিখে যাও।

প্রশ্ন: চাঁদের আলো কোমল কেন?(সুস্পষ্ট ব্যাখ্যা স্বরূ)[মেহরিন তাসনিম, আশুলিয়া, সাভার, ঢাকা]
উত্তর: এই প্রশ্নের উত্তর ইন্টারনেটে পাওয়া একটু মুশকিল হতে পারে, তাই আমি দিচ্ছি। চাঁদের আলো যেহেতু মোটেও তীব্র আলো না, তাই আমাদের চোখের রেটিনার রং দেখতে পারা আলো সংবেদী কোষ Cone গুলো কাজ করতে পারে না, তখন কাজ করে Rod নামের কোষগুলো, যেগুলো কম আলোতে কাজ করতে পারে কিন্তু রং দেখতে পারে না। তাই চাঁদের আলোতে আমরা রং দেখতে পাই না। শুধু তাই না আলো কম বলে চাঁদের আলোতে contrast কম। সেজন্যে মনে হয় চাঁদের আলো হচ্ছে কোমল।

প্রশ্ন: Assalamu Alaikum .How are you,sir? Actually , this is my first text to a special person. How is your family? Now, most important thing ,my question is “Do you continue ‘Tuntuni & Chotacchu’ series ? And what’s the plan about next Ekushey Boi Mela ? I eager to read your books. That’s all.Please,give my respect and love to your family.Good bye.Allah Hafez..(Bushra,Dhaka)
উত্তর: এখনো জানি না, টুনটুনি আর ছোটাচ্চু নিয়ে নুতন কোনো বই লিখতে পারব কিনা! যদি পারি বই মেলায় বের হবে। বই মেলার জন্য দুইটা সায়েন্স ফিকশান লিখেছি। ব্ল্যাক হোলের উপরেও একটা বই লিখেছি। একটা কিশোর উপন্যাস লেখার চেষ্টা করছি। দেখি কতদূর যেতে পারি।

প্রশ্ন: স্যার আমার নাম শান্ত । আমি আপনার অনেক বড় ভক্ত । গত ২ মাস ধরে প্রায় ৯ টি প্রশ্ন আপনাকে লিখেছি । কোনটার উত্তর পাই নি । ( এখন বলতে গেলে অনেক সময় লাগবে। আজ খাক। ) আমি আপনার autograph চাই । প্লিজ !প্লিজ !প্লিজ !প্লিজ !প্লিজ !প্লিজ !প্লিজ !প্লিজ !প্লিজ !প্লিজ !প্লিজ !প্লিজ !প্লিজ !প্লিজ !প্লিজ !প্লিজ !প্লিজ !প্লিজ !প্লিজ !প্লিজ !প্লিজ !প্লিজ !প্লিজ !প্লিজ !প্লিজ !প্লিজ !প্লিজ !প্লিজ !প্লিজ !প্লিজ !প্লিজ !প্লিজ !প্লিজ !প্লিজ !প্লিজ !প্লিজ !প্লিজ !প্লিজ !প্লিজ !প্লিজ !প্লিজ !প্লিজ !প্লিজ !প্লিজ !প্লিজ !প্লিজ !প্লিজ !প্লিজ !প্লিজ !প্লিজ !প্লিজ !প্লিজ !প্লিজ !প্লিজ !প্লিজ !প্লিজ !প্লিজ !প্লিজ !প্লিজ !প্লিজ !প্লিজ !প্লিজ !প্লিজ !প্লিজ !প্লিজ !প্লিজ !প্লিজ !প্লিজ !প্লিজ !প্লিজ !প্লিজ !প্লিজ !প্লিজ !প্লিজ !প্লিজ !প্লিজ !প্লিজ !প্লিজ !প্লিজ !প্লিজ !প্লিজ !প্লিজ !প্লিজ !প্লিজ !প্লিজ !প্লিজ !প্লিজ !প্লিজ !প্লিজ !প্লিজ !প্লিজ !প্লিজ !প্লিজ !প্লিজ !প্লিজ !প্লিজ !প্লিজ !প্লিজ !প্লিজ !প্লিজ !প্লিজ !প্লিজ !প্লিজ !প্লিজ !প্লিজ !প্লিজ !প্লিজ !প্লিজ !প্লিজ !প্লিজ !প্লিজ !প্লিজ !প্লিজ !প্লিজ !প্লিজ !প্লিজ !প্লিজ !প্লিজ !প্লিজ !প্লিজ !প্লিজ !প্লিজ !প্লিজ !প্লিজ !প্লিজ !প্লিজ !প্লিজ !প্লিজ !প্লিজ !প্লিজ !প্লিজ !প্লিজ !প্লিজ !প্লিজ !প্লিজ !প্লিজ !প্লিজ !প্লিজ !প্লিজ !প্লিজ !প্লিজ !প্লিজ !প্লিজ !প্লিজ !প্লিজ !প্লিজ !প্লিজ !প্লিজ !প্লিজ !প্লিজ !প্লিজ !প্লিজ !প্লিজ !প্লিজ !প্লিজ !প্লিজ !প্লিজ !প্লিজ !প্লিজ !প্লিজ !প্লিজ !প্লিজ !প্লিজ !প্লিজ !প্লিজ !প্লিজ !প্লিজ !প্লিজ !প্লিজ !প্লিজ !প্লিজ !প্লিজ !প্লিজ !প্লিজ !প্লিজ !প্লিজ !প্লিজ !প্লিজ !প্লিজ !প্লিজ !প্লিজ !প্লিজ !প্লিজ !প্লিজ !প্লিজ !প্লিজ !প্লিজ !প্লিজ !প্লিজ !প্লিজ !প্লিজ !প্লিজ !প্লিজ !প্লিজ !প্লিজ !প্লিজ !প্লিজ !প্লিজ !প্লিজ !প্লিজ !প্লিজ !প্লিজ !প্লিজ !প্লিজ !প্লিজ !প্লিজ !প্লিজ !প্লিজ !প্লিজ !প্লিজ !প্লিজ !প্লিজ !প্লিজ !প্লিজ !প্লিজ !প্লিজ !প্লিজ !প্লিজ !প্লিজ !প্লিজ !প্লিজ !প্লিজ !প্লিজ !প্লিজ !প্লিজ !প্লিজ !প্লিজ !প্লিজ !প্লিজ !প্লিজ !প্লিজ !প্লিজ !প্লিজ !প্লিজ !প্লিজ !প্লিজ !প্লিজ !প্লিজ !প্লিজ !প্লিজ !প্লিজ !প্লিজ !প্লিজ !প্লিজ !প্লিজ !প্লিজ !প্লিজ !প্লিজ !প্লিজ !প্লিজ !প্লিজ !প্লিজ !প্লিজ !প্লিজ !প্লিজ !প্লিজ !প্লিজ !প্লিজ !প্লিজ !প্লিজ !প্লিজ !

উত্তর: এই দেখ, তোমার জন্য অটোগ্রাফ রেডি করে রেখেছি, কিন্তু পাঠাব কোথায়?

প্রশ্ন: আপনার একটা লেখা পড়েছিলাম যেখানে বলেছিলেন আপনার কোনো এক সন্তান নিজে নিজেই পড়া শিখে গিয়েছিলো , কেননা, আপনারা তাকে গল্পের বই পড়ে শোনাতেন । এক বন্ধুকে লেখাটা দিতাম । কিন্তু, লিংক পাচ্ছি না । আপনার সহায়তা কাম্য ! 🙂 গালিব ৪র্থ বর্ষ খাজা ইউনুস আলি মেডিকেল কলেজ
উত্তর: হা হা হা। তোমার ধারনা তুমি যে লেখাটা খুঁজে পাচ্ছ না সেটা আমি খুঁজে বের করে ফেলব? তোমার বন্ধুকে বল আমি সার্টিফিকেট দিচ্ছি কথাটা সত্যি। শুধু আমার ছেলে মেয়ে না, আমার কথা শুনে আমার পরিচিত যারা তাদের ছেলেমেয়েদের বই পড়ে পড়ে শুনিয়েছে, সবাই নিজে নিজে পড়তে শিখে গেছে। (যদি তোমার কথা বিশ্বাস না করে এই চিঠির উত্তরটা দেখাও।)

প্রশ্ন: Sir amar salam grohon korben.apner kache cethi lekar address ki jante pari? Jeba class 9 sirajgonj
উত্তর: এখনও পাকা একটা ঠিকানা হয়নি। তাই আপাতত আমার বিশ্ববিদ্যালয়ের ঠিকানায় পাঠাতে থাকো।

প্রশ্ন: আচ্ছা, আপনার ছোট বেলায় আপনি কি কখনো আপনার দাঁত ব্রাশ করতে ভুলে গেছেন? – নওরীন গোপালপুর
উত্তর: কতবার!

প্রশ্ন: ইকরা, আচ্ছা আপনিতো জীবনে প্রচুর বই লিখেছেন ,সেগুলোর মাঝে আপনার প্রিয় বই কোনটি? ভালো থাকবেন । আমর ও লিখবেন। ঢাকা।।
উত্তর: সেভাবে বলা মুশকিল। আমার বন্ধু রাশেদ, রাশা, টুকুনজিল এরকম কয়েকটা বই লিখে একধরনের তৃপ্তি হয়েছে বলা যায়।

প্রশ্ন: স্যার, আমি একটা গল্প লিখেছি,,, সেটা আপনাকে দেখাতে চাই৷ তো কিভাবে দেখাবো?
উত্তর: ই-মেইলে আমার কাছে পাঠিয়ে দাও।

প্রশ্ন: মোঃ আবুল খায়ের, যাত্রাবাড়ী, ঢাকা। শ্রদ্ধেয় স্যার, আপনার লেখা পড়তে পড়তে কখন যে কৈশোর পেরিয়ে তারুণ্যে এসে পড়েছি বুঝতে পারিনি। এখনও নিজেকে কিশোর কিশোর মনে হয় তাই আপনার লেখা বের হওয়া মাত্রই পড়ে ফেলি। তরুণ বা বড়দের জন্য আপনি অল্প কিছু বই লিখেছেন সেগুলোও পড়ে ফেলেছি্। যেহেতু আপনি এখন অধ্যাপনা থেকে রিটায়ারমেন্টে আছেন, তাই আপনি কি পারেন না বড়দের জন্য নিয়মিত কিছু বই লিখতে ? সেটা প্রতি বছর অন্তত একটি বই হতে পারে। আপনাকে অনেক অনেক ভালবাসা।
উত্তর: আমারও মাঝে মাঝে বড়দের জন্য লিখতে ইচ্ছা করে, কিন্তু ছোটরা আমাকে হুমকি দিয়ে রেখেছে যে আমি যেন কখনো বড়দের জন্য লিখে সময় নষ্ট না করি!

প্রশ্ন: char ami matra computer chalano sikese ti bangla te lekhte parsina . ami apnake amar thikana bolte parsina amar naam md gakir ahmed . koto din kono golpo likhchen na . bapar ki . ami apnar anek boi poresi kintu tar anek gulatei ami dekesi apni anek zigai likesen issar nai . ekhane apni ki bojate chesen . amar to mone hoi ei jonnoy apnake oi cheleti apna ke chaku meresil . ar jamate islamira oi jonnoy apnar opor ato khepe ache
উত্তর: আমার কেন জানি মনে হচ্ছে তোমার নাম গাকির আহমেদ (gakir ahmed) না, তোমার নাম জাকির আহমেদ (Jakir ahmed), ঠিক কি না? ঠিকানা না লিখলে উত্তর নাই। (তুমি কি আমার নাম জানো? আমার বই মনে করে তুমি না জানি কার বই পড়ছ!)

প্রশ্ন: স্যার পরীক্ষার জন্য কোনো পাঠ্যবর্হিভূত বইপড়তে পারছি না। তাই নিজেকে অনেক একা একা লাগছে। আমার জন্য দোয়া করবেন ” ৫ নভেম্বর আমার টেস্ট পরীক্ষার রেজাল্ট দিবে”-মেহরিন তাসনিম সাভার, ঢাকা
উত্তর: এতো দিনে নিশ্চয় টেস্ট পরীক্ষার রেজাল্ট হয়ে গেছে। খুব ভালো হয়েছে না?

প্রশ্ন: স্যার,আমার মাঝে মাঝে মনে হয় আমাদের কলেজে ভরতি না হয়ে প্রাইভেটে ভরতি হওয়া উচিত ছিল।আমার সব বান্ধবিরা ছয়টা সাতটা করে প্রাইভেত পড়ে।কেও কেও আরো বেশি।তাদের ধারনা কলেজএ ভরতি মানেই সব বিষয় প্রাইভেট পড়তে হবে।স্যার প্রাইভেট ছাড়া কি কেউ ভাল করতে পারে না। নিশি,ময়মনসিংহ।
উত্তর: প্রাইভেট পড়ে সময় নষ্ট কর না। যে টাকা প্রাইভেট পড়তে দিতে হয় সেই টাকা দিয়ে ফুর্তি কর আর নিজে নিজে পড়। দেখবে জীবন উপভোগ করার জন্য কত সময় পাবে।

প্রশ্ন: আসসালামু আলাইকুম স্যার আমি সেকেন্ড টাইম এডমিশন টেস্ট দিচ্ছি। গতবার একটা ইউনিভারসিটি তে এক্সাম দিয়ে আসার সময় বাসে একজনের সাথে প্রশ্ন নিয়ে কথা হচ্ছিলো।ফিজিক্স এর একটা উত্তর নিয়ে আমাদের মধ্যে দ্বিমত হয় আমি ওকে বোঝানোর চেষ্টা করলে ও বলে এটাই হবে আমি মুখস্থ করে আসছি। আমি খুব অবাক হয়ে গেলাম ফিজিক্স এর এত সুন্দর একটা জিনিস না বুঝে মুখস্থ? পরে বুঝতে পারলাম অই উত্তর টা ওর ভুল হলেও অধিকাংশ উত্তর ই ওর ঠিক হয়েছে মুখস্থ করে। ও একটা ভার্সিটিতে চান্স ও পেয়ে গেছে। আর আমি এবারো এক্সাম দিচ্ছি জানিনা কি হবে কারণ আমি তো মুখস্থ করতে পারিনা। আমি জানি স্যার পৃথিবী তে বেচে থাকাটা সবচেয়ে বেশি ইম্পরট্যান্ট। কিন্ত স্যার সত্যি বলছি এবার চান্স না পেলে আশে পাশের মানুষের জ্বালায় আমার মরে যাওয়া ছাড়া কোনো উপায় থাকবেনা। স্যার আমি কোচিং করিনি কারণ আমার ভাল লাগেনা ভেবেছি নিজে নিজে পড়েই পারব কিন্তু সবার দৃঢ় বিশ্বাস কোচিং ছাড়া আমি জিবনেও চান্স পাবনা। স্যার আমার একটা সমস্যা হচ্ছে এক্সাম হলে একটা অংক না মিললে পরের টায় যেতেই পারিনি তাই আমি সময় মেনেজ করতে পারিনা আর এক্সাম খারাপ হয়। এখন কি করব স্যার? আর আমি একজন astrophysicist হতে চাই জানি না আমার স্বপ্ন পূরণ হবে কি না। মাহা, ঢাকা
উত্তর: এতোদিনে নিশ্চয় ভর্তি পরীক্ষা হয়ে গেছে এবং তুমি তোমার পছন্দের কোনো বিষয়ে চান্স পেয়ে গেছ। দোয়া করি তুমি যেন একজন astrophysicist হতে পার।

প্রশ্ন: Hasin, Dhaka. Sir, amar choto thekei poralekha bhalo lage na . Tai ami bhabtam boi manei poralekhar boi. Tai ami kono boi portam na. Kintu class 4 e ammu amake library theke ekta boi porte bollo. Library te giye apnar lekha “Litu brittanto ” er prochchodta khub bhalo laglo. Bashae ene porlam. Pore ato bhalo laglo je bashae apnar shob boi pora shuru korlam. Ekhon ami onek boi pori. Shudhu apnar na, Mark Twain, Humayan Ahmed, Ahsan Habib er boi o pori. Apnar karone amar boi er proti je chinta chilo ta bodle gelo. Tai apnake onek dhonyabad.
উত্তর: না, বই মানেই পড়া লেখার বই না। শুনে খুশি হলাম যে তুমি এখন বই পড়তে পছন্দ কর। বই পড়ার থেকে ভালো কোনো অভ্যাস পৃথিবীতে নেই।

নাম: গোপন! নরসিংদী সদর, নরসিংদী, ঢাকা, (স্যার আপনার কাছে আমার পরিচয় গোপন রাখার জন্য অনুরোধ করছি) স্যার আমি কিছু গল্প লিখেছি। কিন্তু প্রকাশের কোন মাধ্যম খুঁজে পাচ্ছি না। আমার প্রথম গল্প ডায়মন্ড রহস্য। এটা একটা গোয়েন্দা গল্প। আমি এরকম আরো কয়েকটি গল্প লিখেছি। আমি কিশোর সাহিত্যও লিখেছি। কিন্তু আমার একমাত্র বাধা হচ্ছে প্রকাশ কিভাবে করব ভেবে পাচ্ছি না। স্যার, দয়া করে আপনি যদি কোন মাধ্যম জেনে থাকেন তাহলে প্লিজ বলবেন। আমি আশা করি আমার একটু উপকারে আপনি খুশি হবেন। স্যার আপনি জেনে খুশি হবেন যে, আমি আপনার বইয়ের পোকা। স্যার, আপনার কাছে অনুরোধ করছি প্রশ্নের উত্তর দেওয়ার জন্য। আমাকে ক্ষমা করবেন স্যার, কারণ আমি গুছিয়ে কথা বলতে পারিনা। আমি একটা গল্প লিখে একজনকে পড়তে দিয়েছিলাম। সে গল্পটা পড়ে বলল, এটা তুমি লিখেছ? আমি বিশ্বাস করি না। মনে হয় কোন লেখক এর কাছ থেকে চুরি করেছ।
এর উত্তর কি দেব স্যার?
উত্তর: বাংলাদেশে অনেক ম্যাগাজিন, সেখানে পাঠাও যদি আসলেই ভালো লিখতে পার, তারা সেই লেখা ছাপাবে। তোমার পরিচিত কেউ যদি মনে করে তুমি কোন লেখক এর কাছ থেকে চুরি করেছ তার উত্তর দেবার দরকার নেই, ওর দিকে তাকিয়ে মুচকি হাসতে পার! তার কারন তুমি নিশ্চয়ই ভালো লিখেছ।

প্রশ্ন: স্যার, আপনি আবারও অনিয়মিত হয়ে গিয়েছেন। এতোদিন পর পর আসেন। সাইমা, ঢাকা।
উত্তর: এই যে আবার চলে এসেছি! (তুমি ঠিকই বলেছ। কাজের ঝামেলায় কিছুই করতে পারি না। সরি!)

প্রশ্ন: Assalamu Alaikum . Ami Rahma , Ideal School and College er class 7 er student . Tuntuni o Chocchur comic ki ber kora shombhob ?
উত্তর: সম্ভব তো বটেই কিন্তু আমি বের করতে চাই না। তুমি যখন একটা বই পড় তখন তুমি চরিত্রগুলো, তাদের কাজকর্ম কথাবার্তা সবকিছু নিজের মত করে কল্পনা করতে পার। কিন্তু যদি কমিক তৈরি করে ফেলা হয়, তাহলে তুমি নিজের মত করে কল্পনা করবে না, ছবির মত করে কল্পনা করবে। আমি চাই তোমরা সবসময় নিজের মত করে কল্পনা কর।

প্রশ্ন: স্যার আপনার মোবাইল নাম্বরটা দেবেন প্লিজ?? বুলবুল, নাটোর
উত্তর: যখন তোমার সাথে দেখা হবে, তখন নিয়ে নিও। এখানে দিয়ে দিলে মনে হয় একটু ঝামেলা হতে পারে। (সাইক্লোনের পরে সবাই নিশ্চয় তোমার নাম নিয়ে অনেক রকম ঠাট্টা করেছে! আমিও করব?)

প্রশ্ন: আপনার ডাক নাম কি মেকু??? বায়েজিদ হোসেন বুলবুল, কাদিরাবাদ ক্যান্টনমেন্ট পাবলিক স্কুল, দয়ারামপুর নাটোর
উত্তর: না, আমার কোনো ডাক নাম নেই। বলা যেতে পারে আমার ডাক নাম হচ্ছে  ‘জাফর স্যার’! অনেক বড় বড় বয়স্ক মানুষকেও দেখেছি আমাকে ‘জাফর স্যার’ ডাকেন!

প্রশ্ন: স্যার, কেমন আছেন??সবাই তো আপনাকে জাফর স্যার ডাকে, আমি ঠিক করেছি আমি আপনাকে ইকবাল স্যার বলে ডাকব। অবন্তি, ময়মনসিংহ।
উত্তর: তোমার ইচ্ছা!

প্রশ্ন: নাম প্রকাশে অনিচ্ছুক মিরপুর ১ ঢাকা। স্যার আমি আপনার একজন ভক্ত । কয়েকদিন ধরে আমি অনেক সমস্যায় ভুগছি । আমার ক্লাসের কিছু ছেলেরা আমাকে রাস্তায় যাওয়ার সময় থামিয়ে বিরক্ত করে (আমি একজন ছেলে) মাঝে মাঝে হাল্কা চড় থাপ্পড় মারে । একদিন জোর করে সিগারেট মুখের মাঝে ঠেসে দিতে চাইল । আমি বিষন্ন হয়ে পড়ছি দিন দিন এ অবস্থা থেকে মুক্তি চাই , কি করণীয়?
উত্তর: তোমার নিজের ক্লাসের ছেলে হলে ওদের  বিন্দুমাত্র গুরুত্ব দিও না। যদি দেখে তুমি কোনো পাত্তা দিচ্ছ না, তাহলে কয়দিনের ভেতরে উৎসাহ হারিয়ে ফেলার কথা। কিন্তু তুমি যদি বিষণ্ণ হয়ে পড়, ভয় পাও, কোনোভাবে react কর তাহলে পেয়ে বসবে, আরো বেশি করে করবে।

প্রশ্ন: স্যার সাস্টের ভর্তী পরিক্ষার প্রশ্ন কী আপনি করেন? প্রশ্ন অনেক কঠিন কঠিন লাগে…! মো:সুজন ধানমন্ডী ঢাকাত
উত্তর: না, এখন আমি করি না। আগে করেছি। কঠিন কেন লাগবে?

প্রশ্ন: Sir amar salam nien.Apni ki please Humaun Ahmed Sir er Himu & Mishir Ali r story book gulo lekha suru korben.Oi story book gulo amar onek pochondo.(English alphabet die Bangla likhar jonno sorry.Mobile e Bangla keybord nai.Tai jonne eibhabe liklam) NAME:Debi ADDRESS:Dinajpur
উত্তর: হা হা হা। একজনের লেখা কি আর আরেকজন লিখতে পারে?

প্রশ্ন: আপনি এতো ভালো গল্প লিখেন কিভাবে??? নাম :বুলবুল ঠিকানা :আব্দুলপুর লালপুর নাটোর
উত্তর: কাগজ আর কলম নিয়ে তুমি একদিন চেষ্টা করে দেখ, তুমিও পারবে।

প্রশ্ন: প্রিয় স্যার, আপনার লেখা শান্তা পরিবার গল্পে আমরা ভেতরে বেলুনওয়ালা মজার একটি কেকের কথা পড়েছিলাম,চাকু ছোঁয়াতেই কেকটি প্রচণ্ড শব্দে ফেটে গিয়েছিলো! আমরা দুইজন মিলে বাবার জন্মদিনে ব্যাপারটা করতে গিয়ে দেখি আমাদের কেক দেখেই বাবা ঘটনা বুঝে গেছে,তাও ভান করেছে যেন কিছুই বুঝতে পারে নি! কী বেইজ্জতি! আচ্ছা আপনার মাথায় এই কেকের বুদ্ধিটা কীভাবে এসেছিল? কেন জানি মনে হচ্ছে আপনি ব্যাপারটা নিজে করেছিলেন কখনো! কেমন করে বানিয়েছিলেন আমাদের কি শিখিয়ে দেবেন? -নাফিস…
উত্তর: আমি আমার মেয়ের জন্মদিনে এটা করেছিলাম, অনেক মজা হয়েছিল। বড় একটা বেলুন ফুলিয়ে মূল কেকটা তৈরি করা হয়েছিল। সুতো দিয়ে বেধে চারকোনা করা হয়েছিল। শেভিং ক্রিম দিয়ে উপরে কেকের মত করা হয়েছিল। আমারটা যথেষ্ট realistic হয়েছিল তাই কেউ বুঝতে পারেনি। (তোমার আব্বু খুবই সুইট মানুষ, বুঝতে পেরেও না বোঝার ভাণ করেছেন!)

প্রশ্ন: স্যার, একটু বলবেন কি সামনের বইমেলায় আপনার কয়টি বই বের হবে? ধন্যবাদ। সাদিয়া,ঢাকা।
উত্তর: এখনো পুরোপুরি জানি না। পাঁচ ছয়টা হয়ে যাবে, তার সবই যে নূতন হবে তা নয়।

প্রশ্ন: আপনার বাসার ছবি দেখতে চাই । ইকরা । পাবনা

উত্তর: দেখ! (আসলে ঠাট্টা করলাম, এটা আমার নিজের বাসা না, এটা আরেকজনের বাসা! কিন্তু বাসাটা সত্যি এরকম!)

প্রশ্ন: আচ্ছা আপনাকে ধরুন কিছু পাঠাতে চাইলাম কিভাবে পাঠাব ? কুরিয়ারে পাঠালে কিভাবে পাঠাব ? ঋতি । ঈশ্বরদি, পাবনা
উত্তর: কি পাঠাবে আগে শুনি। ইলিশ মাছ পাঠালে একরকম, কলম-পেন্সিল পাঠালে অন্যরকম। (আর কয়টা দিন অপেক্ষা কর, আমার একটা স্থায়ী ঠিকানা হয়ে যাক।)

প্রশ্ন: আপনি কি কখনো ঈশ্বরদি তে এসেছেন ? এখানে (আমি যেখানে থাকি) শীতকালে শেয়াল হুয়া হুয়া করে ডাকে আরও অনেক কিছু করে ।যদি কখনো এখানে আসেন আমাকে জানাবেন আমি আপনার সাথে দেখা করব।ঋতি,ঈশ্বরদি ,পাবনা।
উত্তর: ইন্টারেস্টিং। আমি যতদূর জানি শেয়ালদের হুক্কা হুয়া হুক্কা হুয়া করে ডাকার কথা, ঈশ্বরদির শেয়ালেরা শর্টকাটে শুধু হুয়া হুয়া ডাকছে! ব্যাপারটা কি?

প্রশ্ন: আমি একটা গল্প লিখেছি আমি আপনাকে একদিন ইমেলে পাঠাব পছন্দ হলে দয়া করে আপনার ওয়েবসাইটের গল্পের ওখানে দেবেন।ঋতি ঈশ্বরদি ,পাবনা
উত্তর: পাঠাও। কিন্তু আমার ওয়েব সাইটে দেয়া মনে হয় ঠিক হবে না। তখন অন্য সবাই পাঠাতে থাকবে, আমাকে তখন রীতিমত একটা সাহিত্য পত্রিকা ম্যানেজ করতে হবে।

প্রশ্ন: আমার অনেক দিনের ইচ্ছা আপনার সাথে সামনাসামনি দেখা করা কিন্তু আমার সে ইচ্ছা কি কোন দিনও পূরণ হবে ?না কারণ আমাকে তো বাসার বাইরেই যেতে দেওয়া হয় না । আনিকা ।। ঈশ্বরদি ,পাবনা
উত্তর: হবে, তোমার ইচ্ছা একদিন পূরণ হবে। সারা জীবন তো আর তোমাকে ঘরে আটকে রাখতে পারবে না।

প্রশ্ন: শান্তা পরিবার বইটিতে আপনি বেলুন দিয়ে তৈরি কেক দিয়েছিলেন সেটা সাগর বানিয়েছিল আমি বানানোর চেষ্টা করেছি কিন্তু পারিনি কিভাবে বানাবো বলবেন একটু নাম:ঋতি ঠিকানা :ঈশ্বরদি ‘,পাবনা
উত্তর: গল্পের কাহিনী সত্যি কেমন করে বুঝলে? এবারের চিঠির উত্তরে আমি আরেকজনকে ঠিক এটা লিখেছি। ওখান থেকে পড়ে নাও প্লিজ।

প্রশ্ন: স্যার,আমি আগামী BDIO তে পার্টিসিপেট করতে চাই। কিন্তু কীভাবে করব বুঝতে পারছি না। গুগলএ সার্চ দিয়ে ও তেমন কিছু পেলাম না। আশেপাশের কেউ বলতে পারলো না কী করতে হবে । আপনি বলবেনপ্লিজ কী করতে হবে! প্রশুন, সিলেট
উত্তর: Bangladesh Informatics Olympiad লিখে সার্চ দাও, কিছু একটা পেয়ে যাবে।

প্রশ্ন: Sir apnar priyo kheloar ke? Hasin,Dhaka.
উত্তর: হা, হা, হা! আমি আসলে খেলা ধূলার এক্সপার্ট না। বাংলাদেশের যে কোনো প্লেয়ার একটু ভালো খেললেই আমি তার ভক্ত হয়ে যাই।

প্রশ্ন: অনেকদিন দেখা সাক্ষাৎ নেই ????????????? দিলিপ তানভির ?????????????????????????????????????????????????????????????????????????????????????????????????????????????????????????????????????????????????????????????????????????????????????????????????????????????????????????????????????????
উত্তর: তাই না কি? আশ্চর্য!!!!!!!!!!!!!!!!!!!!!!!!!!!!!!!!!!!!!!!!!!!!!!!!!!!!!!!!!!!!!!!!!!!!!!!!!!!!!!!!!!!!!!!!!!!!!!!!!!!!!!!!!!!!!!!!!!!!!!!!!!!!!!!!!!!!!!!!!!!!!!!!!!!!!!!!!!!!!!!!!!!!!!!!!!!!!!!!!!!!!!!!!!!!!!!!!!!

প্রশ্ন: আমি কিন্তু আপনাকে আংকেল বলেই ডাকবো সবসময়। আর আপনি চিঠিতে নাকি ইমেইলে বেশি রিপ্লাই দেন? ঢাকা থেকে ইশতিয়াক
উত্তর: যেটা ইচ্ছা ডাকো, কোনো সমস্যা নেই। আমি দুটোই উত্তর দেবার চেষ্টা করি। কোনটা পারি সেটা ভিন্ন ব্যাপার।

প্রশ্ন: রাজবাড়ি,ঢাকা। স্যার আর নতুন গল্প দিচ্ছেন না কেন? আর শিশুতোষ গ্রন্থ থেকে দেবেন। আপনার প্রায় সব বই পড়ে শেষ করে ফেলেছি। হুমায়ূন আহমেদ স্যারের সব বই শেষ হওয়া অবস্থা।
উত্তর: শিশুতোষে আপত্তি না থাকলে এটা দেখতে পার। আজকে একটা দিলাম।

প্রশ্ন: নমস্কার, জাফর দাদু। আমি আদ্রিতা , সুনামগঞ্জ। আপনার লেখা বইগুলোর মধ্যে আপনার সবচেয়ে পছন্দের বই কোনটা? আর হুমায়ূন স্যার এর লেখা বইগুলোর মধ্যে আপনার সবচেয়ে পছন্দের বই কোনটা?
উত্তর: নিজের লেখার ভালো মন্দ খুঁজে বের করা খুব কঠিন। এবারের চিঠির উত্তর দেবার সময় দেখ একজনকে কয়েকটা বইয়ের নাম লিখেছি। হুমায়ূন আহমেদের অনেক বই আমার পছন্দ, কোনটা ছেড়ে কোনটা বলব? তার একেবারে প্রথম বইটার প্রথম পাঠক ছিলাম আমি। সেই বইটার জন্য আমার অন্য রকম একটা ভালোবাসা আছে।

প্রশ্ন: আমি আর্টিফিশিয়াল ইন্টেলিজেন্স শিখতে চাই। মোটামুটি প্রোগ্রামিং পারি। কোন বইটা পড়ব(বাংলা বা ইংরেজি যে কোনোভাষার বই সাজেস্ট করতে পারেন) প্রত্যয়, দশ্ম শ্রেণী,সিলেট
উত্তর: দ্বিমিক প্রকাশনী অনেক সুন্দর প্রোগ্রামিংয়ের বই বের করে। খোঁজ নিয়ে দেখ তাদের আর্টিফিশিয়াল ইন্টেলিজেন্সএর উপর কোনো বই আছে কি না।

প্রশ্ন: স্যার, কাকে আপনার বেশি ভাল লাগে?? হিমু না মিসির আলি?? আবিদ, টাঙ্গাইল
উত্তর: বেশি ভালো লাগে যে এই দুজনকে তৈরি করেছে তাকে!

প্রশ্ন: Sir, dorkhasto lekhar shomoy marjin tante hoy na kano??? Saikot Kushtia
উত্তর: তাই না কি? জানতাম না তো। তার মানে জীবনে কত ভুল দরখাস্ত লিখে এসেছি। এখন কি হবে?

প্রশ্ন: সার আমার চিঠিটা খুব বড় ও impoortant . মজা ভেবে এরিয়া যাবেন না ।পুরোটা পড়বেন। অনেক কষ্ট করে লিখা । যদিও আমার কি বোর্ড টা নষ্ট। আপনিতো শুধু পালিয়ে যান । আমি শুনেছি আপনি নাকি আপনার পরিচিত কাওকে ছাড়া বাকিদের উত্তর দেন না। আমি কি ঠিক শুনেছি ? কিছু মনে করবেন না জা4 ইকবাল সার । আসল কথায় আসি। আমি কিছুদিন আগে নাম ঠিকানা সহ একটি সহজ প্রশ্ন করেছিলাম যে টুনটুনি ও ছোটাচ্ছু এর নতুন বইের নাম কি? কবে বের হবে ? আর ইয়েশিম আপুর বাচ্চার নাম কি ? আচ্ছা সার আপনি তো আপনার চুলে কলপ করতে পারেন ( just মজা করলাম) । আগেই বলেছি আমার চিঠিটা বড়ো। সার ২০২০ এ বইমেলাতে আপনার কি কি বই বের হচ্ছে ? এগুলো শুধু আমার না আমাদের ক্লাস আর সব বন্ধুদের প্রশ্ন ।তারা আমার কাছেই তাদের প্রশ্ন নিয়ে আসে(যদিও আমার কি বোর্ড টা নষ্ট ) কারন ”I AM A BIGGEST FAN OF YOU DR. MUHAMMAD ZAFOR IQBAL SIR . WE ARE VERY VERY LOVE YOU AND YOU FAMILY ” please please please please please please please please please please please please উত্তর দেবেন । মুঞ্জির কাদের class 04, ভিকারুন্নেসা নুন স্কুল অ্যান্ড কলেজ ।ফার্মগেট, ঢাকা
উত্তর: টুনটুনি এবং ছোটাচ্চুর নুতন বই লিখতে পারব কি না এখনো জানি না। যা যা লিখতে চেয়েছিলাম সব লিখে ফেলেছি! যদি  সত্যিই লিখতে পারি তাহলে বই মেলায় বের হবে। তোমার সাথে যখন আমার দেখা হবে তখন তুমি আমার কাছ থেকে ইয়েশিমের বাচ্চার নামটা জেনে নিও। (আমি নিজেদের কথা এখানে লিখতে চাই না! যারা পড়বে তারা বিরক্ত হয়ে বলবে, তোমার নাতির নাম জেনে আমার কি লাভ?) চুলে কলপ? তুমি যদি রেগুলার করে দাও তাহলে করতে পারি। আর চুল যখন একটু বড় হবে, গোড়াটা সাদা হয়ে থাকবে, তখনো কিন্তু এসে কলপ করে দিতে হবে। রাজী? ২০২০ এর বইমেলায় আমার দুটি সায়েন্স ফিকশান বের হবে। একটার নাম প্রজেক্ট আকাশলীন অন্যটার নাম গ্লিনা। অন্য বইগুলো এখনো ঠিক হয়নি। লেখার চেষ্টা করছি, দোয়া করো যেন লিখে শেষ করতে পারি।

প্রশ্ন: স্যার আমি ঢাবিতে ফিজিক্স অনার্সে ভর্তিও হয়েছি, সেকেন্ড ইয়ারে আছি। ছোটবেলা থেকেই স্বপ্ন ছিল ফিজিক্স নিয়ে কাজ করার। কিন্তু এখানে এসে দেখছি, ফিজিক্সেও অনেক কিছু না বুঝে মুখস্থ করতে হচ্ছে। তাছাড়া বিশ্ববিদ্যালয়ের পড়ানোর সিস্টেমটাই আমার পছন্দ হচ্ছে না, বিস্তারিত লিখছি না। তো কিছু কারণে আমার সিজিপিএ বেশ কমে গেছে। তাই আমি উদ্বিগ্ন, আমার ভবিষ্যত নিয়ে। যদি রেজাল্ট ভালো না থাকে, তবে কি রিসার্চ ফিল্ডে সুযোগ করা যাবে? পরামর্শ চাইছি। ফাহিম, ঢাকা বিশ্ববিদ্যালয়।
উত্তর: আমি ঠিক বুঝতে পারলাম না, না বুঝে মুখস্ত করতে হচ্ছে মানে কি? যতক্ষণ বুঝতে না পারছ বোঝার চেষ্টা করতে থাকো। যে মুহুর্তে তুমি না বুঝে কিছু মুখস্ত করবে বুঝতে পারবে, তোমার পড়া লেখা সেই মুহুর্তে শেষ!

প্রশ্ন: Shahreen khan Taan, Bashundhara residential area, dhaka.
একটা ভক্ত লিখেছে যে সাকিব, মিরাজের সাথে ছবি তুলতে পারায় আপনার ভাগ্য ভালো। That made me laugh. ‘Cause it’s actually the other way around. সাকিব, মিরাজের ভাগ্য ভালো যে ওরা আপনার সাথে ছবি তুলতে পেরেছে। সেই ভাগ্য আমার হলো না এখনো। কোথায় পাই আপনাকে, বলেন তো? আপনাকে কেউ আবার ছুরিকাঘাত করার আগে ছবি তুলাটা অত্যন্ত জরুরি। কোথায় পাই আপনাকে বলেন।
উত্তর: আমি কিন্তু মনে করি এটা আমার অনেক বড় সৌভাগ্য। আমাকে পেতে চাইছ? পাবে, পাবে! চোখ কান খোলা রেখো দেখবে আমি হঠাৎ করে হাজির হয়ে গেছি! ছুরিকাঘাতের উপর অবশ্য আমার কোনো হাত নেই, তারা কখন কোথা থেকে হাজির হবে কোনো গ্যারান্টি নেই।

প্রশ্ন: প্রিয় স্যার, আপনার নিকট অনুরোধ জীবিত থাকা অবস্থায় অন্তত একবার হলেও কোন স্কুলে পদার্থবিজ্ঞানের ক্লাস নিবেন। সাধারনত কোন পি এইচ ডি করা পদার্থবিজ্ঞানের ছাত্র স্কুলের ক্লাস নিতে চায় না। আপনি যদি নেন তাহলে অনেকে উৎসাহ পাবে। আমার পি এইচ ডি করতে করতে অনেক বছর পার হয়ে যাবে। কিন্তু আপনার তো অনেক সময় আছে তাই না? কোন অযুহাত দিবেন না, প্লিজ নিবেদক মো: আশরাফুল ইসলাম বেড়িবাধ, মাটিকাটা, ঢাকা, বাংলাদেশ
উত্তর: আমার কোনো আপত্তি নেই, কিন্তু হঠাৎ করে আমি যদি একটা স্কুলে ঢুকে পড়াতে শুরু করি দারোয়ান নিশ্চয়ই পাগল মনে করে পুলিশ ডেকে আনবে।

প্রশ্ন: স্যার এখন কিন্তু অনেকেই মিথ্যা নাম দিয়ে প্রশ্ন করবে। সামান্তা
উত্তর: করুক! অন্তত কোনো একটা নাম তো লিখবে।

প্রশ্ন: স্যার ২৫ই মার্চের পর কি আপনি ময়মনসিংহ এসেছিলেন আরও???আবার আসলে আমি কোনভাবেই মিস করতে চাই না। স্রিজন, ময়মনসিংহ
উত্তর: হ্যাঁ এসেছিলাম, মনে হয় আবার মিস হয়ে গেল। পরের বার নিশ্চয়ই দেখা হবে।

প্রশ্ন: স্যার দেশটাতে কি হচ্ছে এখন??আমার দম বন্ধ হয়ে আসছে। অবন্তি, ময়মনসিংহ
উত্তর: এতো তাড়াতাড়ি দম বন্ধ হলে চলবে কেমন করে?

প্রশ্ন: স্যার যারাই ঠিকানা দিচ্ছে তাদেরই আপনি অটোগ্রাফ দিয়ে দিচ্ছেন। এটা ঠিক না।দিয়েন না স্যার নৌশিন, চাঁদপুর
উত্তর: আমি দিতে চাচ্ছি, এখনো দেওয়া শুরু করতে পারিনি।

প্রশ্ন: বয়স হবার সাথে আগ্রহের পরিধি কমে আসে, অবসাদ ও ক্লান্তি বেশি গ্রাস করে । বৈষয়িক ভাবনা বেড়ে যায়। মৃত্যুর দিকে আগাচ্ছি বলে সব অর্থহীন মনে হয়। কীভাবে শারীরিক ও মানসিকভাবে কৈশোরকে ধরে রাখা যায় ? জুবায়ের, চট্টগ্রাম।
উত্তর: তুমি যদি এর উত্তরটা খুঁজে বের করতে পার আমাকে জানিও। আমার জন্য খুবই দরকার, বয়স প্রায় সাতষট্টি হয়ে গেল।

প্রশ্ন: জা4 ইকবাল সার, টুনটুনি ও ছোটাচ্চু সিরিজের পরের বইটির নাম কি ? মোজামmale হক, দিলালপুর, পাবনা
উত্তর: aখno  tক হয়নি!

প্রশ্ন: Pollob1563 Sylhet 1+2+3+4+Infinity=-(1/12) প্রমাণ চাই।।
উত্তর: 1+2+3+4+Infinity=-(1/12) লিখে ইন্টারনেটে সার্চ দাও।

প্রশ্ন: স্যার, আমার এক বান্ধবীর ……… এখন ও খালি বিষন্নতায় ভুগে – নাকি ওর খালি আত্মহত্যা করতে ইচ্ছা করে, কিন্তু সাহস হয় না । যতই বলি এসব ফালতু চিন্তা – জীবনে …… আসবে যাবে – কিছুতেই বিশ্বাস করে না। কি করা যায় বলুন তো ? Animesh, age 25
উত্তর: একটু সময় দাও। কিছু প্রতিষ্ঠান আছে যারা এসব ব্যাপারে সাহায্য করে। তাদের সঙ্গে যোগাযোগ করিয়ে দাও।

প্রশ্ন: আপনার অলটাইম ফেভারিট বিশ্বসাহিত্যের কয়েকটা গল্পগ্রন্থ, উপন্যাস আর ভ্রমণকাহিনির বইয়ের নাম বলবেন প্লিজ? ইশতিয়াক, ঢাকা
উত্তর: এবারেই আরেকজনকে তার উত্তর দিয়েছি, একটু কষ্ট করে সেটা পড়ে নেবে প্লীজ!

১১ সেপ্টেম্বর থেকে ১৩ অক্টোবর, ২০১৯

প্রশ্ন: ৬-৭ মাস হয়ে গেল কোন নতুন গল্প নেই।নতুন গল্প কখন দিবেন ?
**নাম ঠিকানা নেই, তাই উত্তরও নেই! সরি।

প্রশ্ন: স্যার, আমি বড় হয়ে একজন পদার্থবিজ্ঞানী হতে চাই। কিন্তু আমার বাবা মা চায় আমি যেন ইঞ্জিনিয়ার হই। তাই তারা চায় আমি যেন বুয়েটে এডমিশনের চেষ্টা করি। একদম আপনার লেখা “টুনটুনি ও ছোটাচ্চু” সিরিজের সুমন চরিত্রের মতো৷ এখন আমার প্রশ্ন হলো আমি যদি পদার্থবিজ্ঞানী হতে চাই তাহলে আমার লক্ষ্য কি হওয়া উচিত ঢাবি নাকি বুয়েট? বুয়েট থেকে পড়ে কি পদার্থবিজ্ঞানী হওয়া যায়। আমার আব্বু বলছে বুয়েট থেকে পড়েও নাকি পরবর্তীতে বিজ্ঞানী হওয়া যায়। ( আমিও প্রথমে তা বিশ্বাস করেছিলাম, কিন্তু আপনার লেখায় সুমনের কাহিনী পড়ে আমার সন্দেহ হচ্ছে যে বুয়েট শুধুই ইঞ্জিনিয়ারদের জন্য[যেহেতু এটা ইঞ্জিনিয়ারিং ইউনিভার্সিটি]) তাই স্যার আপনার কাছে জানতে চাচ্ছি। বুয়েট থেকে পড়ে পদার্থবিজ্ঞানী হওয়া যায় কিনা? -তানভীর আহাম্মেদ লিওন, গাজীপুর
উত্তর: আমি অন্তত একজনকে জানি, যে দেশে ইলেকট্রিক্যাল ইঞ্জিনিয়ারিং পড়ে বাইরে থেকে পদার্থ বিজ্ঞানে পি এইচ ডি করেছে। কিন্তু সেটা করার জন্য তার যেটুকু আন্তরিক পরিশ্রম ছিল, সেটা সবার থাকে না। পদার্থ বিজ্ঞানী হতে চাইলে পদার্থ বিজ্ঞান পড়বে সেটাই স্বাভাবিক।

প্রশ্ন: স্যার আমি ক্লাস টেনে পড়ি, কাজ চালানোর মতো ম্যাথমেটিক্স, জাভা আর সি প্লাস প্লাস প্রোগ্রামিং পারি । আমি আর্টিফিশিয়াল ইন্টেলিজেন্স শিখতে চাই, এই ব্যাপারে আমি কিছুই জানি না। কোন বই টা পড়া আমার জন্য ভালো হবে( আমি সোফিয়ার মতো রবোট বানাবো একদিন, দেখে নিয়েন!)
**নাম ঠিকানা নেই, তাই উত্তরও নেই! সরি।

প্রশ্ন: টুনটুনি ও ছোটাচ্চু সিরিজেে যখন টুনটুনি তখন ছোটাচ্চু এর পরের বইয়ের নাম ফিরে এলো টুনটুনি ও ছোটাচ্চু
**নাম ঠিকানা নেই, তাই উত্তরও নেই! সরি।

প্রশ্ন: Sir, “hat kata Robin,amar bondhu Rashed,sho’te Sentu” er moto upanyash amar khub bhalo lage.Tai ami chai apni erokom aro upanyash lekhen.Ar Sir please apni amake utshorg kore ekta upanyash lekhen. Hasin,Dhaka.
উত্তর: লিখতে চেষ্টা করব। উৎসর্গের ব্যাপারটা একটু ভিন্ন। যে আমার জীবনে খানিকটা অবদান রেখেছে, আমি শুধু তাদেরকেই বই উৎসর্গ করে আসছি।

প্রশ্ন: হাসিন,ঢাকাI আপনি সাকিব,মিরাজ,মাশরাফির সাথে ছবি তুলেছেন দেখে হিংসে হচ্ছে! কিন্তু আমিও সাব্বির ও তাসকিনের সাথে ছবি তুলেছিI
উত্তর: কংগ্র্যাচুলেশন্স! আর হিংসার কিছু নেই, লেগে থাক, তুমিও পারবে আমার মত।

প্রশ্ন: Dhaka y robotics shikar jonno maslab ase but amdr Sylhet shikar joono ki ase???
**নাম ঠিকানা নেই, তাই উত্তরও নেই! সরি।

প্রশ্ন: স্যার, আজ আমি নিম্ন মাধ্যমিক শিক্ষা স্ত্ররের কিছু বিষয় আপনার সামনে আনব। আপনি বলবেন আমার ধারণা সঠিক না ভুল।
১.অষ্টম শ্রেণীর পাঠ্যপুস্তকের সংখ্যা ১৪ টি। কিন্তু পরিক্ষা হয় ৭ টি। যেমন :
(ক) বাংলা। (সাহিত্য কনিকা ও ব্যাকরণ)
(খ)ইংরেজি ( 1st and 2nd)
(গ)গনিত
(ঘ)বিজ্ঞান
(ঙ)বাংলাদেশ ও বিশ্বপরিচয়
(চ) ধর্ম
(ছ) আইসিটি।
বাকি বিষয় গুলো ধারাবাহিক মূল্যায়ন করার কথা। কিন্তু আদৌ কি তা করা হয়। না স্যার, হয় না। আমি জানি স্যার কারন, আমি ৮ম শ্রেণীর ছাত্র। এ বিষয় গুলোতে আমাদেরকে এমনিতেই ৯৫% – ৯৮% নাম্বার দেয়া হয় । তাহলে স্যার আমার প্রশ্ন, এই বই গুলো কেনো দেয়া হয়? আর স্যার, এই অনিয়ম গুলোর দায় কে নেবে? স্যার, আজ আর লিখলাম না। অন্য বিষয় গুলো পরে লিখব।( প্লিজ স্যার, এর উত্তর দেবেন।)
Email :sagorreza1111@gmail.com
উত্তর: শুধু পড়ে শেখার জন্য বই দেওয়া হয়েছে, কিন্তু কোনো পরিক্ষা হচ্ছে না সেটা নিয়ে আমার কোনো আপত্তি নেই। কিন্তু সত্যিকারের ধারাবাহিক মূল্যায়ন না করে ঢালাও ভাবে সবাইকে দুই হাতে মার্কস দেয়া হলে আমার আপত্তি আছে।

প্রশ্ন: স্যার, সামনে আমার JSC পরিক্ষা । তাই আমার জন্য একটু দোয়া রাখবেন।
উত্তর: নাম ঠিকানা নেই, তাই উত্তরও না দেয়ার কথা ছিল, কিন্তু তুমি যেহেতু দোয়া চেয়েছ তাই দোয়া করে দিচ্ছি। দেখবে তোমার অনেক ভালো পরীক্ষা হবে।

প্রশ্ন: Shahreen Khan Taan10th grade Bashundhara residential area, Dhaka.
স্যার, আপনার এক ছাত্র আজ আমাদের বাসায় বেড়াতে এসেছিল। তার সাথে কথা হচ্ছিল আপনাকে নিয়ে। সে আপনাকে attention seeker বলেছে। আমি সাথে সাথে তার এই উক্তির বিরোধিতা করি এবং জিজ্ঞেস করি একটা ঘটনা বলতে যেটা থেকে মনে হতে পারে আপনি attention seeker. তখন তার সাথে হওয়া আমার কথোপকথনটি তুলে ধরছিঃ
সেঃ অসংখ্য অনুষ্ঠানে স্যার কে প্রধান অতিথি হিসেবে দাওয়াত দেওয়া হয়। বলো দেখি প্রথম কোন criterion সেই invitation satify করলে স্যার তা গ্রহণ করেন?
আমিঃ Umm.. ঠিক জানি না।
সেঃ যে অনুষ্ঠানে মিডিয়া কভারেজ বেশি থাকে স্যার শুধু সেই অনুষ্টানেই যান। তুমি নিজেই স্যার কে প্রশ্ন করতে পার যে স্যার, কোন অনুষ্ঠানে যাওয়ার আগে আপনি সে কোন জিনিসগুলো বিবেচনা করেন। নিজেই বলার কথা। End of conversation এখন আমার প্রিশ্ন, সে যা বলেছে তা কি সত্য? সত্য হলে তার কারণ জানিতে চাই
উত্তর: হা হা হা! ইন্টারেস্টিং! নিজের সম্পর্কে আরাকটা তথ্য জানলাম। কী মজা। (কারনটা তো সে বলেই দিয়েছে, আমি নূতন করে আর কী বলব?)

প্রশ্ন: আমি এস্ট্রোনমি নিয়ে পড়তে চাই । কিন্তু কেউ সাপোর্ট দেয় না !
**নাম ঠিকানা নেই, তাই উত্তরও নেই! সরি।

প্রশ্ন: স্যার আপনাকে একটা প্রশ্ন করব ?
**নাম ঠিকানা নেই, তাই উত্তরও নেই! সরি।

প্রশ্ন: Sir.I want to be more big the physicist then Albert Einstein. And now want to read in MIT… What I can do? -Kazi Abu Talha. Konabari,Gazipur,Dhaka.
উত্তর: তুমি যদি বলতে আমি খুব ভালো পদার্থ বিজ্ঞানী হতে চাই, তাহলে একটা কিছু উত্তর দিতে পারতাম। কিন্তু তুমি যেহেতু আইনস্টাইন থেকে বড় বিজ্ঞানী হতে চাও আমি ঠিক কী বলব বুঝতে পারছি না। তবে এরকম জীবনের লক্ষ থাকা মনে হয় ঠিক না যে আমি অমুক থেকে বড় হব। জীবনের লক্ষ থাকতে হয় যে জীবনে সুখী হব, ভালো মানুষ হব, সবাইকে নিয়ে অনেক আনন্দ করব এরকম। আর এম আই টিতে ভর্তি হতে হলে ওদের নিয়ম মেনে আবেদন করতে হয়।

প্রশ্ন: স্যাৱ আপনাৱ ইমেইল ঠিকানাটা একটু দেয়া যাবে? – ৱোদেলা, ঢাকা
উত্তর: আমাদের ইউনিভার্সিটির ওয়েবসাইটে ঢুকে বের করে নাও প্লীজ। আমি এখানে সরাসরি দিতে চাই না।

প্রশ্ন: স্যার আমার নাম মোস্তফা ।আমি ৯ম শ্রেণীতে পড়ি । যখন আমি আমার নাম নিয়ে চিন্তা করি তখন আমার মনে হয় যে আমি খুবই গম্ভীর কেউ । কিন্তু প্রকৃতপক্ষে আমি একজন হাসি খুসি মানুষ (সবাই ত তাইই বলে) ।আপনার ওকি তাই মনে হয় ?উত্তর না দিলেও হবে (কারন আপনি আমার এই পর্যন্ত ২ টি প্রশ্নের উত্তর দিয়েছেন , উত্তর দিলে খুশি হব কিন্তু না দিলে দুঃখ পাব না ) বিঃদ্রঃ স্যার আপনার সাথে কি কখনো এমন হয়েছে যে , একজনের নাম বা চেহারা দেখে তার সম্পর্কে যা ধারণা হয়েছে পরবর্তীতে দেখেছেন যে তা ভুল ?
উত্তর: নাম বা চেহারা দেখে সাধারনত মানুষটার সম্পর্কে আমি ধারনা করতে পারি না। তবে তুমি খুব হাসিখুশি মানুষ জেনে আমি খুব খুশী হলাম। হাসিখুশী থেকে যাও সারা জীবন।

প্রশ্ন: প্রথমেই ধন্যবাদ বিজ্ঞানের সব চমৎকার বই লিখছেন ইদানিং, এই ছোট্ট একটি কাজ করে আপনি যে কত বিড়াট ভূমিকা রাখছেন তা হয়ত নিজেও টের পাচ্ছেন না। মনে পড়ে, ‘একটুখানি বিজ্ঞান’ পড়ে এতোটাই ভাল লেগেছিল যে, তারপর থেকে বিজ্ঞান ভালবাসার নাম হয়ে দাঁড়ায়; খুবই আশ্চার্যিত ও ভাললাগা কাজ করেছিল বইটি পড়ে। অবশ্যই আরো লিখে যাবেন। আপনার মাঝে কোন কিছু সহজ কথায় বোঝানোর এক চমৎকার, অদ্বিতীয় গুণ রয়েছে।
আমার প্রশ্নটি হল, আপনি এত সুন্দর গদ্যে লেখার পেছনে কিসের ভূমিকা আছে বলে মনে করেন? কোন বইগুলো বা লেখক বাংলা এমন গদ্যে লিখতে আপনাকে অনুপ্রাণিত করেছে বলে মনে করেন? (আমি জানি আপনি বলতে পারেন instinct, তবু একটূ systematical approach এ গিয়ে বোঝার চেষ্টা করা ত ক্ষতির নয়।)
Julian Jawad Ahmad, Uttara, Dhaka.
উত্তর: আমার গদ্যকে সুন্দর বলার জন্য থ্যাংকু। আমার মনে হয় না আমার লেখার স্টাইল ঠিক কোনো নির্দিষ্ট লেখকের অনুপ্রেরণায় হয়েছে, লিখতে গিয়ে দেখি এভাবে ছাড়া অন্যভাবে লিখতে পারি না। তবে বই পড়ার সাথে এর একটা সম্পর্ক আছে। যে যত বই পড়ে লেখা ব্যাপারটা তার জন্য তত সোজা।

প্রশ্ন: Sir,আমি আপনার ৫০টিরও বেশি boi (laptop e boi লিখতে পারছি না) পড়েছিI কিনতু রুহান রুহান boiটি আমি পেলাম নাI ‘boi’তে দিলে পড়তে পারি I
**নাম ঠিকানা নেই, তাই উত্তরও নেই! সরি।

প্রশ্ন: স্যার,চোর পালালে বুদ্ধি বাড়ে কিন্তু গৃহী পালালে?
**নাম ঠিকানা নেই, তাই উত্তরও নেই! সরি।

প্রশ্ন: প্লিজ, ইয়েলের জন মারকি সম্পর্কে কিছু বলেন, যার ব্যাপারে দেশের বাইরে দেশ বইয়ের ৯২ পৃষ্ঠায় বলেছেন, কেন সে অসৎ বিজ্ঞানী? মুরতাজা কামাল পাশা খান, এম এস ইন ইই ক্যান্ডিডেট, ইউনিভারসিটি অফ মিনেসোটা ডুলুথ
উত্তর: হা হা হা! একজন মানুষ সম্পর্কে খারাপ কিছু না লিখি। সে খুব মজার একটা চরিত্র ছিল সেটা বলা যেতে পারে।

প্রশ্ন: Sir, ami Ruhan Ruhan boita paini.Tai ‘golpo’ er modhya dile bhalo hoe. eti-Hasin
উত্তর: আমি যতদূর জানি, আমার বইগুলো নেটে পাওয়া যায়। আরো একটু খোঁজ কর। আমি নিজে বসে বসে এখন আর পিডিএফ করতে পারব না, যদি চোখে পড়ে দিয়ে দেব। (তুমি কী জান, ঢাকা কমিক্স এটার কমিক বের করেছে? খুব সুন্দর ছবি!)

প্রশ্ন: 1+2+3+4+Infinity=-(1/12) প্রমাণ চাই।।
**নাম ঠিকানা নেই, তাই উত্তরও নেই! সরি।

প্রশ্ন: আচ্ছা আপনাকে কেউ বলে নাই, আপনার লেখা এখন আর ভালো হয় না? কত বছর হয়ে গেল, আগের মতো লেখা পাই না। আমি তপু বা দীপু নাম্বার টু পড়লে এখনো কী অসাধারণ লাগে! এখনকার লেখা ওরকম সুন্দর না হোক, কাছাকাছি তো হবে। তা তো হয়ই না বরং দিনকে দিন আরো খারাপ হচ্ছে।………
**নাম ঠিকানা নেই, তাই উত্তরও নেই! সরি। (চিঠির অমার্জিত অংশটা কেটে দিলাম)

প্রশ্ন: Kmn achen
**নাম ঠিকানা নেই, তাই উত্তরও নেই! সরি।

প্রশ্ন: স্যার, এই পৃথিবীতে সবারই একজন প্রিয় ব্যক্তিত্ব আছে,,,,,,তাই না???? আমারো আছে।।।।আর সেটা হলো আপনি ।।।।।।।।। কিন্তু স্যার,,, আপনার প্রিয় ব্যক্তিত্ব কে???????????? নাহিয়ান, কুষ্টিয়া।।।।।।।।।
উত্তর: তার আগে বল, তোমার লেখায় দাড়ি, কমা আর প্রশ্নবোধক চিহ্ন এত বেশী কেন?

প্রশ্ন: স্যার,মেকুর ছয় মাস বয়সে কী হয়েছিল? নিশাত তাসনিম ঐশী,চতুর্থ শ্রেণি,ফুলপুর,ময়মনসিংহ
উত্তর: হায় হায়! এতো বছর আগের ঘটনা এখন কী আর মনে আছে? কিছু একটা কল্পনা করে নাও। আমাকে জানাতেও পার ইচ্ছা করলে।

প্রশ্ন: স্যার আমি যদি দুইা ভিপিএন একসাথে ব্যবহার করি তাহলে আমার আইপি অ্যাড্রেস কি দুইটা হবে ?
**নাম ঠিকানা নেই, তাই উত্তরও নেই! সরি।

প্রশ্ন: স্যার, আপনি বলেছিলেন আপনার নিরিবিলি জায়গা পছন্দ যেখানে মানুষ হারিকেন দিয়ে পড়ে, শেয়ালের ডাক শোনা যায়। এরকম একটা জায়গায় আছে সিলেটের খুব কাছেই। হবিগঞ্জ এর রেমা কালেঙ্গা বন্য প্রাণী অভয়ারণ্য। সেখানে মানুষ রেডিও শোনে। বিদ্যুৎ নেই। মনে হয় ১৯৮০ এর দশকে বসে আছি। আপনি যাবেন? শুভ চন্দ্র দাস বঙ্গবন্ধু শেখ মুজিবুর রহমান হল, সাস্ট
উত্তর: নিশ্চয়ই যাব। আমাকে জানানোর জন্য তোমাকে অনেক ধন্যবাদ।

প্রশ্ন: স্যার, জ্বর হলে আমাদের শীত লাগে কেন? নামঃপিয়া দে
উত্তর: এখানে দেখ। https://www.everydayhealth.com/cold-and-flu/why-do-we-get-chills-with-a-fever/ আমি চাই তোমরা সবাই নেটে তোমাদের বেশীরভাগ প্রশ্নের উত্তর খুঁজে বের করা শিখে নাও।

প্রশ্ন: আপনি জীবনে এতো বই লিখেছে। আপনার লেখা বইগুলোর মধ্যে আপনার সবচেয়ে পছন্দের বই কোনটা?
**নাম ঠিকানা নেই, তাই উত্তরও নেই! সরি।

প্রশ্ন: স্যার, হুমায়ূন স্যারের বই থেকে বানানো ‘দেবী’ সিনেমাটা নিশ্চয়ই দেখেছিলেন, তাই না? আপনার অনুভূতি কেমন ছিল? আর হুমায়ূন স্যার সিনেমাটি দেখলে কি বলতেন বলে আপনার মনে হয়?
**নাম ঠিকানা নেই, তাই উত্তরও নেই! সরি।

প্রশ্ন: স্যার, হুমায়ূন আহমেদ স্যারের সৃষ্ট কোন চরিত্রটি আপনাকে বেশি আকর্ষণ করে, হিমু না মিসির আলি??
**নাম ঠিকানা নেই, তাই উত্তরও নেই! সরি।

প্রশ্ন: সোনিয়া, শাহজাহানপুর
অনেকদিন পর লিখছি, এতো এতো প্রশ্নের ভীরে মনে হল আমি আর প্রশ্ন নাহ করি কিন্তু একটা অনুরোধ করি। আমার ১৭ সেপ্টেম্বর জন্মদিন ছিল, আপনি কি একটা কাগজে শুভ জন্মদিন সোনিয়া লিখে অটোগ্রাফ দিয়ে দিবেন?? তবে সেটা আমার জীবনের অন্যতম উপহার হতো। একটু কি বাচ্চামো হয়ে গেলো? তবে আমি বাচ্চাই থাকতে চাই…
উত্তর: সরি সোনিয়া, ১৭ সেপ্টেম্বর পার হয়ে গেছে আর তোমার জন্মদিনের শুভেচ্ছা জানানো হল না। তোমার ঠিকানা জানালে বিলম্বিত জন্মদিনের শুভেচ্ছা জানাতে পারি। আর বাচ্চামোর কথা বলছ? তোমারা যদি বাচ্চা হতে ভয় পাও তাহলে আমার অবস্থা কী হবে?

প্রশ্ন: স্যার , নিষাত,নিনিতের সাথে কি আপনার কথা হয় ??ওদের জন্য আমার অনেক মায়া হয়। স্মৃতি ।
উত্তর: না, নিয়মিত হয় না। হ্যাঁ, সবারই এই দুইজনের জন্য এক ধরনের ভালোবাসা আছে।

প্রশ্ন: স্যার গতিশিল আধান তার আশাপাশে চৌম্বকক্ষেত্র সৃষ্টি করে,স্যার এটার কারনটা কী??স্যার এখানে যদি চার্জের গতিশিলতা দায়ী করে তবে তো সেখানে ঘর্ষনের ফলে তাপ উৎপন্ন হবে। স্যার যদি চার্জের গতিশিলতা দায়ী না করি তবে স্থির আধান কেন চৌম্বকক্ষেত্র সৃষ্টি করে না………?? মো,সুজন
উত্তর: গতিশীল চার্জের কারনে চোম্বক ক্ষেত্র তৈরি হওয়ার মাঝে ঘর্ষণ বা তাপের কোনো ভুমিকা নেই। চার্জ গতিশীল হলে কেন চোম্বক ক্ষেত্র তৈরী হয় বুঝতে হলে থিওরি অফ রিলেটিভিটির দরকার হয়। এই যে নেটে এটা দেখে নাও। https://physics.stackexchange.com/questions/65335/how-do-moving-charges-produce-magnetic-fields

প্রশ্ন: sir, আপনার প্রিয় ঔপন্যাসিক কে ? এবং আপনার প্রিয় উপন্যাস কোনটি ? দিশা, নবম শ্রেণী সরকারি এস সি বালিকা উচ্চ বিদ্যালয়
উত্তর: অনেক অনেক অনেক। কয়জনের নাম বলব? একজন হচ্ছেন জন স্টেনবেইক, বইয়ের নাম ইস্ট অফ ইডেন।

   

প্রশ্ন: গাছের তো খাদ্য তৈরিতে কার্বন ডাই অকাইড, পানি,সূর্যের আলো ব্যাবহার করে আর মানুষ চাল-ডাল এসব দিয়ে।তাহলে গাছ কি করে নিজের খাদ্য নিজে তৈরি করে আর মানুষ নিজেরখাদ্য নিজে তৈরি করে না?(একটু কষ্ট করে “বাচ্চা ভয়ঙ্কর কাচ্চা ভয়ঙ্কর” বইটা ওয়েবসাইটেে দিয়ে দেন।কোথাও পরিষ্কার pdf পাচ্ছি না) রাইয়ান আল রোহান ৫ম শ্রেণি ২নং চাঁদভা সরকারি প্রাথমিক বিদ্যালয় আটঘরি, পাবনা
উত্তর: নিজের খাদ্য নিজে তৈরী করা এতো সোজা না, অনেক কঠিন। গাছ পারে বলেই অন্য সব প্রাণী টিকে আছে। যদি পৃথিবীতে গাছ না থাকতো কী সর্বনাশ হত চিন্তা করতে পার? “বাচ্চা ভয়ঙ্কর কাচ্চা ভয়ঙ্কর” বইয়ের পিডিএফ আমার কাছেও নাই। যদি কোথাও পাই দিয়ে দেব।

প্রশ্ন: সহজভাবে কিছু না নিতে পারা কি কোনো রোগ?মানে,আমাকে নিতান্তই সহজ কিছু বললে বা আমি কিছু পড়লে মনে হয় ব্যাপারটা বোধহয় এত সহজ না।ঘাপলা আছে।তারপর ঘাপলা খুজতে খুজতেই সময় নষ্ট।স্যার,এ থেকে উদ্ধারের কোনো উপায় কি আছে?
**নাম ঠিকানা নেই, তাই উত্তরও নেই! সরি।

প্রশ্ন: খুবই সাধারণ প্রশ্ন ! স্যার কেমন আছেন? শরীরের অবস্থা কেমন? অভি
উত্তর: খুবই সাধারণ উত্তর: ভালো আছি। শরীরের অবস্থাও ভালো।

প্রশ্ন: স্যার, ১,আপনি সকালে ঘুম থেকে উঠার পর রাতে ঘুমাতে যাওয়ার আগ পর্যন্ত কি কি করেন??আমার অনেক জানতে ইচ্ছা করে আপনি এত কিছু মেনেজ করেন কিভাবে?জানি বলা টা কঠিন তবু প্লিজ প্লিজ বলেন তো মানে যেদিন উত্তরটা দিচ্ছেন সেদিন ই কি কি করেছেন। ২,আমার একটা আবদার আছে আপনার কাছে।কিন্তু আপনার এখানে আমি কিছু চাইলে পরে সবাই সেটা চায়।যেমন অটোগ্রাফ। তাই উওর দেওয়ার সময় আমার আবদারটা কেটে দিয়েন প্লিজ।।।… অবন্তি, ময়মনসিংহ।
উত্তর: যখন তোমার সাথে দেখা হবে তখন জেনে নিও। এখানে বসে বসে লিখতে চেষ্টা করলে রাত ভোর হয়ে যাবে। (তোমার আবদারটা কেটে দিলাম!)

প্রশ্ন: শ্রদ্ধেয় স‍্যার, আশা করি আপনি ভালো আছেন। একটা অনুরোধ করব। আপনি কি আমার অনুরোধ টি দয়া করে রাখবেন? আমি চাই আপনি এমন একটি বই লেখেন যেখানে ট্রাইটন একটি গ্রহের নাম বইয়ের চরিত্রগুলো উপস্থিত থাকে। স‍্যার, প্লিজ আপনি এরকম একটি বই লেখেন। ইতি, আপনার ভক্ত। দশম শ্রেণি।
উত্তর: সেই কবে “ট্রাইটন একটি গ্রহের নাম” বইটা লিখে ছিলাম (মনে হয় তোমাদের জন্মেরও আগে!), এখন আমার বইয়ের কাহিনীটা মনে পর্যন্ত নাই!

প্রশ্ন: একটি বস্তুকে কেন রঙিন দেখায় ? Thanapara,Gaibanaha
উত্তর: https://www.sciencelearn.org.nz/resources/47-colours-of-light নেটে এটা খুঁজে দেখ। আমি চাই তোমার সব প্রশ্নের উত্তর প্রথমে নেটে খুঁজে দেখবে।

প্রশ্ন: দাদু ডাকার অনুমতি দেওয়ার জন্য অনেক thanks.কিন্তু ডাকতে যে বললে, ডাকলে শুনতে পাবেন তো? তাহলে শুনুন।”জাফর দাদু”।আদ্রিতা,সুনামগন্জ।
উত্তর: বাহ! ভালোই তো শুনাচ্ছে।

প্রশ্ন: Sir class 9 10 er chatroder ke ki dr banano hocche?
**নাম ঠিকানা নেই, তাই উত্তরও নেই! সরি।

প্রশ্ন: আল্লাহ স্যার এটা কি হলো?? আপনি এবার ১তারিখ রাতে উওর দিয়েছেন কিন্তু ১১ই সেপ্টেম্বর পর্যন্ত কিন্তু আমি প্রশ্ন করেছিলাম ১৩তারিখ। তাহলে কি আমি উওর পাবো না???
**নাম ঠিকানা নেই, তাই উত্তরও নেই! সরি।

প্রশ্ন: ধন্যবাদ স্যার ছবিটার জন্য। জি স্যার চিনতে পেরেছি। আপনাকে চিনব না???আপনি পিছনে দারিয়ে আছেন বাম দিক থেকে দ্বিতীয়। হয়েছে?? স্মৃতি, ময়মনসিংহ
উত্তর: হ্যাঁ, উত্তর ঠিক হয়েছে। এ প্লাস।

প্রশ্ন: স্যার,কিছু উপদেশ দেন।বাণী ও দিতে পারেন।
**নাম ঠিকানা নেই, তাই উপদেশও নেই, বাণীও নেই! সরি।

প্রশ্ন: আসসালামু আলাইকুম স্যার, আপনার লেখা টুনটুনি ও ছোটাচ্চু সিরিজের প্রথম দুইটা বই খুবই ইন্টারেস্টিং,কিন্তু তৃতীয় বইটিতে আপনি একটি চরিত্র(ফারিয়া)বাদ দিয়ে দিছেন যার জন্য সেই বইটা বেশি ভালো লাগল না। আপনি এই সিরিজের নতুন বই প্রকাশ করবেন কি? করলে অবশ্যই ফারিয়া চরিত্রটি দিবেন।
আপনি আমার নামের ও কোনো চরিত্র দিতে পারেন কারন খুবই সুন্দর নাম আমার-
শায়েখ আল হাসান, গাজীপুর।
উত্তর: থ্যাঙ্ক ইউ। টুনটুনি ও ছোটাচ্চু অনেক লেখা হয়েছে আরো লেখার ধৈর্য আছে বলে মনে হয় না।

প্রশ্ন: স্যার তাহলে আমার ঠিকানাও পাঠিয়ে দেই? আমার জন্যও একটা অটোগ্রাফ পাঠাবেন, প্লীজ। নাহিন,লক্ষীপুর।
উত্তর: পাঠিয়ে দাও।

প্রশ্ন: কিভাবে আপনাকে কোনো ছবি পাঠাতে পারি । ঋতি Ishwardi,pabna
উত্তর: একটা লিঙ্ক দাও, আমি ডাউনলোড করে নিই।

প্রশ্ন: আপনার একটা ফানি ছবি দেখবেন please!!!!
**নাম ঠিকানা নেই, তাই ফানি ছবিও নেই! সরি।

প্রশ্ন: Sir ,আপনি এই ওয়েবসাইট টাতে এত কম বই দিয়েছেন কেন ?আর যেগুলো দিয়েছেন তা সবই পড়া হয়ে গেছে ।এখন আমি বসে বসে কি পড়ব বলুন তো! আর আমার ঠিকানাটি দিয়েছি ,আপনি একটা otograph দিবেন .please!!!
নাম- ঋতি ( সবাই গুলিয়ে ফেলে রীতির সাথে) ঠিকানা-Ishwardi,pabna….. ( দরকার হলে বলবেন আরেক বার ঠিকানা দিব)
উত্তর: ঈশ্বরদী তো অনেক বড় জায়গা, শুধু তোমার নাম দিলে চিঠি চলে যাবে?

প্রশ্ন: Ami jotodur jani apni bangla English e pochondo koren na.kintu koshto kore pore Niben.ami patuakhali te thaki.Shekhane PGJHS er chatro.Class 9 e uthe dekhlam je sob ma babara science neyar jonno pagol.prothom Jedin sir Bollo je ke ke science nite chao.tokhon class e ekta hirik pore gelo.commerce o arts e chilo matro 5 Jon.apnar Ki mone hoy na ei desh ki macher bazaar hoye geche.science na porle brainless.kono future nai egula Ki?
**নাম ঠিকানা নেই, তাই উত্তরও নেই! সরি।

প্রশ্ন: Hi uncle i like your stories l just love your stories especially Chotasso and Toni
**নাম ঠিকানা নেই, তাই উত্তরও নেই! সরি।

প্রশ্ন: স্যার,,,এই পেইজটিতে আপনার টিভি নাটকগুলো সংযুক্ত করে দেওয়া যায় না??? অথবা নাটকের গল্পগুলি??? জান্নাতুল নাঈমা রাফিয়া,,, মাদারিপুর।
উত্তর: রক্ষে কর প্লিজ! এই ভাঙ্গাচুরা ওয়েব সাইট ম্যানেজ করতেই আমার জান বের হয়ে যাচ্ছে, তার উপর আবার নাটক? আমার নাটক আমি কোথায় খুঁজে পাব তাও জানি না।

প্রশ্ন: স্যার: আপনি কী এখনও SUST ক্লাস করান? আমি এবছর ভর্তী পরিক্ষা দিবো SUST এ! মো.সুজন,ধানমন্ডি, ঢাকা।
উত্তর: না। নিয়মিত আর ক্লাশ করাতে হয় না। আশা করি সাস্টে তোমার পছন্দের একটি বিভাগে তুমি ভর্তি হতে পারবে।

প্রশ্ন: আপনি কি ২৭ সেপ্টেম্বর এ ন্যাশনাল বিকাশ বিজ্ঞান উতসব এ আসবেন?
**নাম ঠিকানা নেই, তাই উত্তরও নেই! সরি।